0% found this document useful (0 votes)
1K views184 pages

BarBri - MBE Questions

The subcontractor may assert a claim in the pending action against the developer seeking payment for cost overruns. The patron cannot assert a third-party claim against the bouncer. The court should grant the motion to dismiss because the amended complaint substituting the actual manufacturer was filed after the statute of limitations expired.

Uploaded by

Michael F
Copyright
© © All Rights Reserved
We take content rights seriously. If you suspect this is your content, claim it here.
Available Formats
Download as PDF, TXT or read online on Scribd
0% found this document useful (0 votes)
1K views184 pages

BarBri - MBE Questions

The subcontractor may assert a claim in the pending action against the developer seeking payment for cost overruns. The patron cannot assert a third-party claim against the bouncer. The court should grant the motion to dismiss because the amended complaint substituting the actual manufacturer was filed after the statute of limitations expired.

Uploaded by

Michael F
Copyright
© © All Rights Reserved
We take content rights seriously. If you suspect this is your content, claim it here.
Available Formats
Download as PDF, TXT or read online on Scribd
You are on page 1/ 184

Barbri MBE questions

Study online at https://quizlet.com/_6umlok

1. A developer contracted with a general con- Yes, because the subcon-


tractor to build an office building, and com- tractor's claim against the
pletion of the building was two years late. developer arises form the
The developer filed a breach of contract ac- same transaction or occur-
tion in federal district court against the gen- rence as the developer's
eral contractor, seeking damaged caused original claim, but the sub-
by the delay. The general contractor filed contractor may assert the
a third-party claim against a major sub- claim in an independent ac-
contractor, claiming that the subcontractor tion if it prefers.
caused any delay and should be liable to the
general contractor for anything the gener-
al contractor has to pay the developer. The
subcontractor believes that the developer
interfered with the subcontract and that the
developer's interference caused not only
the delay but also substantial cost overruns
for the subcontractor.

May the subcontractor assert a claim in the


pending action against the developer seek-
ing payment for the cost overruns?

2. A tourist from State A was severely injured No, because the patron has
in a bar fight in State B. The tourist filed no legal basis to assert a
a battery action against one of the bar's claim against the bouncer
patrons, seeking $100,000 for his injuries. (patron is denying liability all
The defendant patron claims that the tourist together) and is not seeking
is mistaken about who hit him. The patron to recover from the bouncer
says that he did not touch the tourist. The any portion of the patron's
patron claims that it was the bar's bounc- liability to the tourist.
er--who looks like the patron-- who hit the
tourist and then continued to pummel him.
The bouncer claims that he never touched
or harmed the tourist.

Can the patron assert a third-party claim


against the bouncer to bring him into the
action?

1 / 184
Barbri MBE questions
Study online at https://quizlet.com/_6umlok
3. A homeowner discovered that the siding on Grant the motion, because
his house was defective and had allowed the amended complaint was
water to enter the structure, causing dam- filed after the SOL expired
age to the wood framing. The homeowner and the actual manufactur-
tried for some time to negotiate a settlement er did not receive timely no-
with the corporation that the homeowner tice of the action. (R. Amend-
believed had manufactured the defective ments to claims substituting
siding. When no settlement was forthcom- a new defendant are allowed
ing, the homeowner filed an action in fed- and relate back if (1) claims
eral district court against the corporation arise from the same transac-
one week before the SOL expired. Service tion or occurrence; (2) with-
of process was effected on the corpora- in the time allotted for serv-
tion several months later. After inspecting ing the original complaint [90
the home, the corporation filed and served days of filing]; (3) so that they
its answer in which it denied manufactur- received such notice that a
ing the siding used on the homeowners claim was against them so
house. Upon examining the corporation's that they would not be over-
evidence, the homeowner conceded that the ly prejudiced; and (4) knew
siding was manufactured by another com- or should have known there
pany. With leave of court, the homeowner would be an action against
then filed an amended complaint substi- them.)
tuting the actual manufacturer of the sid-
ing for the original incorrect defendant. The
amended complaint was served on the man-
ufacturer approximately seven months after
the original complaint was filed and after the
SOL had expired. The manufacturer was un-
aware of the action until it was served with
the amended complaint. The manufacturer
filed a motion for SJ on the grounds that the
homeowner's claim against it is barred by
the SOL.

How should the court rule on the motion?

4. A motorist from State A struck and injured No, because the federal
a pedestrian in State B. The pedestrian, a rules permit service under
state B resident, brought an action in a the rules of the state in which
State B federal court against the State A service will be effected (R.
2 / 184
Barbri MBE questions
Study online at https://quizlet.com/_6umlok
motorist, seeking $100,000 in damages. The Service can be made as al-
summons and complaint were served on a lowed by the rules of the
receptionist at the motorist's place of busi- state where the fed. court
ness in State A. State A's rules permit ser- sits or the state where ser-
vice of process in this manner, while State vice was to be affected.)
B's rules do not.

If the motorist moves to dismiss the com-


plaint on the basis of improper service of
process, is the court likely to dismiss the
action?

5. A customer slipped and fell in a store, suf- No, because the manager's
fering a severe injury. Several weeks after notes are protected by a
the accident, anticipating that the customer qualified immunity from dis-
would file an action against it, the store's covery under the work prod-
attorney had the store manager interview uct doctrine (R. Work prod-
any employees who were near the accident uct doctrine is a qualified im-
to determine what they saw or heard. The munity [not absolute immuni-
store manager did so, taking handwritten ty]; not have to be produced
notes. The notes are now in the store's pos- unless it can be shown that
session. The customer subsequently filed there is a substantial need or
a civil action against the store in federal an undue hardship)
district court. The complaint alleged that the
store negligently left a spill on the floor of
the store, causing the customer's fall. The
customer's attorney served on the store a
request for production of documents, which
included a request for all documents and
reports prepared by the store that relate to
the customer fall and injury.

Must the store produce to the customer the


notes taken by the store manger when he
interviewed the store's employees?

6. A merchant owned a skate rental busi- Reach the merits of the mer-
ness that she operated out of a specially chant's challenge because
equipped van. She would drive to various enforcement of the ordi-

3 / 184
Barbri MBE questions
Study online at https://quizlet.com/_6umlok
parks and public beaches within her home nance will harm her busi-
state and rent roller skates, related safety ness and the rights of the
equipment and lightweight stereo/earphone public are linked to her rights
sets to passerby on an hourly basis. She (R. A seller of goods may
also sold skates and skating equipment. have third party standing to
About 50% of the merchant's time is spent challenge a law that ad-
in a single city, and she earns about 70% versely affects the rights
of her gross rental and sale income at that of her customers. [standing
city's beach areas. After receiving numer- plus this])
ous complaints from beach goers about the
sidewalks congested with roller skates, the
city council passed an ordinance prohibit-
ing roller skating on public property be-
tween the hours of 7 am and 9 pm

If the merchant seeks to enjoin enforcement


of the ordinance in federal district court on
the basis that it is unconstitutional, what
should the court do?

7. Recently enacted legislation required farm- Deny the petition, because


ers in certain counties of a western state to there is no substantial feder-
use drip irrigation systems instead of tradi- al question that is dispositive
tional methods in order to conserve water of the case. (Would be an ad-
for agricultural and other uses. A farmer visory opinion if State Court
who refused to use the drip system was has already ruled that the
charged pursuant to the enforcement pro- law violates the State Con-
visions of the legislation. A state court en- stitution and this was decid-
joined him from using other irrigation meth- ed on adequate and inde-
ods and fined him. pendent state grounds) (R.
When may a Supreme Court
The farmer appealed to the state supreme rule on a State court de-
court, renewing his trial court claims that cision? (1) Has to be a fi-
the irrigation legislation violated a state nal judgment from a state
constitutional provision prohibiting certain court; (2) Had to come from
governmental intrusions into private com- highest state court; (3) Sub-
mercial activities and that it was preempted stantial federal question be-
by federal water management statutes. The ing raised; and (4) Decision
state supreme court held that the state con- from state court did not rest
4 / 184
Barbri MBE questions
Study online at https://quizlet.com/_6umlok
stitution prohibited the challenged legisla- on adequate and indepen-
tion, and construed the relevant statutes as dent state grounds.)
being within the parameters of the federal
statutes, and thus preempted.

If the state petitions for certiorari to the US


Supreme Court, how should the Court rule
on the petition?

8. It was common practice in a particular state As applied to the loans out-


for a security interest in land to be struc- standing at the time the bill
tured as a deed absolute, which gave a was enacted, the law im-
lender absolute title to the borrower's prop- pairs the contract rights of
erty as security for the loan. The lender the lenders and such rights
would reconvey only on complete payment are guaranteed by the Con-
of the loan by the debtor party, and could tracts Clause of the feder-
dispose of the land immediately without a al Constitution. (R. Contracts
foreclosure sale on default. A new gover- clause only applies to state
nor of the state whose campaign platform laws that retroactively inval-
was built around abolishing the deed ab- idate already existing con-
solute mortgage encouraged the legislature tracts; cannot do so unless
to enact a bill that immediately outlawed the government act serves
use of the deed absolute, declaring that all an important government in-
such deeds would be considered mere liens terest and state law is nar-
against the secured property. The law ap- rowly tailored to that govern-
plied not only to loans made in the future, ment interest.)
but also to the thousands of such loans
in existence at the time the legislation was
passed. As soon as the governor signed
the legislation, lending institutions and in-
dividuals who had loaned money secured
through deeds absolute challenged the con-
stitutionality of the new law.

What is the strongest argument that the


challengers can make?

9. A philanthropist told his friend, who was The policy is constitutional,


a state governor, that he planned to build because the museum is a

5 / 184
Barbri MBE questions
Study online at https://quizlet.com/_6umlok
a museum. The governor thought that the private entity and so may
museum would bolster the state's tourism constitutionally hire and fire
industry and offered to arrange to have the as it desires. (Just giving
state purchase land and grant it to the mu- land is not enough for state
seum to enable the philanthropist to build a action) (R. If a private citizen
bigger museum with his money than orig- is discriminating, it is usually
inally planned. The philanthropist agreed, not going to violate an indi-
and the museum was built. vidual's constitutional rights;
only if the government is sig-
The philanthropist undertook the hiring of nificantly involved in the the
the museum's senior staff. He was of Ger- activity will you find state ac-
man descent and was ashamed of Ger- tion, incidental involvement
many's actions during World War II. To as- is not enough.)
suage his own conscience, he refused to
hire anyone whom he believed to be of Ger-
man descent. A restoration expert applied
for a job as chief curator of the museum,
but the philanthropist refused to hire him
because of his German background. The
restoration expert discovered the philan-
thropist's rationale and brings suit against
the museum, claiming that the hiring prac-
tice violates his constitutional rights.

How is the court most likely to rule?

10. The National Park Service recently created Because most women are
a new personnel level for field employees, less than six feet tall, the
which became the highest salaried position restriction is an invalid dis-
available to Park Service field employees. crimination on the basis of
The position is restricted to employees over gender in violation of the DP
six feet in height. A female ranger who is five Clause of the Fifth Amend-
feet, three inches tall seeks your advice as ment. (R. Equal protection
to whether she can challenge the validity of applies to states, not federal;
the height restriction in federal court.

If you decide to file suit on her behalf, which


of the following would be your strongest
argument against validity of the restriction?
6 / 184
Barbri MBE questions
Study online at https://quizlet.com/_6umlok

11. To encourage minority business and foster In favor of the publisher, be-
pride in minority heritage, a state adopted cause the tax violates the
legislation exempting magazines and oth- 1st Amendment freedoms of
er periodicals from the state's receipts tax speech and press. (R. Con-
if 20% of the magazine is devoted to arti- tent based and viewpoint
cles concerning minorities ( a commission regulations on speech are
was set iup to sample magazines to deter- preemptively unconstitution-
mine on a yearly basis whether they should al and tested under a strict
be exempt.) A publisher produced a sports scrutiny standard of review
magazine in the state that occasionally con- and are rarely upheld)
tained articles about minority athletes, but
the commission determined that the pub-
lisher's magazine was not eligible for the
receipts tax exemption. After paying the tax
assessed on her magazine, the publisher
sued for a refund.

12. The legislature of a state was concerned It is constitutional if it does


that the numerous and strident television, not prohibit the dissemina-
radio, and newspaper advertisements by tion of truthful information
auto dealerships annoy and mislead the about price and the availabil-
public. Therefore, it enacted comprehensive ity of products, and is nar-
legislation regulating the timing and con- rowly tailored to serve a sub-
tent of such ads, limiting their duration, fre-
stantial government inter-
quency, and the types of claims and infor- est. (R. Commercial speech
mation made and given. allowed as long as they
serve a substantial govern-
Which of the following statements is most ment interest; Intermediate
accurate as to the constitutionality of the scrutiny used for: zoning
state's ad regulation? laws on adult movie theaters
and bookstores, nude danc-
ing bands, symbolic speech,
content neutral time place
and manner restrictions.)

13. Auto workers went on strike in a town heav- No, because the anchor did
ily reliant on the auto industry. While negoti- not record the conversation,
ations between the union and management and the information is truth-
were ongoing, a person intercepted and ful and about a matter of
7 / 184
Barbri MBE questions
Study online at https://quizlet.com/_6umlok
recorded a phone call between the union's public significance. (R. Press
president and management's chief negotia- has a right to publish info
tor. A state statute makes it illegal to record
about a matter of public con-
a phone call without the consent of the par- cern and it can only be re-
ties being recorded. The statute also make stricted by a narrowly tai-
is illegal to play an illegally recorded con-lored sanction designed to
versation on television or radio. further a state interest of the
highest order; applies even
The person who recorded the call anony- if the information has been
mously sent the recording to a local TV sta- obtained unlawfully.)
tion. The TV station news anchor played the
recording on air.

Can the anchor who played the recording be


prosecuted under the statute?

14. A father conveyed his property to his son For the sister, because she
and daughter "as joint tenants with right of has a valid right of first re-
surviviorship, but if they ever attempt to sell fusal. (R. rights of first re-
the property during their lifetimes, a right fusal have to follow RAP; not
of first refusal based on the sale price is an unreasonable restraint on
hereby granted to my sister." Unbeknownst alienation if can sell, but
to the son or the sister, the daughter quit- must first offer ROFR ) (Not
claimed her interest in the property to a pur- violate RAP here because
chaser. The following month, the daughter right of first refusal is for the
was killed in a snowmobile accident. The life of the sister, so we will
purchaser of the daughter's interest filed a know during her life whether
suit for partition of the property. The son or not ROFR will be exer-
filed an appropriate counterclaim for quiet cised)
title, asserting that he was the owner of the
entire parcel. The sister also filed a counter-
claim, asserting that her right of first refusal
was valid and that she was prepared to ex-
ercise her right to purchase the property for
the contract price.

In a jurisdiction in which the RAP is unmod-


ified by statute, how should the court rule?

8 / 184
Barbri MBE questions
Study online at https://quizlet.com/_6umlok
15. A tenant entered into a written five-year A year-to-year tenancy at
lease to rent an office from a landlord for $600 (R. Holdover Tenant li-
$6,000 per year beginning October 1. The able under new terms of
lease required that rent in the amount of lease/money amount if ten-
$500 be paid on or before the first of each ant was notified of a rent
month. Two months before the five-year increase prior to the end
term was up, the tenant received a new lease of the original lease; year-to
identical to the one he had already signed, year if commercial and
except that the lease term began on the month-to-month if residen-
upcoming October 1 and the stated amount tial)
of rent per month was $600. The tenant re-
turned the lease to the landlord unsigned,
with a letter stating that he did not intend to
renew the lease and would be moving out on
September 30. The tenant did not move out
on September 30. On October 1, the landlord
received a check for $500 from the tenant.
The notation on the check indicated that it
was for the October rent. The landlord de-
posited the check in her account. She then
sent a letter to the tenant stating that he was
$100 in arrears in his rent. The tenant did not
move out of the office during October, and
the landlord did nothing to remove him.

Most courts would hold that the tenant has


what type of tenancy?

16. An owner of three acres of lakefront proper- The nephew's easement is


ty subdivided it and sold two acres to a buy- a legal interest that the de-
er, retaining the one acre actually fronting veloper had record notice
on the lake. The deed for the two acres of, even though there is
expressly included an easement over the no tract index. (Recorded
westernmost 30 feet of the one-acre par- notice, so no track index
cel retained by the owner for access to the is irrelevant.) (R. Easement
lake. The buyer recorded his deed in the can be created by: express
county recorder's office, which maintained easement, adverse prescrip-
an alphabetical grantor-grantee index only. tion/possession, landlocked;
Fifteen years later, the owner died, leaving
9 / 184
Barbri MBE questions
Study online at https://quizlet.com/_6umlok
the one-acre parcel to his wife. She sold it Easement terminated by:
to a developer that planned to build condo- abandonment,
miniums. A month later, the buyer died, and
his two acres passed by will to his nephew.
Three weeks after taking title to the prop-
erty, the nephew visited the property and
discovered that the developer had erected
a chain link fence all along the boundary
between the nephew's land and the acre of
lakefront land. The nephew brings an action
to enjoin the developer from obstructing his
easement across the acre of lakefront prop-
erty.

Which of the following best describes why


the nephew should prevail in this litigation?

17. A landowner owned two adjoining parcels A profit appurtenant (R. Prof-
of land containing a number of lakes. She it is a nonpossessory inter-
conveyed the eastern parcel, which con- est in land that entitles the
tained a campground, to a fisherman. The profit holder to enter onto
deed transferring the parcel granted to the the servient land to take min-
fisherman "and to invited guests of the erals and substance from
campground all hunting and fishing rights the land [can be appurtenant
and use of the lakes on the western parcel or gross]; Can only trans-
for the benefit of the campground." Subse- fer an appurtenant profit with
quently, the fisherman assigned his hunting the land itself; Can trans-
and fishing rights to a hunter. fer gross profit without trans-
ferring the land itself; Ease-
When the landowner discovered the hunter ment in gross only allows
hunting and fishing on her land, she you to use the land, not take
brought an appropriate action to declare his stuff from the land.)
rights void.

If the court rules for the landowner, it will be


because the fisherman's right to hunt and
fish on the western parcel is.:

18.

10 / 184
Barbri MBE questions
Study online at https://quizlet.com/_6umlok
An investor rented his property to a pottery
The equipment was installed
maker, who intended to use the back part of
for the pottery maker's ex-
the building for living quarters, and the front
clusive benefit and she did
part as a pottery studio. The pottery mak-
not intend for it to stay. (R.
er installed a kiln, some lights, and some
Residential fixture: look at
storage units in the front part for her use.
nature of chattel, intent for
Sometime later, the investor mortgaged the
whether chattel will stay, and
property to a bank to secure a loan. The how much damage will be
mortgage was recorded, but the investor did
caused to the real proper-
not personally tell the pottery maker that he
ty if the chattel is removed;
had done so. In fact, she only learned ofCommercial fixture: we ap-
it when the investor defaulted on the loan
ply the trade fixtures doc-
and the bank foreclosed on the mortgage trine, which allows a tenant
and told the pottery maker that she wouldto remove all trade fixtures
have to quit the premises. The pottery maker
prior to lease expiring. [Ex-
began removing the equipment and fixturesception: accessions cannot
that she had installed in the building. The
be removed, which are struc-
bank objected and sought an injunction totural additions to the prop-
prevent her from doing so. erty like a deck.]) (Here, fix-
tures are on the commercial
Under these circumstances, on what basis part, so they can all be re-
should the court deny the injunction moved prior to the expira-
tion of the lease.) (notice of
there being a mortgage by
landowner on the property is
irrelevant.)

19. A testator executed a will, devising his land The daughter's child and the
"to my son and my daughter, share and son's wife each own an un-
share alike." Shortly thereafter, the daugh- divided one-half interest in
ter died intestate, leaving a child as her only the land. (R. Under common
heir. The next year, the testator and his son law, if a beneficiary prede-
were involved in a car accident The testator ceases the testator, it would
died immediately. The son died six days lat- pass to testator's residuary
er leaving a will that bequeathed his entire unless there is an anti-lapse
estate to his wife. The jurisdiction has the statute.)
following statute: "If a devisee, including a
devisee of a class gift, who is a grandparent
or a lineal descendant of a grandparent of
11 / 184
Barbri MBE questions
Study online at https://quizlet.com/_6umlok
the testator is dead at the time of execution
of the will or fails to survive the testator, the
issue of such deceased devisee shall take
the deceased's share under the will.

Who owns the land.

20. A balloonist sued the manufacturer of defla- Yes, because structural en-
tion panels for hot air balloons after one of gineers reasonably rely on
the panels failed while his balloon was de- such reports in the course of
scending, causing the balloon to crash and their profession. (R. Experts
the balloonist to suffer severe injuries. At tri- do not have to have personal
al, the balloonist calls as a witness a struc- knowledge to make a state-
tural engineer who testifies that, common to ment/opinion; Experts do not
industry practice, her opinion is based on have to offer into evidence
several reports done by an independent lab- reports used to make their
oratory on the burst strength and material statement/opinion.)
composition of the deflation panel closures.
The balloonist's attorney then asks the en-
gineer whether, in her opinion, the closures
caused the deflation panel to give way. The
manufacturer objects.

Should the court admit this testimony

21. Using Character in Criminal Cases (5) 1. Character evidence is in-


admissible unless defendant
opens the door [prosecutor
cannot introduce evidence of
defendant's bad character if
it is offered to show defen-
dant acted in propensity with
this bad character] 2. Defen-
dant is allowed to present
evidence of relevant good
character to show he act-
ed in conformity with good
character and did not com-
mit the crime [reputation or

12 / 184
Barbri MBE questions
Study online at https://quizlet.com/_6umlok
opinion evidence only on di-
rect] 3. If the defendant takes
advantage of 2 to show he
did not commit the crime,
defendant opened the door
and prosecution can rebut
with evidence of defendant's
bad character; 4. Evidence
of prior bad acts are never
admissible to show propen-
sity, but they can be of-
fered to show motive, iden-
tity, absence of mistake, in-
tent, common plan/scheme.
[this evidence is always sub-
ject to FRE 303 regarding
unfair prejudice.] 5. If defen-
dant testifies he automatical-
ly places his character for
truthfulness/credibility at is-
sue.

22. Using Character in Civil Cases (3) 1. Character evidence is in-


admissible unless it is direct-
ly in issue or an essential
element of a plaintiff's claim
or defense. [no propensi-
ty][defamation, negligent en-
trustment, hiring, or reten-
tion, or child custody] 2. If
litigant has some other pur-
pose for introduction of char-
acter and it is relevant, then
the rule prohibiting character
will not keep it out. 3. If de-
fendant testifies he automat-
ically places his character for
truthfulness/credibility at is-
sue.
13 / 184
Barbri MBE questions
Study online at https://quizlet.com/_6umlok

23. The defendant was charged with embez- Yes, to hep show that the
zling $1 million from his employer, a bank, defendant did not embezzle
by transferring the funds to a secret off- funds. (R. Normally you can-
shore account in the bank's name. Only not bolster your witness's
the defendant and the bank's VP were au- credibility before it has been
thorized to draw funds from the account. attacked. Exception: if it
The defendant testified that he had wired is being offered for some
$1 million to the account but had done so other purpose.) (Here, he
at the direction of the bank's VP. The de- was charged with embezzle-
fendant stated under oath that he had no ment, so he is allowed to
intent to embezzle bank funds. The gov- present evidence of his good
ernment's cross-examination of the defen- character reputation to show
dant concentrated exclusively on his rela- he is honest and not the type
tionship and conversations with the VP, who of person that would embez-
has committed suicide. zle.

The defense now seeks to call a second


witness, who is prepared to testify that he
ad worked with the defendant for 10 years
and that the defendant had a reputation in
both the business and general communities
as being a very honest person.

Is the witness's testimony admissible.

24. Five ways to impeach a witness 1. Prior inconsistent state-


ment (if given under oath
it is for both impeachment
but also as substantive evi-
dence; can use extrinsic ev-
idence) 2. Bias or motive to
misrepresent (allows admis-
sible and even allow extrin-
sic evidence if refuted) 3. Pri-
or conviction (crime involv-
ing dishonesty or false state-
ment, it is usable to impeach
in any case and no discretion
by judge to exclude; felony
14 / 184
Barbri MBE questions
Study online at https://quizlet.com/_6umlok
but not exceed 10 years) 4.
Specific acts of misconduct
that bear on truthfulness or
untruthfulness. (can be in-
quired into on cross-exami-
nation, but no extrinsic evi-
dence) 5. Bad reputation for
truth or veracity. (can use ex-
trinsic evidence)

25. At the defendant's trial for grand theft auto No, because it is inadmis-
and other offenses, the prosecution offers sible hearsay (Not admissi-
to introduce the testimony of a police offi- ble under prior identification
cer. The officer will testify that he showed because the witness is not
a photographic lineup containing the defen- available to testify, so we do
dant's picture to a witness who had seen the not know if the witness is ly-
defendant feeling form the stolen vehicle at ing or not.)
the conclusion of a high-speed chase, and
the witness selected the defendant's pic-
ture. The witness has left the state and she
refuses to return.

Should the court admit the evidence?

26. The victim collapsed at her desk while Inadmissible, because it is


drinking her morning coffee and was rushed hearsay not within any ex-
to the hospital. Later that night, the victim's ception. (R.Dying declara-
brother went to visit the victim in the inten- tion 4 requirements 1. un-
sive care unit. Barely conscious, the victim availability on part of the de-
said "I've thought about this all day and it clarant at trial; 2 Belief that
must have been my assistant. She brought death is imminent; 3 state-
me my coffee this morning before I could ment concerning the cause
make it for myself, and she's never done that or circumstances surround-
before. Don't let her get away with murder." ing the belief of death; and 4.
The victim soon lost consciousness and applies in any civil case and
lapsed into a coma, and she remains in this in criminal homicide cases.)
vegetative state. It was determined that she (Therefore, here it would not
was poisoned. The assistant is arrested and count as a dying declaration
charged with attempted murder. because it is a criminal case

15 / 184
Barbri MBE questions
Study online at https://quizlet.com/_6umlok
that is for attempted murder,
At the assistant's trial, the prosecution not homicide.)
wishes to call the victim's brother to testify
to the victim's statement about the assis-
tant.

The court should find the statement:

27. On an icy day, a vehicle driven by the de- Yes, because it is a state-
fendant struck the plaintiff's car in the rear, ment by an opposing party.
smashing a taillight and denting the plain- (R. Statement by opposing
tiff's bumper. Before the plaintiff could say party being offered against
anything, the defendant rushed out of his him functions as an admis-
car and told the plaintiff, "Look, if you'll take sion and is admissible.)
$500 for the damage, I'm sure my insurance
company will pay for it." The plaintiff re-
fused and sued the defendant for damage
to his car and minor personal injuries. The
plaintiff wishes to testify as to the defen-
dant's statement at the time of the accident.
The defendant objects.

Should the court allow the defendant's


statement to be admitted?

28. The plaintiff sued the defendant dry clean- The coat is admissible
er, claiming that it had permanently ruined based on the plaintiff's tes-
her $10,000 mink coat by cleaning it with a timony (R. Before an object
solvent that left an extremely offensive odor is admissible at trial, the pro-
that smelled like "skunk." Further attempts ponent of the evidence must
to have the odor removed by other cleaning present some additional ev-
services were unsuccessful. The odor was idence to establish that the
so bad that she could no longer wear the object is what the proponent
coat. claims it is; you DO NOT
HAVE to present extrinsic
At the trial, the plaintiff testified to the above evidence.)
facts. She then identified a mink coat as
her coat that the defendant had ruined. She
testified that it still smelled the same as it

16 / 184
Barbri MBE questions
Study online at https://quizlet.com/_6umlok
did after the defendant had cleaned it. The
plaintiff's counsel offered to introduce the
coat for the purpose of having the jury smell
it. Defense counsel objected.

How should the court rule?

29. A patient was scheduled to undergo non The patient, regardless of


emergency surgery for the removal of her whether she establishes
appendix by her family doctor. The day damages at trial because
of the surgery, the doctor was called out there was no consent by pa-
of town because of a family illness. Even tient. (There was no emer-
though the surgery could be postponed, the gency and no consent)
doctor asked the surgeon on call, who was
an expert in appendectomies, to take his
place. The patient was not informed of the
switch in doctors.

If the patient sues the surgeon on a battery


theory, who will prevail?

30. An American tourist was visiting another The passenger did not suffer
country when he was warned by US health physical injury from her dis-
authorities to go immediately to a hospi- tress. (R. Must suffer physi-
tal because he had a serious and extreme- cal harm or injury for NIED
ly contagious disease that required him to [Exceptions: 1. mishandling
be quarantined. He decided to ignore the a corpse; or 2. False report
warning and instead traveled on an airline of death])
flight back to the US. Despite the tourist's
belief that he would not be discovered and
his best efforts to keep a low profile, the
news media were tipped off to what he had
done and publicized it. When a passenger
who had been sitting next to the tourist on
the plane learned about it, she became ex-
tremely upset, fearing that she would con-
tract the disease. The passenger brought a
negligence action to recover for the distress

17 / 184
Barbri MBE questions
Study online at https://quizlet.com/_6umlok
she suffered but the jury rejected her claim.
Why?

31. A landowner who had owned and operat- Yes, because the electric
ed a small airport notified the electric com- company used unreason-
pany that he was discontinuing operations able force to protect its prop-
and that it should shut down the electrical erty. (R. You cannot use
current that had supplied his communica- deadly force to protect prop-
tions equipment. The equipment had been erty.)
surrounded by a fence and signs warning
of high voltage. Because the electric com-
pany had maintained a transformer next
to the landowner's communications equip-
ment that contained many valuable and
reusable parts, it decided to leave the power
on to prevent theft until it could schedule
removal of the transformer. Three days later,
a trespasser who knew that the airport had
closed went onto the property looking for
something to steal. He could find nothing
of value except the transformer. He noticed
the signs warning of the high voltage but be-
lieved that the power had since been turned
off. He scaled the fence with the intent to
dismantle the transformer. As soon as he
touched the transformer, he was seriously
injured by the electric current.

If the trespasser asserts a claim against the


electric company to recover damages for
his injuries, will he prevail?

32. A state statute prohibits leaving a child No, because the jury could
under the age of five years unattended in find that it was foreseeable
an automobile. A mother parked her car that the son would cause
at a supermarket parking lot. She left her damage to cars in the park-
four-year-old son in the car with his seat ing lot if the mother left him
belt fastened while she did her grocery unattended. (R. Negligence
shopping. While the mother was shopping, per se should only be picked

18 / 184
Barbri MBE questions
Study online at https://quizlet.com/_6umlok
the son undid his seat belt, left the car, if statute designed to prevent
and started riding on the grocery carts that that type of harm and protect
customers had left in the parking lot. The that particular class.)
son crashed one of the carts into another
shopper's car, causing damages. The shop-
per brought a negligence action against the
mother to recover for the damage caused
by the son. At trial, the shopper presented
evidence of the statute and the facts stated
above. At the conclusion of the shopper's
case, the mother moved for a directed ver-
dict in her favor.

Should the court grant the mother's mo-


tion?

33. After leaving ceremonies at which the chief The associate justice made
justice of a state supreme court had been the statements knowing they
named distinguished jurist of the year, an were false. (R. Defamation:
associate justice was interviewed by the False statement by the de-
press. The associate justice told a reporter fendant that injures the plain-
that the chief justice "is a senile imbecile tiff's reputation; Public figure:
who lets his clerks write all his opinions. He must show published defam-
hasn't had a lucid thought in decades, and atory statement with actual
he became a judge by being on the payroll of malice [knowledge of falsity
the mob." Enraged, the chief justice brought or reckless disregard of the
an action for defamation against the associ- truth] ;Public concern/mat-
ate justice. ter: only have to show
negligence and damages
Which of the following, if established by the [4 per se situations where
chief justice in his defamation action, would damages are presumed: 1.
permit recovery against the associate jus- Loathsome disease 2. Im-
tice? pugning a women's chastity
when it is not true 3. Accus-
ing someone of committing
a crime of moral turpitude 4.
Impugning someone's pro-
fession, trade or business])

19 / 184
Barbri MBE questions
Study online at https://quizlet.com/_6umlok
34. A small processor of specialized steel (A)
agreed in writing with a small manufacturer
of children's toys that it would supply, and Because the large toymaker
the manufacturer would buy, all of the man- acts in good faith in setting
ufacturer's specialized steel requirements its requirements to approx-
over a period of years at a set price per imately those of the small
ton of steel. Their contract did not include manufacturer into whose
a nonassignment clause. Recently, the toy shoes it stepped, the con-
manufacturer decided to abandon its line tract may be assigned. The
of steel toys, so it made an assignment of contract in this question is
its rights and delegation of its duties un- a "requirements" contract:
der the contract to a toymaker many times The steel processor must
larger. The large toymaker notified the steel sell the small manufactur-
processor of the assignment and relayed to er of children's toys all the
the processor its good faith belief that its specialized steel it requires
requirements will approximate those of the for its toys. Generally, the
assignor. right to receive goods un-
der a requirements contract
Must the steel processor supply the require- is not assignable because
ments of the large toymaker? the obligor's duties could
change significantly. In fact,
(A) Yes, because there was no nonassign- here, a significant change
ment clause in the contract. would seem possible be-
cause the large toymaker
(B) Yes, because the large toymaker acted in is a larger company than
good faith to assure the steel processor that the small manufacturer and
its requirements will approximate those of its needs could be greater.
the small manufacturer into whose shoes it However, the UCC allows
stepped. the assignment of require-
ments contracts if the as-
(C) No, because requirements contracts are signee acts in good faith not
not assignable under the UCC to alter the terms of the con-
tract. [UCC 2-306] (The UCC
(D) No, because the steel processor did not applies here because goods
give prior approval of the assignment. are involved.) Thus, assum-
ing the large toymaker's re-
quirements remain about the
same as the small manu-
facturer's requirements, the
20 / 184
Barbri MBE questions
Study online at https://quizlet.com/_6umlok
steel processor would be
required to honor its con-
tract, now assigned to the
large toymaker. (A) is wrong
because requirements con-
tracts may be nonassigna-
ble, even without a nonas-
signment clause. Thus, the
clause would be irrelevant.
The only thing that could al-
low assignment of a require-
ments contract is a good
faith limitation, as addressed
in choice (B). (C) is wrong
because the UCC does al-
low requirements contracts
to be assigned, as long as
the good faith limitation is
satisfied. (D) is similarly in-
correct. The UCC would al-
low assignment without ap-
proval by the obligor if there
is a good faith limitation on
the requirements.

35. A hockey player who was playing in the (C)


final game of the season before a hostile
crowd in the opponent's packed stadium The fan will prevail in his
had an opportunity to get his team into the battery action because the
playoffs, but he missed a shot into an open player had the requisite in-
net as the horn sounded, ending the game. tent for battery. A prima fa-
As the crowd cheered and jeered, the puck cie case for battery requires
bounced back to him and he shot it in anger plaintiff to prove (i) an act by
toward the stands. A fan who had been look- defendant that brings about
ing the other way turned back toward the a harmful or offensive con-
rink just in time to be struck in the face by tact to the plaintiff's per-
the puck. He suffered a broken nose and a son, (ii) intent on defendant's
severe gash under his eye. After the game, part to bring about harm-
the league commissioner fined the player ful or offensive contact, and
21 / 184
Barbri MBE questions
Study online at https://quizlet.com/_6umlok
for violating league rules by intentionally (iii) causation. The intent el-
directing the puck out of the playing area. ement is satisfied as long
as the defendant knew with
If the fan sues the player for battery, will the substantial certainty that the
fan likely prevail? harmful or offensive contact
would result. Here, the play-
(A) No, because by attending a hockey er's conduct caused a harm-
game, the fan assumed the risk of pucks ful contact to the fan, be-
being shot into the stands. cause the player set into mo-
tion the force that caused
(B) No, because the player did not have the injury to the fan. His inten-
intent to strike the fan with the puck. tionally shooting the puck
into the crowded stands is
(C) Yes, because the player knew that it was enough to establish that he
substantially certain that a fan would be hit knew with substantial cer-
by the puck. tainty that the puck would
strike a spectator.
(D) Yes, because the player violated league
rules by intentionally shooting the puck out
of the playing area.

36. A retailer entered into an oral contract with (C)


an office supply wholesaler to buy 100 file
boxes for an upcoming back to school sale The wholesaler may not re-
at the retailer's store. The wholesaler agreed
cover damages. Under the
to deliver the file boxes in two weeks at aUCC Statute of Frauds, a
cost of $4 per file box. A week later, the contract for the sale of goods
retailer phoned the wholesaler and asked iffor $500 or more is unen-
she could increase her order to 200 file box-
forceable unless evidenced
es. The wholesaler agreed. The wholesaler by a writing signed by the
delivered the 200 file boxes as promised, but
party sought to be held li-
the retailer accepted only 150 upon discov-able. The original contract
ering that she lacked storage space for allwas for $400 and, thus,
200. was not within the Statute.
Whether a modification must
May the wholesaler recover damages with be in writing to be enforce-
respect to the 50 file boxes that were not able depends on whether
accepted? the entire contract price
as modified is within the
22 / 184
Barbri MBE questions
Study online at https://quizlet.com/_6umlok
(A) Yes, because the retailer accepted $600 Statute. Here, the retailer
worth of file boxes. and the wholesaler modified
their original contract to 200
(B) Yes, because the modification was for file boxes, bringing the to-
less than $500. tal price to $800. Thus, the
modification was unenforce-
(C) No, because the contract as modified able under the Statute of
was for $800. Frauds, and the wholesaler
cannot collect damages with
(D) No, because the wholesaler is a mer- respect to the 50 unaccept-
chant with respect to file boxes. ed file boxes. (B) is incorrect
because, as noted above,
when determining whether a
contract for the sale of goods
is enforceable, we look at the
whole contract price as mod-
ified; the price of the modi-
fication itself does not mat-
ter. (D) is incorrect because
the fact that the wholesaler
is a merchant with respect
to the goods being sold (file
boxes) has no bearing on the
enforceability of the contract
here.

37. In a criminal battery case brought against (D)


the defendant, the prosecutor asked the
court to take judicial notice of the fact that The effect of the judge's
a car driven from Chicago to Detroit has noticing that a car driven
to cross state lines. The defense attorney from Chicago to Detroit must
raised no objection, and the judge declared cross state lines is that the
that she was taking judicial notice of the fact judge will now instruct the
as requested by the prosecution. jury that it may, but is not
required to, accept that fact
What is the effect of such judicial notice? as conclusively proven. Un-
der the Federal Rules, in a
(A) To raise an irrebuttable presumption. civil case, the court must in-
struct the jury to accept the
23 / 184
Barbri MBE questions
Study online at https://quizlet.com/_6umlok
(B) To satisfy the prosecutor's burden of judicially noticed fact as con-
persuasion on that issue. clusive. [Fed. R. Evid. 201(f)]
Because this question deals
(C) To shift the burden of persuasion on that with a prosecution for crim-
issue to the defendant. inal battery, the applicable
rule is that the jury be in-
(D) That the judge should instruct the jury structed that the fact that has
that it may, but is not required to, accept the been judicially noticed may
noticed fact as conclusively proven. be accepted by it as con-
clusive, but that the jury is
not required to do so. (A)
would be correct if this were
a civil case. In such an in-
stance, the jury would be in-
structed to accept as con-
clusive the judicially noticed
fact. This would have the ef-
fect of raising an irrebuttable
presumption. (B) is incorrect
because, in a criminal case,
the prosecution has the bur-
den of proving every ele-
ment of the crime beyond a
reasonable doubt. Only the
jury can decide, after all of
the evidence is in, whether
the burden of persuasion is
satisfied.

38. A landowner possessed a 40-acre tract of (C)


land. He had inherited 30 acres and had pos-
sessed the other 10 acres for longer than The landowner will win be-
the statutory period necessary to acquire cause the terms of the deed,
title by adverse possession from a ranch- not of the contract, control
er. The landowner entered into a land sale his liability. There is an im-
contract promising to convey the 40 acres plied covenant in every land
to a developer. The contract provided that sale contract that at clos-
the landowner would convey marketable ti- ing the seller will provide the
tle. The developer paid the landowner the buyer with a title that is "mar-
24 / 184
Barbri MBE questions
Study online at https://quizlet.com/_6umlok
purchase price and accepted a deed from ketable." Marketable title is
him. The developer promptly recorded the title reasonably free from
deed. The rancher, having learned of the doubt, i.e., title that a reason-
sale, brought a successful action against ably prudent buyer would be
the developer to quiet title. The developer willing to accept. It need not
realized for the first time that there were no
be a "perfect" title, but the
covenants for title in his deed. The develop-title must be free from ques-
er brings an action against the landowner. tions that might present an
unreasonable risk of litiga-
What is the most likely outcome of the suit? tion. Generally, this means
an unencumbered fee sim-
A The developer will win, because the ple with good record title.
landowner breached the terms of the con- Generally, a title acquired by
tract. adverse possession is not
considered marketable be-
B The developer will win, because the cause the purchaser might
landowner misrepresented the size of the be later forced to defend
tract. in court the facts that gave
rise to the adverse posses-
C The landowner will win, because the terms sion against the record own-
of the deed control his liability. er. Here, the marketability re-
quirement did not have to be
D The landowner will win, because the de- implied, it was an express
veloper was negligent in not checking the term of the contract. Un-
covenants of title at the time of closing. der the doctrine of merger,
the contract merges into the
deed, and the terms of the
contract are meaningless.
Even though the contract
specified a "good and mar-
ketable title," it is the deed
that controls, and the deed
contained no covenants of ti-
tle. A deed does not incorpo-
rate the title terms of a con-
tract. Thus, (A) is wrong. (B)
is wrong; it is not supported
by the facts. (D) is wrong be-

25 / 184
Barbri MBE questions
Study online at https://quizlet.com/_6umlok
cause the developer's negli-
gence is irrelevant.

39. A defendant charged with driving while in- (D)


toxicated pleaded not guilty and insisted on
a trial. Right before the trial began, he fired The judge must raise the is-
his attorney and decided to defend himself. sue of competency. If it ap-
At one point during opening arguments, the pears to the judge that the
defendant began to act like a cat, meowing defendant might be incom-
and chasing an imaginary squirrel out of the petent, the judge has a con-
courtroom. stitutional obligation to con-
duct further inquiry and de-
If no one else raises the issue of the defen- termine whether in fact the
dant's competency to stand trial, what is the defendant is incompetent. If
responsibility of the trial judge here? the defendant is tried and
convicted but it later ap-
A The trial judge has no responsibility, be- pears that he was incom-
cause the defendant decided to defend him- petent to stand trial, the
self. judge's failure to raise the
issue or to request a de-
B The trial judge has no responsibility, be- termination of competency
cause she cannot decide whether the defen- does not constitute a waiv-
dant is competent to stand trial. er of the competency issue.
Therefore, if the trial judge
C The trial judge must raise the issue of observes the defendant act-
competency, because the defendant is rep- ing in such a way that may
resenting himself. indicate he is incompetent
to stand trial (e.g., meow-
D The trial judge must raise the issue of ing, chasing imaginary squir-
competency, because the Constitution ob- rels), she should conduct
ligates her to do so. further inquiry to determine
the competency of the de-
fendant. (C) is incorrect. If
there is evidence that the de-
fendant might not be compe-
tent to stand trial, the judge
should conduct an indepen-
dent inquiry into competen-
cy regardless of whether
26 / 184
Barbri MBE questions
Study online at https://quizlet.com/_6umlok
the defendant is represent-
ing himself or is represented
by an attorney.

40. While cross-examining a defendant on tri- (D)


al for robbery and assault with a deadly
weapon, the prosecutor asks him whether The question is proper. The
he was convicted of fraud within the previ- defendant has taken the
ous year. stand in his own defense,
and therefore the prosecu-
Is this question proper? tor can attack his credibili-
ty as a witness. Under Fed-
eral Rule 609, evidence of
A No, because fraud is not probative of a conviction of a crime requir-
tendency to commit violence. ing proof of an act of dis-
honesty or false statement
B No, unless the proper foundation was laid. can always be used to at-
tack a witness's character
C Yes, because fraud is a form of stealing, for truthfulness. (A) is incor-
and so it will tend to show that the defendant rect because even if fraud
could commit robbery. were probative of the ten-
dency to commit violence,
D Yes, because it tends to show that the evidence of other crimes is
defendant would lie. not admissible to prove that
a person has a propensity to
commit criminal acts. (C) is
incorrect for the same rea-
son. (B) is wrong because
no foundation is needed to
show a prior conviction for
impeachment purposes.

41. A vendor entered into a written contract Direction (iv) needs to be


with a purchaser for the sale of a large tract corrected in its course but
of land. The contract set forth an accurate not its distance. In land con-
metes and bounds description of the land tracts and deeds, property
based on a professional survey. At closing, may be described in vari-
the purchaser discovered that the deed was ous ways as long as the
incorrectly transcribed and did not agree description is unambiguous.

27 / 184
Barbri MBE questions
Study online at https://quizlet.com/_6umlok
with the description of the land in the con- From a designated starting
tract. The deed described the property to be point that can be identified
conveyed as follows: by reference to a govern-
ment survey or a natural
"(i) from the southwest corner of [a spec- or artificial monument, the
ified starting point], proceed South 45 de- boundaries of the property
grees East 200 feet to [a specified point]; (ii) can be described by succes-
from that point, proceed South 45 degrees sive calls of courses (e.g.,
West 100 feet to [a specified point]; (iii) from angles) and distances un-
that point, proceed North 45 degrees West til returning to the starting
200 feet to [a specified point]; and (iv) from point. A course is a state-
that point, proceed South 45 degrees East ment of direction generally
100 feet to the starting point." stated as some number of
degrees east or west of due
The purchaser refused to proceed with the north or south. In each call a
closing and brought an action to reform the distance must be stated to-
deed to make it conform to the intention of gether with the course. Thus,
the parties. the boundary in direction (iv)
runs at an angle 45 degrees
Which of the following corrections should east of due south (i.e., south-
be made for the deed to properly describe east) for a distance of 100
the land? feet. However, because di-
rection (i) went southeast, di-
A Direction (i) should be changed to "South rection (ii) went southwest,
45 degrees East 100 feet." and direction (iii) went north-
west, the fourth direction has
B Direction (iii) should be changed to "North to be northeast for a dis-
45 degrees West 100 feet." tance of 100 feet to bring
the final boundary back to
C Direction (iii) should be changed to "North the starting point. (In this
45 degrees East 200 feet." type of question, diagram
the boundaries as shown be-
D Direction (iv) should be changed to "North low to help you visualize the
45 degrees East 100 feet." property.) Therefore, the cor-
rection in choice (D) is cor-
rect. (A), (B), and (C) are
incorrect because none of
those proposed changes in
distance or direction would
28 / 184
Barbri MBE questions
Study online at https://quizlet.com/_6umlok
be sufficient to bring the fi-
nal call back to the starting
point.

42. A plaintiff filed a civil action based on negli- (B)


gence against a defendant in federal district
court, alleging that the defendant negligent- The court should deny the
ly ran a red light at an intersection and col- plaintiff's motion. Summary
lided with the plaintiff's vehicle, causing the judgment may be granted
plaintiff's injuries. if, from the pleadings, affi-
davits, and discovery mate-
A week after the close of discovery, the rials, it appears that there is
plaintiff filed a motion for summary judg- no genuine dispute of mater-
ment on the issue of whether the defendant ial fact and the moving party
was negligent. With the motion, the plaintiff is entitled to judgment as a
filed (i) his own sworn affidavit, which stat- matter of law. The court may
ed that the traffic signal was green as he not decide disputed fact is-
entered the intersection, (ii) an affidavit of a sues on a motion for sum-
witness who was driving the car behind him, mary judgment; if there is
which stated that the witness saw the entire a genuinely disputed mate-
incident and that the plaintiff's traffic signal rial fact (meaning a dispute
was green as he approached and entered backed by evidence on both
the intersection; and (iii) an affidavit of an- sides of the issue), the case
other witness, which stated that she saw the must go to trial. Here, al-
entire incident and that the defendant's sig- though the plaintiff's case
nal had been red for several seconds before may seem stronger, the de-
the defendant entered the intersection and fendant has presented some
was still red when the defendant entered the evidence showing that she
intersection. was not negligent. Thus, the
case must proceed to trial.
In response to the motion, the defendant (C) and (D) are incorrect be-
filed her own affidavit, which stated that her cause, in a motion for sum-
traffic signal was green when she entered mary judgment, the credibil-
the intersection. She also filed the sworn ity of the witnesses and the
deposition transcript of a pedestrian on the strength of the evidence is
scene, in which the pedestrian said that, al- not weighed.
though he was intoxicated, he saw the entire
accident and that the defendant's traffic sig-
nal was green as the defendant approached
29 / 184
Barbri MBE questions
Study online at https://quizlet.com/_6umlok
and entered the intersection.

How should the court rule on the plaintiff's


motion for summary judgment?

A Deny the motion, because the plaintiff


cannot obtain summary judgment on issues
on which he has the burden of proof.

B Deny the motion, because both par-


ties have come forward with evidence from
which a jury could find for either party if the
jury believes that party's evidence.

C Grant the motion, because valid reasons


exist to question the credibility of the evi-
dence that the defendant presented.

D Grant the motion, because the evidence


presented by the plaintiff is substantially
more persuasive than the evidence present-
ed by the defendant.

43. A landlord owned a prestigious downtown (B)


office building. A law firm leased the entire
building from the landlord for a term of 20
The law firm and the ac-
years. The lease included a provision thatcounting firm are liable. After
taxes on the building would be paid by "the
an assignment, the original
lessee, his successors, and assigns." The tenant is no longer in priv-
law firm occupied the building and paid the
ity of estate with the land-
rent and taxes for eight years. At the endlord. However, a tenant may
of the eight-year period, the law firm as-still be held liable on its orig-
signed the balance of the lease to an ac- inal contractual obligations
counting firm and vacated the premises. The
to the landlord on privity of
assignment was written, but there was no contract grounds. Here, the
provision concerning the accounting firm'slaw firm is liable because it
assumption of the duties under the lease. made the original deal with
the landlord, which included
The accounting firm occupied the building the obligation to pay taxes
30 / 184
Barbri MBE questions
Study online at https://quizlet.com/_6umlok
and paid the rent and taxes for five years. on the building. The law firm
At the end of the five-year period, the ac- remains in privity of contract
counting firm subleased the building for fivewith the landlord throughout
years to an investment company and vacat- the term of the lease unless
ed the premises. The sublease was written, it is otherwise discharged. In
but there was no provision concerning the an assignment, the assignee
investment company's assumption of the stands in the shoes of the
duties under the lease. The investment com- original tenant in a direct re-
pany now occupies the building and has lationship with the landlord.
paid the rent but not the taxes. The landlordEach is liable to the other
has sued all three (i.e., the law firm, the ac-
on all covenants in the lease
counting firm, and the investment company) that run with the land, which
for failure to pay the taxes. would include the obligation
of the lessee to pay tax-
The landlord should prevail against whom? es on the property. Here,
the accounting firm is liable
A The law firm only. because as an assignee it
is in privity of estate with
B The law firm and the accounting firm, but the landlord. The accounting
not the investment company. firm remains in privity of es-
tate until it assigns to some-
C The accounting firm and the investment one else. The sublease to
company, but not the law firm. the investment company is
not an assignment. A sub-
D The law firm, the accounting firm, and the lessee is not personally li-
investment company. able to the landlord for rent
or for the performance of any
other covenants made by the
original lessee in the main
lease (unless the covenants
are expressly assumed) be-
cause the sublessee does
not hold the tenant's full es-
tate in the land (so no priv-
ity of estate). Here, the in-
vestment company is not li-
able because, as a nonas-
suming sublessee, it is not in
privity of contract or estate
31 / 184
Barbri MBE questions
Study online at https://quizlet.com/_6umlok
with the landlord. Therefore,
(B) is the correct choice, and
(A), (C), and (D) are wrong.

44. A man shopping for a leather jacket at a (A)


clothing store could not decide between two
jackets, so the proprietor, who knew the man
The proprietor can recover
and his family well, let him take one of the
the cost of the jacket from
jackets on approval. No mention was made the grandfather's estate be-
by the proprietor of the method of payment cause the proprietor is an
he expected. The man wore the jacket on intended third-party benefi-
a visit to his grandfather, who liked it sociary and his right to en-
much that when the man told him what the force the contract has vest-
jacket cost and that he had taken it on ap-ed. The rights of an in-
proval, the grandfather said he would buy it
tended third-party beneficia-
for him if he promised to give some of his ry vest when the beneficia-
old clothes to a favorite charity for the poor
ry (i) manifests assent to the
at Christmastime. The man wholeheartedly promise in a manner invit-
agreed to donate the clothes to the charityed or requested by the par-
at Christmas. Very pleased, the grandfatherties; (ii) brings suit to en-
called the shop and told the proprietor to force the promise; or (iii)
send the bill for the jacket to him, which he
materially changes his po-
did. Before the bill was paid and before the
sition in justifiable reliance
Christmas season arrived, the grandfather on the promise. Here, the
fell ill and died. The grandfather's executor
proprietor qualifies as an in-
has refused to pay the bill, and the man has
tended beneficiary of the
not yet given any old clothing to the charity.
agreement between the man
and his grandfather because
Will the proprietor be able to recover the the proprietor was express-
price of the jacket from the estate? ly designated in the con-
tract, he was to receive per-
A Yes, because the proprietor was the in- formance directly from the
tended beneficiary of the promise between grandfather, and he stood
the man and his grandfather. in an existing contractual
relationship with the man
B Yes, because the man has no duty to give that required the man to ei-
the clothing to the charity. ther pay for the jacket or
return it, making it likely
C No, because the grandfather's implied that the young man's pur-
32 / 184
Barbri MBE questions
Study online at https://quizlet.com/_6umlok
promise to pay the proprietor arising from pose in making the arrange-
the phone call is unenforceable. ment with his grandfather
was to satisfy the obligation
D No, because a condition has not yet oc- to the proprietor. The pro-
curred. prietor can enforce the con-
tract because his rights vest-
ed when he sent the bill to
the grandfather at the grand-
father's request. Thus, the
proprietor will prevail against
the grandfather's estate. (D)
is wrong because the man's
giving the clothes to the
charity is not a condition
that must be fulfilled be-
fore the grandfather's estate
must pay. The grandfather
promised to pay for the jack-
et if the man promised to
donate the clothes; i.e., the
consideration for the grand-
father's promise was the
man's promise, not his ac-
tually donating the clothes.
As soon as the man made
the promise, the grandfa-
ther's duty to pay became
absolute. (If the man does
not donate the clothes, he
will be in breach of his con-
tract with his grandfather,
but the grandfather's perfor-
mance was not conditioned
on the man's donating the
clothes.)

45. A man and a woman lived together for many (A)


years but never got married. Although the
state in which they reside does not recog- The man takes sole title to
33 / 184
Barbri MBE questions
Study online at https://quizlet.com/_6umlok
nize common law marriage, it has statutes the property under his right
that prohibit discrimination on the basis of of survivorship. A joint ten-
marital status. The man and the woman pur- ancy carries the right of sur-
chased a large property, taking title as joint vivorship. Thus, when one
tenants. Subsequently, the woman accumu- joint tenant dies, the prop-
lated a $20,000 debt. She was too embar- erty is freed of her interest
rassed to tell the man but was able to con- and the surviving joint ten-
vince a bank to hold a mortgage on the ant holds the entire proper-
property in exchange for the money. The ty. Therefore, the man owns
bank was also willing to accept the woman's the property. (B) is wrong be-
signature alone, and the man never learned cause the bank has no in-
about the mortgage. Two years later, the terest. Most states, like the
woman died without having paid off the one in this question, regard a
mortgage. She left no will, and her only heir mortgage as a lien on title. In
at law is her sister. The state in which the these states, a mortgage of
property is located is a "lien theory" mort- the property by one joint ten-
gage state. ant does not, by itself, sever
a joint tenancy until default
Who has title to the property? and foreclosure proceedings
have been completed. The
A The man. bank's rights were lost when
the woman died prior to fore-
B The man and the bank. closure. When the woman
died, her interest in the prop-
C The man and the sister. erty evaporated, and with it
the bank's security interest.
D The man, the sister, and the bank. On the other hand, in a title
theory state, a mortgage is
considered to be an actual
transfer of title to the proper-
ty, rather than just a lien on
the property. Thus, a mort-
gage by a joint tenant trans-
fers the legal title of the joint
tenant to the mortgagee (the
money lender). This action
destroys the unity of title and
thus severs the joint tenancy.

34 / 184
Barbri MBE questions
Study online at https://quizlet.com/_6umlok
46. A defendant was charged with arson (a (B)
felony) of an antique shop. Only one corner
of the shop was damaged before the fire The record of the defen-
was extinguished. Under a plea agreement, dant's conviction should be
the defendant pled guilty and received a admitted to prove that the
suspended sentence. Because the owner of defendant set the fire. The
the shop had not yet insured a recently ac- record of the conviction is
quired 400-year-old refectory table that was hearsay; Under the Federal
destroyed by the fire, he sued the defendant Rules, however, such judg-
for damages. At trial, the owner offers the ments fall within the hearsay
properly authenticated record of the defen- exception for records of
dant's conviction for arson. felony convictions. Under the
Federal Rules, judgments of
Should the record be admitted into evi- felony convictions are ad-
dence? missible in both criminal and
civil actions to prove any
A Yes, as proof of the defendant's character fact essential to the judg-
in order to infer liability. ment, whether the judgment
arose after trial or upon a
B Yes, as proof that the defendant set the plea of guilty. [Fed. R. Evid.
fire. 803(22)] For purposes of
this Rule, a felony is any
C No, because the conviction was not the crime punishable by death
result of a trial. or imprisonment in excess of
one year. Arson is a felony.
D No, because it is hearsay not within any Consequently, a properly au-
exception. thenticated copy of the de-
fendant's conviction of this
crime is admissible to prove
the fact that the fire that de-
stroyed the table was set by
the defendant, a fact essen-
tial to the judgment of con-
viction. Note that the actual
plea of guilty is also admissi-
ble as a statement of a par-
ty-opponent This type of judi-
cial admission is not conclu-
sive, and the defendant may
35 / 184
Barbri MBE questions
Study online at https://quizlet.com/_6umlok
explain the circumstances of
the plea. The plea, being
an admission, is nonhearsay
under the Federal Rules.

47. The defendant and an accomplice were on (D)


trial together for burglary. Both had giv-
en confessions implicating themselves and The confession is admissi-
their accomplice. At trial, the defendant ble with the judge's limit-
maintained that his confession had been ing instruction. Where two
obtained through improper coercion by the persons are tried together
police. For the purpose of countering the and one has given a con-
claim of coercion, the prosecution seeks to fession implicating the oth-
place the accomplice's confession into ev- er, the general rule is that
idence. After objection by the defendant's the Sixth Amendment right
counsel, the judge agrees to issue a limiting to confront adverse witness-
instruction to the jury that the confession es prohibits the use of such
is to be considered only with regard to the a statement. This problem
question of whether the defendant's confes- arises because of the inabil-
sion was coerced. ity of the nonconfessing de-
fendant to compel the con-
May the accomplice's confession be admit- fessing co-defendant to take
ted under that condition? the stand for cross-examina-
tion at their joint trial. As ex-
A No, because admission of the confession ceptions to the general rule,
violates the defendant's right of confronta- the statement may be ad-
tion. mitted if: (i) all portions of
the statement referring to the
B No, unless the accomplice takes the stand other defendant can be elim-
and subjects himself to cross-examination inated (so that there is no
regarding the confession. indication of that defendant's
involvement); (ii) the con-
C Yes, as long as all portions of the con- fessing defendant takes the
fession referring to the defendant can be stand and subjects himself
eliminated. to cross-examination with re-
spect to the truth or falsi-
D Yes, because the judge's instruction limits ty of what the statement as-
consideration of the confession only to the serts; or (iii) the confession
issue of coercion. of the nontestifying co-de-
36 / 184
Barbri MBE questions
Study online at https://quizlet.com/_6umlok
fendant is being used to
rebut the defendant's claim
that his confession was ob-
tained coercively, in which
case the jury must be in-
structed as to the purpose
of the admission. (B) and
(C) are incorrect because
neither of those conditions
is necessary for the confes-
sion to be admitted as long
as the judge issues a limit-
ing instruction, as discussed
above.

48. A woman purchased a tract of land from a (C)


man by warranty deed. Unbeknownst to the
woman, the man was not the actual owner The woman would succeed
of the tract. The woman built a home on in a suit for damages against
the tract and moved into it. Two years lat- the man for breach of the
er, the actual owner learned of the man's covenants of seisin, right
transaction with the woman and prevented to convey, quiet enjoyment,
the woman from entering the tract from that warranty, and further as-
point forward. This led to a costly court bat-
surances, but not on the
tle. When the woman notified the man and covenant against encum-
told him that she thought it was his duty tobrances. A general warranty
straighten this out, he ignored her. deed gives the grantee six
covenants of title: the right
The woman would succeed in a suit for dam- to seisin, the right to convey,
ages against the man for breach of which of a covenant against encum-
the following covenants of title? brances, the covenant of qui-
et enjoyment, the covenant
A The covenant of quiet enjoyment only. of further assurances, and
a general warranty. Under
B The covenants of seisin, right to con- the covenants of quiet en-
vey, quiet enjoyment, warranty, further as- joyment, warranty, and fur-
surances, and the covenant against encum- ther assurances, the man
brances. promised that (i) the woman
would not be disturbed in
37 / 184
Barbri MBE questions
Study online at https://quizlet.com/_6umlok
C The covenants of seisin, right to convey, her possession of the tract;
quiet enjoyment, warranty, and further as- (ii) he would defend the
surances. woman's title against law-
ful claims; and (iii) he would
D The covenants of seisin and right to con- perform whatever acts are
vey only. necessary to perfect the
woman's title. Because the
man neither owned the tract
of land nor was acting as
the actual owner's agent, he
breached the covenants of
seisin and right to convey at
the time of the conveyance
to the woman. When the
actual owner prevented the
woman from re-entering the
property, this interfered with
the woman's quiet enjoy-
ment, and the man's refusal
to "straighten this out" was
a breach of the covenant
of further assurances. Thus,
(C) is the correct answer.
There is nothing in the facts
to suggest the property is
encumbered; thus, the man
did not breach the covenant
against encumbrances, and
(B) is therefore incorrect.

49. Based on recommendations of a state com- (A)


mission studying the effect of pornograph-
ic films on violent criminal activity, a state The court will likely find the
adopted legislation banning films intended legislation to be a constitu-
for commercial distribution that appealed as tional regulation of obscen-
a whole to the prurient interest in sex of the ity. Obscenity, which is not
average person in the community, portrayed protected speech under the
sex in a patently offensive way to citizens of First Amendment, is defined
the state, and which a reasonable person in by the Supreme Court as
38 / 184
Barbri MBE questions
Study online at https://quizlet.com/_6umlok
the United States would find had no serious a description or depiction of
literary, artistic, political, or scientific value. sexual conduct that, taken
as a whole, by the aver-
In ruling on a constitutional challenge to the age person, applying con-
legislation from a film distributor in the state temporary community stan-
who was convicted of distributing films in dards, appeals to the pruri-
violation of the legislation, will the federal ent interest in sex, portrays
court likely find the legislation to be consti- sex in a patently offensive
tutional? way, and—using a nation-
al reasonable person stan-
dard—does not have seri-
A Yes, because it uses a national "reason- ous literary, artistic, politi-
able person" standard for determining the cal, or scientific value. Thus,
social value of the work. the legislation here is con-
stitutional because it uses
B Yes, because it uses a statewide standard a reasonable person stan-
rather than a community standard for de- dard, rather than a commu-
termining whether the material is patently nity standard, for determin-
offensive. ing the value of the work. (B)
is incorrect because while a
C No, because it uses a statewide standard statewide standard for deter-
rather than a national standard for determin- mining whether the material
ing whether the material is patently offen- is patently offensive is per-
sive. missible, it is not mandato-
ry. A state may use a "com-
D No, unless the court finds that the munity standard" for mak-
legislation is necessary to advance the ing this determination. (C) is
state's compelling interest in reducing vio- incorrect because, again, a
lent criminal activity. statewide standard for deter-
mining whether the material
is patently offensive is per-
missible. Only the "social val-
ue" element of the obscen-
ity test requires a nation-
al standard. (D) is incorrect
because the legislation is
valid regardless of whether
it is necessary to achieve
the state's compelling inter-
39 / 184
Barbri MBE questions
Study online at https://quizlet.com/_6umlok
est in reducing violent crime.
Speech that falls within the
definition of obscenity is un-
protected speech; the gov-
ernment does not need a
specific compelling interest
to ban it.

50. An environmentalist divided her 25-acre (B)


property into 100 quarter-acre residential
lots. At the time the environmentalist sold The veteran's best defense
her lots, there was a recycling center about is that the covenant does
one mile from the western boundary of the not clearly "touch and con-
development. She included in the deed of all cern" the land. While recy-
100 grantees the following provision: cling may benefit the com-
munity at large, "touch and
"Grantee covenants for herself and her concern" involves the rela-
heirs and assigns that all aluminum cans, tionship between landown-
glass bottles, and grass clippings of ers at law. Recycling by
Grantee and her heirs and assigns shall be the veteran does not direct-
recycled. This covenant runs with the land ly benefit the other landown-
and shall remain in effect as long as there is ers in the use and enjoy-
a recycling center within five statute miles ment of their land. Thus, (B)
of the development." is correct. (A) is wrong be-
cause even though the vet-
A buyer purchased a lot in the development. eran's deed does not contain
Her deed, which contained the recycling the covenant, he has record
clause, was duly recorded. Two years later, notice because the restric-
the buyer decided to give the property to tion is in his chain of title.
her niece as a gift. The niece's deed to the (C) is wrong because servi-
property contained the recycling covenant, tudes implied from a com-
and she too recorded her deed. Shortly after mon scheme apply only to
the niece took possession of the house, the negative covenants, and the
recycling center moved its location to a new recycling requirement is an
site about four and a half miles from the affirmative covenant. Thus,
development. When the niece put the house this defense does not go to
up for sale, she said nothing to prospective the point. (D) is wrong be-
buyers about recycling. cause it goes only to is-
sues in equity. The suit in-
40 / 184
Barbri MBE questions
Study online at https://quizlet.com/_6umlok
The house was purchased by a veteran who cludes a claim for damages
had lost the use of his legs. The veteran's at law. In any case, balanc-
deed did not contain the recycling clause, ing of hardships is not gen-
and he hired a local disposal service to car- erally applied in such cases
ry away his garbage and a landscaper to (although some courts might
maintain the yard. The landscaper bagged elect to do so).
the grass clippings and they were removed
by the disposal service, which put all the
trash and clippings in a landfill. When the
veteran's neighbors informed him of his
duty to recycle, he told them that he knew
nothing of the covenant and that it would be
difficult for a person in his physical condi-
tion to haul cans, bottles, and clippings to
the recycling center. Unfazed, the neighbors
filed suit to require the veteran to comply
with the covenant or pay damages.

The veteran's best defense is which of the


following?

A The veteran's deed did not contain the


covenant.

B The covenant does not touch and concern


the land.

C An intelligent inspection of the neigh-


borhood would raise no inference that the
covenant existed.

D The veteran's physical condition requires


a balancing of hardships by the court.

51. A defendant visited her doctor to seek treat- (C)


ment for a bullet wound. While he was treat-
ing the wound, the doctor asked the de- The court should over-
fendant how she was shot. The defendant rule the objection because
41 / 184
Barbri MBE questions
Study online at https://quizlet.com/_6umlok
replied that she was struck by a police of- the physician-patient privi-
ficer's bullet while running away from a jew- lege cannot be invoked for
elry store she had robbed, but she implored information dealing with a
the doctor not to tell this to anyone. The doc- nonmedical matter. Under
tor promised that he would not. Although the physician-patient privi-
the defendant was never charged by the po- lege, a physician is fore-
lice, the owner of the jewelry store brought closed from divulging in ju-
suit against her seeking the value of the dicial proceedings informa-
stolen goods. The defendant denied robbing tion that he acquired while
the store. At the trial, the owner calls the attending a patient in a pro-
doctor to testify to the statement made to fessional capacity, which in-
him by the defendant. The defense attorney formation was necessary to
objects on the ground that such testimony enable the physician to act in
is barred by the jurisdiction's physician-pa- his professional capacity. In-
tient privilege. formation given by a patient
that deals with a nonmed-
Should the objection be sustained? ical matter is not protected
by the privilege. Hence, the
A Yes, because the doctor acquired this in- defendant's admission that
formation while attending the defendant in she was shot while running
the course of treatment. from a jewelry store that she
robbed is not barred by the
B Yes, because the doctor agreed to the de- privilege. (A) is incorrect be-
fendant's specific request that this informa- cause, although it is true
tion be kept confidential. that the doctor acquired the
information while attending
C No, because the physician-patient privi- the defendant in the course
lege is inapplicable to the defendant's state- of treatment, the privilege
ment. is inapplicable because, as
discussed above, the state-
D No, because the doctor is the one who ment deals with a nonmed-
is entitled to either claim this privilege or ical matter.
waive it.

52. A plaintiff filed a negligence action against (D)


a defendant in federal district court after
a two-car accident. The plaintiff's attorney The plaintiff must provide the
created a list of everyone he could identi- defendant with the names
fy who observed the accident or otherwise of the people on the plain-
42 / 184
Barbri MBE questions
Study online at https://quizlet.com/_6umlok
had information relevant to the accident. tiff's attorney's list. In gen-
The list includes one eyewitness whom the eral, discovery may be had
plaintiff's attorney was able to identify only of any nonprivileged matter
through the expenditure of several thou- that is relevant to any par-
sand dollars in investigation costs. The de- ty's claim or defense and
fendant served the following interrogatory proportional to the needs of
on the plaintiff: "Please state the name of the case, including the iden-
each person of whom you are aware who tity of individuals with knowl-
may know or have information relevant to edge of any discoverable
this action." matter. Because the Feder-
al Rules of Civil Procedure
Must the plaintiff provide the defendant with state that the identities of
the names of all of the people on the plain- people with knowledge of
tiff's attorney's list? discoverable matters are dis-
coverable, the plaintiff's at-
torney's list is not considered
A No, because the names on the list are work product. Thus, (A) and
protected from discovery under the work (C) are incorrect. (B) is incor-
product doctrine. rect because the identity of
the eyewitness is discover-
B No as to the name of the eyewitness found able regardless of the extent
through the plaintiff's investigation efforts, of the plaintiff's investigation
but yes as to the other names on the list. efforts.

C Yes, because, while the names are subject


to qualified immunity from discovery under
the work product doctrine, the defendant
will be able to show sufficient need to obtain
a court order requiring the names' disclo-
sure.

D Yes, because the names are relevant to the


claims and defenses of the parties, and they
do not constitute work product.

53. The owner of a valuable painting hired pro- (C)


fessional movers to transport it to an auc-
tion house when she decided to sell it. As Because the owner was
the movers were carrying it to their van, a across the street at the time
43 / 184
Barbri MBE questions
Study online at https://quizlet.com/_6umlok
window air conditioner that a tenant had of impact, she was not with-
been trying to install fell out of his second in the zone of danger, thus
floor window and crashed through the paint- precluding her recovery for
ing and onto the ground. The owner had negligent infliction of emo-
been watching from her apartment across tional distress. A defendant
the street and saw her painting destroyed. breaches a duty to avoid
She became extremely upset and needed negligent infliction of emo-
medical treatment for shock. tional distress when he cre-
ates a foreseeable risk of
If the owner brings a claim for negligent physical injury to the plain-
infliction of emotional distress against the tiff through causing a threat
tenant, is she likely to recover? of physical impact that leads
to emotional distress. Dam-
ages generally are recover-
A Yes, because she suffered physical symp- able only if the defendant's
toms from her distress. conduct causes some phys-
ical injury, rather than purely
B Yes, because she was a foreseeable plain- emotional distress (although
tiff. a severe shock to the ner-
vous system that causes
C No, because she was not within the zone physical symptoms is suffi-
of danger. cient). If plaintiff's distress is
caused by threat of physi-
D No, because she suffered no physical im- cal impact to her, she must
pact. have been within the zone
of danger. Here, the owner
witnessed the air condition-
er striking her painting from
across the street. This van-
tage point placed her outside
the zone of danger from the
falling air conditioner. Thus,
the owner cannot recover for
negligent infliction of emo-
tional distress.

54. A landowner validly conveyed a small office (D)


building to the Green Party "as long as they
use it for operating quarters until the next The son may enjoin the sale
44 / 184
Barbri MBE questions
Study online at https://quizlet.com/_6umlok
presidential election." After the next presi- because he has an interest
dential election, which was in three years, in the property. A fee sim-
the building would go to a private organi- ple determinable is an es-
zation that monitors and prepares compre- tate that automatically termi-
hensive listings of gas prices throughout nates on the happening of
the country. A year after the conveyance, a stated event. The Green
the landowner died, validly devising all of Party's interest in the office
her property to her son. Although this ju- building is a fee simple de-
risdiction is a common law jurisdiction with terminable because it lasts
respect to all real property considerations, as long as the Party is us-
the state's probate laws provide that future ing the building for operating
interests or estates in real property may be quarters. However, the grant
passed by will or descent in the same man- does not provide for the con-
ner as present or possessory interests. Last tingency of the Green Party
week, the Green Party and the gas monitor- ceasing to use the building
ing organization joined together to sell the as operating quarters before
office building in fee simple absolute to a the next presidential elec-
developer. The son filed suit to prevent the tion. This gap would be filled
sale of the property to the developer. by a possibility of reverter re-
tained by the landowner. Be-
In this action, who should prevail? cause the landowner passed
that interest to her son in her
will, there can be no contract
A The Green Party and the gas monitoring to sell the property without
organization, because together they own a his signature. Note: Although
fee simple absolute in the building. the gas monitoring organiza-
tion appears to have an in-
B The Green Party and the gas monitoring defeasibly vested remainder
organization, because the attempted restric- (i.e., it is created in an ascer-
tions on the use of the property violate the tained company, is certain to
Rule Against Perpetuities. become possessory, and is
not subject to being defeat-
C The Green Party and the gas monitoring ed, divested, or diminished
organization, because the deed restriction in size), its interest is not ca-
was an unlawful restraint on alienation. pable of taking on the natural
termination of the preceding
D The son, because he did not sign the con- estate and so is character-
tract of sale. ized as a springing executo-
ry interest. (A) is wrong be-
45 / 184
Barbri MBE questions
Study online at https://quizlet.com/_6umlok
cause the son also has an
interest in the land. (B) is
wrong because the interest
in the office building will pass
to the gas monitoring orga-
nization, if at all, within 21
years. (C) is wrong because
the Green Party is not pro-
hibited from transferring any
interest; it could pass a de-
feasible fee.

55. A vintner divided his vineyard into two (C)


parcels, drawing the boundaries so that the
single well that had irrigated the entire vine- The farmer will most likely
yard fell on the border of the two properties. prevail in his suit for one-half
The vintner then conveyed the eastern par- the cost of the well re-
cel to his friend by a deed that contained the pairs because the covenant
following covenant: runs with the land. When
a covenant runs with the
If the well located on the boundary of the land, subsequent owners of
eastern and western parcels continues to the land may enforce or be
be used for irrigation purposes and be- burdened by the covenant.
comes in need of repair or replacement, If all of the requirements
the grantee, his heirs, and assigns and the for the burden to run are
grantor, his heirs, and assigns each promise met, the successor in in-
to pay one-half of the cost of such repair terest to the burdened es-
or replacement. This covenant shall run with tate will be bound by the
the land. arrangement as effectively
The deed from the vintner to the friend was as if he had himself ex-
not recorded, and the vintner did not record pressly agreed to be bound.
a copy of the deed with the records for the To be bound: (i) the par-
western parcel. ties must have intended that
the covenant run with the
The friend later sold the eastern parcel to land; (ii) the original parties
a farmer. The farmer's deed did not contain must have been in horizon-
the covenant about the well. After 15 years tal privity; (iii) the succeed-
of use by the owners of both the eastern ing party must be in verti-
and western parcels, the well began to fail. cal privity with the original
46 / 184
Barbri MBE questions
Study online at https://quizlet.com/_6umlok
The farmer took it upon himself to have the promisor; (iv) the covenant
well repaired at a cost of $30,000. About must touch and concern the
two weeks later, the farmer discovered the land; and (v) generally, the
deed from the vintner to the friend in some burdened party must have
old files. By this time, the western parcel actual or constructive notice
had passed to the vintner's son by inheri- of the covenant. Here, the in-
tance and again to the son's daughter by in- tent is shown by the express
heritance from the now-deceased son. The language of the covenant,
daughter knew nothing of the covenant con- which says that it is intend-
cerning the well. The farmer presented the ed to run with the land. Even
daughter with the bill for the well repair with without that language, the
a copy of the vintner/friend deed and a note use of the words "heirs" and
that said he expected to be reimbursed for "assigns" would show the in-
$15,000. The daughter refuses to pay, and tent for the covenant to run.
the farmer sues. The original parties were in
horizontal privity because at
The jurisdiction has a 10-year statute of the time the vintner entered
limitations for acquiring property by ad- into the covenant, he and
verse possession, and the following record- the friend shared an inter-
ing statute: "Any conveyance of an interest est in the land independent
in land shall not be valid against any sub- of the covenant—as grantor
sequent purchaser for value, without notice and grantee. The daughter
thereof, unless the conveyance is record- is in vertical privity with the
ed." vintner because she holds
the entire interest in the
For whom is the court most likely to rule? western parcel held by the
vintner. The covenant touch-
es and concerns the land be-
A The daughter, because the deed from the cause promises to pay mon-
vintner to the friend was never recorded. ey to be used in a way
connected with the land are
B The daughter, because the farmer has ac- held to touch and concern
quired the well by adverse possession. the property. Because the
daughter was unaware of
C The farmer, because the covenant runs the covenant, the required
with the land. notice seems to be miss-
ing. While it is generally true
D The farmer, because he is a bona fide pur- that the owner of the bur-
chaser. dened land must have no-
47 / 184
Barbri MBE questions
Study online at https://quizlet.com/_6umlok
tice, it should be remem-
bered that the requirement is
a function of the recording
statute. (At common law, the
covenant was enforceable in
an action for damages re-
gardless of notice; this was
changed by the recording
statutes.) However, because
the daughter is a donee (an
heir) and not a bona fide pur-
chaser, she is not protect-
ed by the recording statute
and thus is subject to the
covenant even without no-
tice. For that reason, (A)
is wrong. (D) is wrong be-
cause the farmer's status
as a bona fide purchaser
has no effect on his abili-
ty to enforce the covenant.
A successor in interest to
the original promisee may
enforce the covenant (enjoy
the benefit) if there was in-
tent and vertical privity, and
the covenant touches and
concerns the land. Notice is
not required for the bene-
fit to run. Thus, because the
above requirements are met
here, the farmer may enforce
the covenant regardless of
his status as a bona fide pur-
chaser. Had the farmer taken
the property as a donee, the
above analysis would be the
same.

48 / 184
Barbri MBE questions
Study online at https://quizlet.com/_6umlok
56. A father executed a deed to his art gallery (B)
"to my daughter for her life, and on my
daughter's death to her children; provided, The daughter's two children
however, that if my daughter stops painting, have a vested remainder
to my brother." The daughter has two chil- subject to open and subject
dren and is still painting. to complete divestment. A
remainder is a future interest
At the time of the grant, what is the best created in a transferee that
description of the interest of the daughter's is capable of taking in pos-
two children? session on the natural termi-
nation of the preceding es-
A A contingent remainder. tate. A remainder is vested
if the beneficiaries are as-
B A vested remainder subject to open and certainable and their taking
to total divestment. in possession is not subject
to a condition precedent. A
C A vested remainder subject to open. vested remainder created in
a class of persons that is cer-
D An executory interest. tain to take but is subject to
diminution by reason of oth-
ers becoming entitled to take
is a vested remainder sub-
ject to open. Vested remain-
ders may be subject to to-
tal divestment if possession
is subject to being defeated
by the happening of a con-
dition subsequent. Here, the
daughter's two children have
a remainder because, on the
expiration of the daughter's
life estate, they will be en-
titled to possession of the
property. The remainder is
not subject to a condition
precedent and the benefi-
ciaries are in existence and
ascertained, so the remain-
der is vested, not contingent.
49 / 184
Barbri MBE questions
Study online at https://quizlet.com/_6umlok
The remainder is subject to
open because the daughter
may have more children. Fi-
nally, the remainder is sub-
ject to total divestment be-
cause the daughter's chil-
dren's right to possession
is subject to being defeated
by the daughter's ceasing to
paint. (A) is wrong because
the remainder is vested, not
contingent; i.e., it is not sub-
ject to a condition precedent,
and the beneficiaries are as-
certainable.

57. A defendant is on trial for manslaughter af- (B)


ter he hit a victim in a bar, causing the victim
to fall and hit his head on the marble bar top. The defense witness's testi-
The defendant claims that he hit the victim mony should be admitted as
in self-defense after the victim lunged at proper impeachment of the
him with a knife. During the prosecution's prosecution's witness. Im-
case, a witness testifies that she heard the peachment is the casting of
victim's friend shout at the defendant, "You an adverse reflection on the
just killed a helpless man!" A defense wit- veracity of a witness. A wit-
ness is called to testify that he was there ness may be impeached by
and does not remember hearing the victim's either cross-examination or
friend say anything. extrinsic evidence, such as
by putting other witnesses
Should the defense witness's testimony be on the stand who contra-
admitted? dict the witness's testimo-
ny. Here, the defense is us-
A No, it is irrelevant to any issue in the case. ing the testimony of its wit-
ness to impeach the pros-
B Yes, it is proper impeachment of the pros- ecution witness's testimony
ecution's witness. as to what the victim's friend
said. This is proper.
C No, it is improper impeachment of the
prosecution's witness because it relates to
50 / 184
Barbri MBE questions
Study online at https://quizlet.com/_6umlok
a collateral matter.

D No, it is improper impeachment because


it does not positively controvert the prose-
cution witness's testimony, as the defense
witness merely says he does not remember.

58. While at a party, the defendant ran into an (B)


acquaintance. The acquaintance proceeded
to ridicule the defendant about his looks. This choice states the Mod-
After an hour of verbal abuse by the ac- el Penal Code test. Pursuant
quaintance, the defendant suddenly took to the Model Penal Code, a
a champagne bottle that was on a nearby defendant is entitled to ac-
table and struck the acquaintance over the quittal if he suffered from
head, killing him instantly. At his arrest, the a mental disease or defect
defendant told the police that voices inside and as a result lacked sub-
his head told him to shut the acquaintance stantial capacity to either: (i)
up, permanently. appreciate the criminality of
his conduct; or (ii) conform
The defendant was tried in a jurisdiction his conduct to the require-
that follows the Model Penal Code test for ments of law. (A) is wrong
insanity. At trial, the defendant's lawyer in- because it would be help-
troduced psychiatric testimony indicating ful only if the jurisdiction fol-
that the defendant suffered from a mental lowed the Durham insani-
illness. ty test, pursuant to which a
defendant is entitled to ac-
Which of the following, if proved by the de- quittal if his crime was the
fense, would most likely relieve the defen- product of mental disease
dant of criminal responsibility? or defect. (C) is wrong be-
cause it presents a valid de-
A The defendant's actions were a product of fense under the M'Naghten
his mental illness. rule, which provides for ac-
quittal if a disease of the
B The defendant could not appreciate the mind caused a defect of rea-
criminality of killing the acquaintance, or son, such that the defen-
he could not conform his conduct to the dant lacked the ability at the
requirements of the law. time of his actions to ei-
ther: (i) know the wrongful-
C The defendant did not know that killing ness of his actions; or (ii)
51 / 184
Barbri MBE questions
Study online at https://quizlet.com/_6umlok
the acquaintance was wrong, or he could understand the nature and
not understand the nature and quality of his quality of his actions. (D) is
actions. wrong because it presents
the irresistible impulse test,
D The defendant was unable to control him- which provides for acquittal
self or conform his conduct to the law. if, because of mental illness,
the defendant was unable
to control his actions or to
conform his conduct to the
law. Note that the Model Pe-
nal Code test combines the
M'Naghten and irresistible
impulse tests. Thus, choic-
es (C) and (D) contain el-
ements of the Model Penal
Code test, but are not as
good as (B) because the
question asks for the set of
facts that gives the defen-
dant the greatest likelihood
of being relieved of criminal
liability. Therefore, (B), which
sets forth the complete test
used in the jurisdiction, is the
best choice.

59. A landowner included in his will a provision (A)


giving "all of my property, both real and
personal, wherever situated, to my widow The children have a contin-
for life, and after her death to any of our gent remainder. A remainder
children who may survive her." is a future interest created in
a transferee that is capable
What is the gift to the children? of taking in present posses-
sion on the natural termina-
A A contingent remainder. tion of the preceding estate
created in the same dispo-
B A vested remainder. sition. Note that, as a rule
of thumb, remainders always
C A shifting executory interest. follow life estates. A remain-
52 / 184
Barbri MBE questions
Study online at https://quizlet.com/_6umlok
der will be classified as con-
D Void, as violating the Rule Against Perpe- tingent if its taking is subject
tuities. to a condition precedent, or
it is created in favor of un-
born or unascertained per-
sons. Here, the interest in
the children follows a life es-
tate and is a remainder be-
cause it is capable of tak-
ing in possession on the nat-
ural termination of the pre-
ceding estate. It is subject
to the condition precedent
of surviving the landowner's
widow and, additionally, is in
favor of unascertained per-
sons Thus, the interest is a
contingent remainder. (B) is
incorrect because a vested
remainder can be created in
and held only by ascertained
persons in being, and can-
not be subject to a condition
precedent. (C) is incorrect
because a shifting executory
interest is one that divests
the interest of another trans-
feree; i.e., it cuts short a prior
estate created by the same
conveyance. The gift to the
children does not divest the
interest of the widow; she re-
tains a life estate in the prop-
erty. The children's interest
takes in possession only on
the natural termination of the
widow's estate (i.e., at her
death).

53 / 184
Barbri MBE questions
Study online at https://quizlet.com/_6umlok
60. A beneficiary has filed a petition in the pro- (C)
bate court to contest the validity of a tes-
tator's will. The beneficiary contends that The judge should rule this
when the testator executed the will eight affidavit to be inadmissi-
years before, he had a severe mental ill- ble hearsay. This affidavit is
ness and was incapable of forming a valid clearly hearsay, and there
testamentary intent. In support of this con- is nothing in the facts that
tention, the beneficiary seeks to offer an shows that it is admissi-
affidavit prepared by the testator's former ble under any of the ex-
attorney, which states that she was asked ceptions to this rule. Hence,
to prepare a will for the testator just four (A) is wrong. (B) is wrong
months before this will was made. The attor- because the observations
ney had refused to do so because it was her of the attorney would not
opinion that the testator seemed incoherent be deemed a "communi-
and paranoid. cation received from the
client." Also, while the at-
How should the judge rule on the admissi- torney-client privilege gen-
bility of this affidavit? erally survives the client's
death, it does not apply to
communications relevant to
A Admissible. an issue between parties
who are claiming through the
B Inadmissible, as being violative of the at- same deceased client, such
torney-client privilege. as in the probate proceed-
ings here. (D) is incorrect
C Inadmissible, because it is hearsay not because a lay person could
within any exception. probably testify to her opin-
ion in this situation since it is
D Inadmissible, because it is improper opin- rationally based on her own
ion evidence. perception, it is helpful to a
determination of a fact in is-
sue, and it is not based on
scientific, technical, or other
specialized knowledge.

61. A homeowner entered into a written agree- (D)


ment with a contractor whereby the contrac-
tor agreed to completely remodel the home- The contractor can recover
owner's bathroom "to her specifications" at $2,000 as lost profits plus
54 / 184
Barbri MBE questions
Study online at https://quizlet.com/_6umlok
a cost of $10,000. The homeowner's speci- the $4,000 in costs he in-
fications were highly detailed and required curred before the homeown-
custom-made fixtures that would not be us- er breached the contract.
able in other bathroom remodeling jobs. The purpose of a damages
The contractor ordered the custom-made remedy is to give compen-
fixtures and paid $4,000 for them when they sation for the breach; i.e.,
were delivered to his place of business. Fig- to put the nonbreaching par-
uring up the cost of the fixtures and la- ty where he would have
bor, the contractor estimated that he would been had the promise been
make a total profit of $2,000 on the job after performed. In most cases,
payment for materials and workers. Before the plaintiff's standard mea-
the contractor began work on the project, sure of damages will be
but after he had paid for the fixtures, the based solely on an "expec-
homeowner told the contractor that she had tation" measure, i.e., suffi-
had a change of heart and would probably cient damages for him to buy
be selling the house the following year, and a substitute performance. A
so would not need a custom bathroom. The reliance measure of dam-
contractor made no attempt to sell the fix- ages, on the other hand,
tures to another contractor and filed suit awards the plaintiff the cost
against the homeowner for damages. of his performance, i.e., his
expenditures in performing
What is the contractor likely to recover? his duties under the con-
tract. In certain situations,
A Nothing, because he failed to mitigate an award of compensatory
damages. damages will contain both
an expectation and a re-
B His expectation damages of $2,000. liance component. In a con-
struction contract, if the own-
C $4,000, the cost of materials as restitution. er breaches the contract af-
ter the builder has already
D $2,000 as expectation damages, plus begun his performance, the
$4,000 in reliance damages. builder will be entitled to
any profit he would have de-
rived from the contract plus
any costs he has incurred to
date. The contractor has be-
gun performance by order-
ing and purchasing the cus-
tom-made fixtures at a cost
55 / 184
Barbri MBE questions
Study online at https://quizlet.com/_6umlok
of $4,000. Because they are
usable only for the home-
owner's purposes, their cost,
which is treated just like
any other expenditure of la-
bor and material in a par-
tially completed construction
contract, can be recovered
as reliance damages. The
other element of his re-
covery is the $2,000 profit
that he would have derived
from the contract—his ex-
pectation damages. His to-
tal recovery will therefore be
$6,000.

62. A homeowner, a citizen of State A, hired (C)


an electrician, a citizen of State B, to fix
the wiring in her basement and hired a gas The electrician's best argu-
worker, also a citizen of State B, to install a ment is that the gas work-
new gas stove in her kitchen. Unfortunately, er's claim against the electri-
the home caught fire and burned down while cian is not a proper third-par-
they were both working on their separate ty claim. Under Rule 14,
jobs. The homeowner sued the gas worker a defendant may assert a
for negligence in federal court in State A, third-party claim against "a
seeking $100,000. The homeowner promptly nonparty who is or may
served the gas worker, and the gas worker be liable to it for all or
timely filed an answer with the court. One part of the claim against it."
month after filing the answer, the gas worker In other words, a third-par-
moved to file and serve a third-party com- ty claim must be a deriv-
plaint against the electrician, alleging that ative claim; the third-par-
the electrician was the sole cause of the ty plaintiff must be seek-
accident. ing indemnification or con-
tribution from the third-par-
Which of the following arguments is most ty defendant. Here, the gas
likely to achieve the electrician's goal of dis- worker's claim is not that
missal of the third-party complaint? the electrician must indem-
nify him or that the elec-
56 / 184
Barbri MBE questions
Study online at https://quizlet.com/_6umlok
A The gas worker's motion for leave to file trician is a joint tortfeasor
a third-party complaint is untimely and thus who may be jointly liable un-
should be denied as a matter of law. der principles of contribution.
Rather, the gas worker is al-
B The court does not have subject matter leging that he (the gas work-
jurisdiction over the third-party complaint er) is not liable and that the
because the electrician's claim and the gas electrician is. Because the
worker's claim do not arise from a common claim is not derivative, it is
nucleus of operative fact. not properly asserted as a
third-party claim under Rule
C The gas worker's claim against the electri- 14. (A) is incorrect. A defen-
cian is not a proper third-party claim. dant may serve a third-par-
ty complaint as of right with-
D Dismissing the gas worker's claim will not in 14 days of serving his
impede his ability to protect his rights in a original answer. Thereafter,
separate action. he must make a motion to
serve the complaint, and
it is within the trial court's
discretion whether to grant
or deny the motion. Here,
it is unlikely that a court
would deny a defendant's
motion to serve a third-par-
ty complaint at such an ear-
ly stage of the proceed-
ing. (B) is incorrect because
both claims arose from a
single occurrence: the fire
that burned down the home-
owner's house. Thus, there
would be supplemental juris-
diction for the gas worker's
claim because it arises from
the same set of facts as the
homeowner's claim against
the gas worker, which is
based on diversity of citizen-
ship. (D) the gas worker's
ability to bring a separate ac-
57 / 184
Barbri MBE questions
Study online at https://quizlet.com/_6umlok
tion against the electrician
is not a compelling reason
for dismissing a properly as-
serted third-party claim

63. A statute in the jurisdiction, which was en- (B)


acted with the express purpose of prevent-
ing public employees from taking advan- The illegal alien cannot be
tage of the status of illegal aliens, made it a convicted of a crime under
felony to accept money or other benefits in the statute because it was
exchange for issuing a state identification enacted for his protection,
card. During an undercover investigation, and the clerk cannot be con-
an illegal alien was recorded offering $500 victed of conspiracy under
to a clerk in exchange for issuance of a card. the statute because the il-
The clerk agreed to the deal and later that legal alien, who would oth-
day exchanged the card for the money, after erwise be liable as an ac-
which both parties were arrested. complice, is not subject to
conviction because of a leg-
In a jurisdiction following the common law islative intent to exempt him.
approach to conspiracy, which of the follow- If a statute is intended to
ing statements is correct? protect members of a lim-
ited class from exploitation
A The clerk can be convicted of violating or overbearing, members of
the statute and conspiracy to violate the that class are presumed to
statute, and the illegal alien can be convict- have been intended to be im-
ed of no crime. mune from liability, even if
they participate in the crime
B The clerk can be convicted only of violat- in a manner that would oth-
ing the statute, and the illegal alien can be erwise make them liable.
convicted of no crime. Thus, the illegal alien would
not be liable as an accom-
C The clerk can be convicted only of violat- plice under the statute, mak-
ing the statute, and the illegal alien can be ing (C) incorrect. The clerk
convicted as an accomplice to violation of cannot be convicted of con-
the statute. spiracy. One of the implica-
tions of the common law re-
D The clerk can be convicted of violating quirement that there be at
the statute and conspiracy to violate the least two guilty parties in
a conspiracy arises when
58 / 184
Barbri MBE questions
Study online at https://quizlet.com/_6umlok
statute, and the illegal alien can be convict- the crime involves members
ed of conspiracy to violate the statute. of a class protected by the
statute. If members of a con-
spiracy agree to commit an
act that violates a statute
designed to protect persons
within a given class, a per-
son within that class not only
cannot be guilty of the crime
itself, as discussed above,
but also cannot be guilty of
a conspiracy to commit the
crime. (D) is therefore in-
correct. Because the mem-
ber of the protected class
cannot be guilty of conspir-
acy, if no other guilty par-
ty exists, the other member
of the agreement cannot be
guilty of criminal conspiracy
because there were not two
guilty parties to the agree-
ment. Thus, because the ille-
gal alien cannot be convict-
ed of conspiracy under the
statute, neither can the clerk.
(A) is therefore incorrect and
(B) is correct.

64. If more than _______ months' imprisonment (B)


is authorized, the offense is considered "se-
rious" for determining whether a defendant An offense is considered se-
has a constitutional right to a jury trial. rious, making a jury trial
a constitutional right, when
A nine more than six months' im-
prisonment is authorized.

B six

59 / 184
Barbri MBE questions
Study online at https://quizlet.com/_6umlok

C three

D twelve

65. Which of the following statements regard- (B)


ing speedy trials is correct?
Whether a defendant assert-
A Delays caused by counsel assigned by ed his right is one factor in
the court to the defendant should ordinarily determining whether the de-
be attributed to the state. fendant's right to a speedy
trial was violated. The de-
termination is made by an
B One factor in determining whether a de- evaluation of the totality of
fendant's right to a speedy trial was violated the circumstances, and the
is whether the defendant asserted his right. following factors should be
considered: (i) length of the
delay, (ii) reason for the de-
C The remedy for a violation of the consti- lay, (iii) whether the defen-
tutional right to a speedy trial is dismissal dant asserted his right, and
without prejudice. (iv) prejudice to the defen-
dant.

D A defendant is entitled to speedy trial re- The remedy for a violation of


lief for the period between the dismissal of the constitutional right to a
charges and later refiling. speedy trial is dismissal with
prejudice.

Delays caused by counsel


assigned by the court to the
defendant should ordinarily
be attributed to the defen-
dant and NOT to the state.

A defendant is NOT enti-


tled to speedy trial relief for
the period between the dis-

60 / 184
Barbri MBE questions
Study online at https://quizlet.com/_6umlok
missal of charges and later
refiling

66. A defendant was convicted after a jury trial (C)


of violation of federal statutes prohibiting
the sale of automatic weapons to foreign The appeals court should af-
nationals. It was established at trial that firm the trial court because
the defendant had purchased a number of the right to remain silent
stolen United States Army heavy machine does not include the right
guns and attempted to ship them abroad. to protect others from in-
The trial court expressly based its imposi- crimination. The defendant
tion of the maximum possible sentence for was not privileged to refuse
the conviction on the defendant's refusal revealing the names of
to reveal the names of the persons from the stolen weapon sellers.
whom he purchased the stolen weapons. The United States Supreme
His counsel argues that this consideration Court held, in Roberts v.
is reversible error. United States (1980), that a
defendant's refusal to coop-
If the defendant appeals the sentence im- erate with an investigation
posed, what should the appeals court do? of the criminal conspiracy
of which he was a member
may properly be considered
A Reverse the trial court, because the con- in imposing sentence. This
sideration of the defendant's silence vio- is because the Fifth Amend-
lates his Fifth Amendment privilege against ment right to remain silent
self-incrimination. does not afford a privilege to
refuse to incriminate others.
B Reverse the trial court, because the con- (C) is therefore correct and
sideration of collateral circumstances in (A) is incorrect. (B) is incor-
sentencing violates his due process rights. rect because the court's con-
sideration of the defendant's
C Affirm the trial court, because the right to refusal to cooperate does
remain silent granted by the Fifth Amend- not violate due process. (D)
ment does not include the right to protect is not an accurate statement
others from incrimination. of the law.

D Affirm the trial court, because citi-


zens must report violations of the criminal
statutes.
61 / 184
Barbri MBE questions
Study online at https://quizlet.com/_6umlok

67. A man was driving very erratically when (B)


he was stopped by state troopers and ar-
rested for drunk driving. He was advised The question was improper
of his constitutional rights and elected to because it effectively com-
remain silent. At trial for his drunk driving ments on the defendant's
charge, the man testified in his own defense, post-arrest silence and thus
stating that he had just left his doctor's violates his right to due
office and had been administered medica- process of law. A prosecutor
tion without being told that it would seri- may not comment on the de-
ously and immediately hamper his coordi- fendant's silence after being
nation. On cross-examination, the prosecu- arrested and receiving Mi-
tor asked whether the defendant just made randa warnings. The warn-
up this medication story after the fact to ings carry an implicit assur-
evade legitimate liability for driving while ance that silence will car-
intoxicated and the man said he had not. The ry no penalty. Thus, the de-
prosecutor then asked why the defendant fendant's invocation of his
had not told the arresting officer about the right to remain silent cannot
medication, and defense counsel objects. be used as evidence against
him at trial, nor can he be
The trial court should rule that the question questioned or cross-exam-
is: ined about his decision to
remain silent, even if he
A Improper, because to require the defense testifies at trial. Thus, (B)
to inform the prosecution of defendant's is the correct answer, and
testimony prior to trial would be unconsti- (C) is wrong. (Distinguish: If
tutional pretrial discovery. the defendant fails to dis-
close potential exculpatory
B Improper, because use of defendant's evidence to the police after
post-arrest silence violates his right to due waiving his right to remain
process of law. silent, he may be cross-ex-
amined at trial on that fail-
C Proper, because defendant's silence was ure in an effort to show
not used as direct evidence but only for that it is a recent fabrica-
impeachment on cross-examination. tion. In this case, however,
the defendant invoked his
D Proper, because defendant's post-arrest right to remain silent.) (A)
silence is a prior inconsistent statement is wrong; the question is
which is admissible to show recent fabrica- improper, but not because
tion. it would be unconstitutional
62 / 184
Barbri MBE questions
Study online at https://quizlet.com/_6umlok
pretrial discovery. (D) is also
wrong. Although statements
made without proper Miran-
da warnings can sometimes
be used to impeach, the de-
fendant's silence would not
be classified as a prior state-
ment.

68. Jeopardy attaches in a jury trial when: (B)

Jeopardy attaches in a jury


A Opening statements begin trial when the jury is empan-
eled and sworn. Under the
Fifth Amendment right to be
B The jury is empaneled and sworn free of double jeopardy, a de-
fendant may not be retried
for the same offense once
C The first witness is sworn jeopardy has attached.

Jeopardy does not attach in


D The judge delivers jury instructions a jury trial when the first wit-
ness is sworn. This is when
jeopardy attaches in a bench
trial, not in a jury trial.

By the time opening state-


ments begin or the judge de-
livers jury instructions, jeop-
ardy has already attached.

69. Two friends entered a bar looking to get (B)


money to pay off a loan shark, but with no
plan how to do so. They struck up conver- The first friend can be con-
sations with two women. The first friend left victed of common law rob-
the bar, having induced one of the women bery, but the second cannot
to return home with him. Once in his house, be convicted. Robbery is the
the first friend told the woman that she taking and carrying away of
would not be allowed to leave unless she the personal property of an-

63 / 184
Barbri MBE questions
Study online at https://quizlet.com/_6umlok
gave him all of her money. Fearing for her other from the other's person
safety, the woman gave him all of the cash or presence by force or intim-
she had in her possession. Meanwhile, the idation. In the instant case,
second friend remaining at the bar noticed the first friend committed a
that the other woman left her credit card on robbery when he threatened
the counter. When the woman looked away, the woman and told her that
the friend picked up the credit card and she could not leave without
put it into his pocket. Shortly thereafter, the giving him all of her mon-
woman realized her card was gone and ac- ey. The only issue would be
cused the man of taking it. The man pretend- whether the asportation el-
ed to be insulted, slapped the victim, and ement is satisfied. Howev-
went off with the credit card in his pocket. er, the asportation element
is satisfied by any slight
Which of the two friends can be convicted moving, and it is likely that
for common law robbery? the first friend moved the
money at some point dur-
A Both can be convicted. ing the robbery. Thus, a jury
could find the first friend
B The first friend can be convicted, but the guilty of robbery. In the case
second cannot be convicted. of the second friend, how-
ever, a conviction for rob-
C The second friend can be convicted, but bery is unlikely. Although a
the first cannot be convicted. close call, the taking or re-
tention of the property was
D Neither of the two friends can be convict- not by force or intimidation in
ed of robbery. the second case. The crime
against the property was al-
ready completed when the
man slapped the victim. Fur-
thermore, the slap was not
to prevent the woman from
physically taking the cred-
it card back; rather, it was
a ruse used to deflect the
accusation that the second
friend took the credit card.
Thus, (A) is wrong, and this
provides a second reason
why (C) is incorrect.
64 / 184
Barbri MBE questions
Study online at https://quizlet.com/_6umlok

70. A defendant held up a gasoline station. Dur- (C)


ing the robbery, he shot and killed a cus-
tomer who attempted to apprehend him. The The motion to dismiss
defendant was prosecuted for premeditated should be denied. For pur-
murder and convicted. Thereafter, he was in- poses of the Double Jeop-
dicted for armed robbery of the station. Be- ardy Clause, two crimes do
fore the trial, his attorney moved to dismiss not constitute the "same of-
the indictment on the ground that further fense" if each crime re-
proceedings were unconstitutional because quires proof of an addi-
of the defendant's prior conviction. tional element that the oth-
er crime does not require,
Should the motion to dismiss be granted? even though some of the
same facts may be neces-
A Yes, because once the defendant was con- sary to prove both crimes.
victed on any of the charges arising out of Here, even though the same
the robbery, the prosecution was constitu- facts are involved for both
tionally estopped from proceeding against crimes, the robbery charge
the defendant on any charge stemming from requires proof of a taking by
the same transaction. force but not a death, while
the murder charge requires
B Yes, because the Double Jeopardy Clause proof of a death but not of
prohibits a subsequent trial on what is es- a taking of property. Thus,
sentially a lesser included offense. (C) is correct and (A) is in-
correct. (B) is incorrect be-
C No, because there is no constitutional re- cause armed robbery is not
quirement that all known charges against a a lesser included offense of
defendant be brought in the same prosecu- premeditated murder. (D) is
tion. incorrect because the pros-
ecution would be estopped if
D No, because estoppel does not apply violation of one statute con-
when a defendant is charged with violating stituted a lesser included of-
two different statutes. fense of the other statute.

71. A woman was arrested outside of a house (C)


shortly after she had broken in and stolen
some jewelry. She was indicted for larce- The woman's motion should
ny and later for burglary. She was tried be denied because the Dou-
on the larceny indictment and convicted. ble Jeopardy Clause does
Thereafter, she was brought to trial on the not prohibit the second pros-
65 / 184
Barbri MBE questions
Study online at https://quizlet.com/_6umlok
burglary indictment. Relying on the Double ecution. The Double Jeop-
Jeopardy Clause of the Constitution, the ardy Clause of the Fifth
woman moves to dismiss the indictment. Amendment provides crimi-
nal defendants with the right
Should the motion be granted? to be free of double jeop-
ardy for the same offense.
A Yes, because the Double Jeopardy Clause However, two crimes do not
requires that all offenses arising out of constitute the same offense
the same transaction be adjudicated in the if each crime requires proof
same trial. of an additional element that
the other crime does not re-
B Yes, because the Double Jeopardy Clause quire, even though some of
allows the imposition of separate sentences the same facts may be nec-
for separate offenses occurring during the essary to prove both crimes.
same criminal episode only if the offenses Here, larceny requires a
are tried together. taking and carrying away
of the property of another,
C No, because larceny and burglary are which burglary does not re-
offenses that may constitutionally be tried quire, and burglary requires
and punished separately, even if they arise a breaking and entry, which
out of the same transaction, because each larceny does not require.
requires proof of a fact that the other does Hence, they are distinct of-
not. fenses for purposes of the
Double Jeopardy Clause. (A)
D No, because the only protection double and (B) are incorrect be-
jeopardy affords to a defendant charged cause the Supreme Court
with multiple counts is under the doctrine of does not use a "same trans-
collateral estoppel. action" or "same episode"
test suggested by these an-
swer choices; instead, the
Blockburger test is used re-
gardless of whether the two
offenses were tried together
at a single trial or at sep-
arate trials. (D) is incorrect
because while double jeop-
ardy also protects against
inconsistent factual determi-
nations at a subsequent trial,
66 / 184
Barbri MBE questions
Study online at https://quizlet.com/_6umlok
it protects against multiple
prosecutions as well, as long
as the crime is the "same of-
fense."

72. During a presidential campaign, a candi- (A)


date's campaign manager secretly engaged
in activities that may have violated both The pardon is valid. Article
state and federal laws. After the candi- II, Section 2 of the United
date was elected President, the FBI inves- States Constitution grants
tigated the manager's activities as well as the President the power to
whether the President was involved. After grant reprieves and pardons
the campaign manager was indicted in fed- for offenses against the Unit-
eral court, but before trial, the President ed States, except in cases
granted a blanket pardon to the campaign of impeachment. This par-
manager for "all federal crimes that may don power is not subject to
have been committed in the past 20 years." control by Congress, and it
includes the power to com-
Is the pardon valid? mute a sentence on any con-
ditions the President choos-
A Yes, because the pardon power is an un- es (as long as the conditions
qualified power (except as to impeachment). do not offend some other
constitutional provision). (B)
B No, because a presidential pardon that in- is incorrect because even
terferes with an inquiry into the President's if the action of issuing the
own actions constitutes an obstruction of pardon amounted to the
justice. crime of obstruction of jus-
tice (a questionable assump-
C No, presidential pardons must relate to tion), the pardon itself would
specific crimes; the President cannot issue not be invalidated. The pow-
blanket pardons. er to pardon is a constitution-
al power, superior to laws
D No, because a President's pardon power found in statutes. (C) is in-
is limited to crimes that allegedly took place correct because the pardon
while the President is in office. power is not so limited. Blan-
ket pardons are valid. (D) is
also incorrect because the
pardon power is not so lim-
ited. Presidents may pardon
67 / 184
Barbri MBE questions
Study online at https://quizlet.com/_6umlok
offenses that occurred be-
fore the President took of-
fice.

73. Shortly after a professor at a state univer- (D)


sity completed her second year of teach-
ing, she was informed that her contract was The strongest argument the
not being renewed for the following year. professor could make is that
By state law, a professor does not acquire the decision was not based
tenure until after she has completed three on her ability to teach. The
consecutive years of teaching. Before ac- professor is an at-will em-
quiring tenure, state law does not require ployee, and under most cir-
either a statement of reasons or a hearing cumstances may be dis-
when a professor's contract is not renewed, charged "for any reason or
and the university administration refused to no reason at all." Thus,
give either to the professor. normally, evidence regard-
ing the motives for dismissal
Which of the following, if established, sets is irrelevant. The question
forth the strongest constitutional argument here, however, is what the
that the professor could make to compel strongest argument is that
the university to furnish her a statement of the professor could make,
reasons for the failure to rehire her and an and (D) creates at least an
opportunity for the hearing? inference that an impermis-
sible motive might be pre-
sent (gender, free speech,
A She purchased a home in anticipation etc.). (A) is a weaker an-
of renewal of her contract, because most swer because the profes-
professors who had taught two years were sor has no property interest
rehired. in continued employment; a
mere expectation of con-
B She had been voted the most popular tinued employment is not
professor on campus in each of her first two enough, even when coupled
years of teaching. with reliance (her buying a
house). There must be a
C She was the only teacher at the university legitimate claim or entitle-
whose contract was not renewed that year. ment—created by a contract
or clear policy—that employ-
D There is evidence to indicate that the de- ment can be terminated only
for cause. The bases alleged
68 / 184
Barbri MBE questions
Study online at https://quizlet.com/_6umlok
cision not to rehire the professor was not in (B) are arguably irrele-
based on her ability to teach. vant; the professor's popu-
larity may or may not have
anything to do with her abil-
ity, and even if it does, she
remains an at-will employee.
(C) might under some cir-
cumstances offer an argu-
ment, but there could be any
number of valid explanations
for keeping others and letting
a particular professor go, in-
cluding budget constraints,
subject needs, etc. (D) is,
accordingly, the strongest of
the possibilities.

74. A credit card company filed a civil action (C)


against a consumer in federal district court
seeking to recover the unpaid balance on An entry of default may be
the consumer's account. The credit card set aside for "good cause
company properly served process on the shown." Although not specif-
consumer, but the consumer failed to file ically required by the Federal
or serve a timely answer to the complaint. Rules, a majority of courts
The clerk of court on motion of the credit also will require some show-
card company made an entry of default, but ing of a meritorious defense.
default judgment has not been entered. (A) is incorrect because the
grounds are not limited to
On which grounds may the court set aside the grounds required for set-
the entry of default? ting aside regular court judg-
ments. (B) is incorrect be-
A For the same limited grounds for which cause that answer is also
any court judgment may be set aside. too limiting on the concept
of "good cause shown." For
B Only if the court finds that the consumer example, a default might re-
could not have filed and served a timely sult from an honest mistake
answer despite using reasonable diligence. of the attorney, but the entry
of default may nonetheless
C If the consumer demonstrates that there be set aside if the attorney
69 / 184
Barbri MBE questions
Study online at https://quizlet.com/_6umlok
was good cause for his failure to file and acts promptly to correct the
serve a timely answer and that he has a mistake. (D) is incorrect be-
viable defense. cause a majority of courts
will also require a showing of
D On any grounds that the court, in its dis- a meritorious defense.
cretion, finds just.

75. A landowner owned a beachfront lot and (A)


home in a subdivision occupying several
hundred acres near a lake. The recorded The express easement for
subdivision plan grants to each owner in the landowner's old proper-
the subdivision an easement to use the pri- ty benefits that property only
vate roads therein for personal ingress and and cannot be used for the
egress. landowner's expanded ac-
cess to the new property. An
Following seismic activity in the area, the easement is a liberty, privi-
level of the lake dropped substantially, ex- lege, or advantage that one
posing a considerable amount of land be- may hold in the lands of an-
tween the new shoreline and the old beach- other. The holder of an ease-
front. It was judicially determined that this ment has the right to use
"new" land belonged to the county, which a tract of land (called the
put portions of it up for sale. The landown- servient tenement) for a spe-
er purchased the land extending from her cial purpose; e.g., laying util-
old property line to the new shoreline, and ity lines, or for ingress and
constructed a boat launching ramp on the egress. An easement can be
new property. She then permitted persons created, as in this question,
who did not own land in the subdivision to by express grant. If the par-
drive through her old property to reach the ties to the original creation
boat launching ramp on her new property, of the use specifically state
and thus to utilize the lake, for a small fee. the location of the easement,
The homeowners' association brought suit its dimensions, and the spe-
against the landowner, seeking to enjoin her cial use or limits to such
from using or permitting nonresidents of use, the courts will honor
the subdivision from traveling its streets to this expression of specific in-
reach the boat launching ramp. tent. Absent specific limita-
tions, it will be assumed that
How should the court rule? the parties intend that the
easement meet both present
A For the homeowners, because the scope and future reasonable needs
70 / 184
Barbri MBE questions
Study online at https://quizlet.com/_6umlok
of the easement granted to the landowner of the dominant tenement.
as an owner in the subdivision does not However, a basic change in
extend to the use that she is making of the
the nature of the use is
new property. not allowed. The landown-
er's easement by express
B For the landowner, because she has an grant merely allows her to
express easement over the streets of the use the private roads in the
subdivision. subdivision for her personal
ingress and egress to and
C For the landowner, because she has an from her beachfront proper-
easement by necessity as to the new prop- ty. The use of the easement
erty over the streets of the subdivision. for access to a new boat
launching ramp for which a
D For the landowner, because she has an fee is charged goes beyond
implied easement over the streets of the the specific language of the
subdivision benefiting the new property grant (and arguably beyond
since it abuts her old property. the reasonable needs of the
dominant tenement). There-
fore, the homeowners will be
able to prevent use of the
subdivision streets to reach
the boat launching ramp.

76. A man and a woman were in a two-car (D)


traffic accident. Immediately after the acci-
dent, the man was treated by a physician The woman is entitled to dis-
at a nearby hospital's emergency room. The covery regarding the physi-
man later filed a negligence action againstcian's observations, opin-
the woman in federal district court, seeking
ions, and treatment of the
compensatory damages. The woman now man because the physician
seeks discovery regarding the emergency developed opinions about
room physician's observations, opinions, the man's injuries for pur-
and treatment of the man. poses other than litigation
or trial. (A) is incorrect be-
Is the woman entitled to discovery regard- cause the physician's obser-
ing that information? vations, opinions, and treat-
ment of the man are relevant
A No, because such discovery is not rele- to the man's claims and the
vant to the claim or defense of a party. driver's defenses. (B) is in-
71 / 184
Barbri MBE questions
Study online at https://quizlet.com/_6umlok
correct because parties may
B No, because the woman is not entitled to obtain discovery from indi-
obtain discovery from persons who are not viduals with knowledge of
parties to the action. any discoverable matter, not
just parties to the action.
C Yes as to the physician's observation and (C) is incorrect because it is
treatment, but the physician's opinions are not applicable to the situa-
discoverable only if the man intends to call tion here, where the physi-
the physician as an expert witness at trial. cian was not retained in an-
ticipation of litigation or trial.
D Yes, because the physician observed and
treated the man and developed opinions
about the man's injuries for purposes other
than litigation or trial.

77. After picking up a load of hazardous chem- (B)


ical waste, a truck driver for a waste man-
agement company set out on the road to his If the driver was effectively
next stop. However, he had failed to secure required by statute to take
the latch on the back panel of the truck. an occasional quick look at
Consequently, the panel opened while the his speedometer to make
truck was on the road, and a metal canister sure that he was comply-
full of chemical waste fell onto the road. A ing with appropriate speed
car struck the canister, causing the car to limits, then his momentary
veer off the road and injure the driver. The glance at the speedometer
driver filed suit against the company for his in the instant case would, as
injuries. a matter of law, not consti-
tute negligent conduct. Be-
The jurisdiction in which the above events cause this is a matter of law,
took place has adopted a rule of partial com- the judge would be autho-
parative negligence. At trial, the driver of rized to correct this aspect of
the car admitted that he had momentarily the jury's verdict. If the dri-
taken his eyes off the road to look at his ver is thus found to be not
speedometer. When he had looked up again, negligent in this matter, his
the canister was there and he could not stop recovery will not be reduced.
in time. The jury found that the company, (A) is incorrect because in
through its truck driver, had acted willfully most states that have adopt-
and wantonly and was 90% at fault, while the ed comparative negligence,
driver of the car was 10% at fault. The driver the plaintiff's negligence will
72 / 184
Barbri MBE questions
Study online at https://quizlet.com/_6umlok
filed a motion for judgment notwithstanding be considered even in cas-
the verdict, seeking recovery for 100% of his es where the defendant has
damages. acted willfully and wanton-
ly. (C) is incorrect because
If the judge grants the motion, what is the the fact that the defendant is
most likely reason? more than 50% at fault does
not mean that the plaintiff is
A A plaintiff's comparative negligence is not entitled to receive 100% of
taken into account in cases of willful and his damages from the de-
wanton conduct by the defendant. fendant in a partial compar-
ative negligence jurisdiction.
B A state ordinance mandating motorists to It only means that the plain-
stay within the posted speed limit requires tiff's recovery is not totally
as a matter of law an occasional glance at defeated. (D) is incorrect be-
the speedometer. cause, although the trans-
portation of chemical waste
C The company was more than 50% at fault. would probably be consid-
ered an abnormally danger-
D The company was engaged in an abnor- ous activity, liability for con-
mally dangerous activity. ducting an abnormally dan-
gerous activity attaches only
if the harm results from the
kind of danger to be antici-
pated from such activity; i.e.,
the injury must flow from the
normally dangerous propen-
sity of the activity. The can-
ister falling from the truck
is not the "normally danger-
ous propensity" of transport-
ing chemical waste.

78. The criminal statutes of the state define (A)


manslaughter and murder as they were de-
fined at common law. As to insanity, the The court should reverse
state has the following provision: the defendant's conviction
because Instruction #6 re-
"Under the defense of insanity a defendant quires the defendant to dis-
may be entitled to acquittal if, because of prove one of the elements
73 / 184
Barbri MBE questions
Study online at https://quizlet.com/_6umlok
mental illness, the defendant was unable to of murder. Due process re-
control his or her actions or to conform his quires in criminal cases that
or her conduct to the law." the state prove guilt be-
yond a reasonable doubt.
The defendant was put on trial in the state The prosecution has the bur-
for the murder of his wife and her co-work- den of proving all of the ele-
er. The evidence at trial established that ments of the crime charged.
the defendant's wife was having an affair Thus, if malice aforethought
with the co-worker, and that the defendant is an element of murder
learned of it and killed the pair. The defen- and voluntary manslaughter
dant did not take the stand in his own de- is distinguished from murder
fense. In his closing statement to the jury, by the existence of adequate
the defendant's attorney made a statement, provocation, the defendant
"Ladies and gentlemen, you must consid- cannot be required to prove
er that there are some things that would that he committed the homi-
provoke any one of us to kill, and there cide in the heat of passion
are things that make one unable to control (i.e., with adequate provo-
one's actions." The defendant's attorney re- cation). Such a requirement
quested that the judge give the jury instruc- would impose on the defen-
tions on manslaughter and on insanity, and dant the burden of disprov-
the judge agreed to do so. The judge also ing the element of malice
issued the following instructions: aforethought, because "heat
of passion" negates malice.
"INSTRUCTION #6: In order to mitigate an Although the defendant can
intentional killing to voluntary manslaugh- be given the burden of go-
ter, the burden of proof is on the defendant ing forward with some ev-
to establish that adequate provocation ex- idence on the provocation
isted." issue, once he has done
so, the prosecution bears
"INSTRUCTION #8: Insanity is an affirmative the burden of proving that
defense and the burden of proof is on the the killing was not done
defendant to establish that such insanity in the heat of passion. In
existed at the time of the killing." the case at issue, Instruc-
tion #6 requires a defen-
The jury found the defendant guilty of mur- dant to prove that he com-
der, and he appealed. He asserts that the mitted the intentional killing
jury instructions violated his rights under under adequate provocation.
the federal Constitution. At common law, and con-
sequently in the state, mal-
74 / 184
Barbri MBE questions
Study online at https://quizlet.com/_6umlok
How should the appeals court rule? ice aforethought is an ele-
ment of murder. Therefore,
A Reverse the defendant's conviction, be- this instruction in effect re-
cause Instruction #6 was improper. quires the defendant to dis-
prove the element of malice
B Reverse the defendant's conviction, be- aforethought, thereby reliev-
cause Instruction #8 was improper. ing the state of its burden of
proving all elements of the
C Reverse the defendant's conviction, be- crime. As discussed above,
cause both Instructions #6 and #8 were im- such an instruction cannot
proper. pass constitutional muster.
On the other hand, for an
D Uphold the defendant's conviction, be- affirmative defense such as
cause neither Instruction #6 nor Instruction insanity, it is permissible to
#8 was improper. impose the burden of proof
on the defendant. Thus, In-
struction #8 does not af-
fect the state's obligation to
prove all elements of the
crime, and is permissible un-
der the general principles
mentioned above.

79. An environmental group, wishing to stop the (D)


issuance by a federal agency of a mining
permit to a coal company, commences an Under Rule 24, a nonparty
action in federal court against the federal may intervene in an action
agency, seeking, among other things, a per- as a matter of right in two
manent injunction barring the issuance of situations. First, a nonparty
the permit to the coal company. may intervene when it has
If the coal company seeks to join the litiga- an unconditional right to do
tion as a matter of right, must the federal so by a federal statute. Sec-
court grant the motion? ond, a nonparty may inter-
vene if (i) it has an interest
A No, because intervention of outside par- in the property or transaction
ties is a matter within the sole discretion of the is the subject matter of
the judge. the action; (ii) the disposition
or resolution of the action
B No, unless the coal company has been may as a practical matter
75 / 184
Barbri MBE questions
Study online at https://quizlet.com/_6umlok
given an unconditional right to intervene by impair the nonparty's abili-
a federal statute. ty to protect its interest; and
(iii) the nonparty's interest is
C Yes, because the coal company has an not adequately protected by
interest in getting the mining permit. an existing party in the ac-
tion. Here, although no fed-
D Yes, unless the court concludes that the eral statute gives the coal
coal company's interest in getting the per- company the right to inter-
mit is adequately protected by the federal vene, it does have an inter-
agency. est at stake in the action—its
interest in getting the mining
permit issued to it—and its
ability to obtain the permit
will as a practical matter be
impaired if the environmen-
tal group succeeds in getting
an injunction against its is-
suance. Thus, the coal com-
pany should be allowed to
intervene unless the court
concludes that the feder-
al agency adequately repre-
sents the coal company's in-
terest.

80. At a waterfront bar, a college student sought (D)


to provoke a fight with a merchant seaman
by making insulting remarks. Eventually the Even though the student's
seaman had had enough and threw a punch words may have been in-
that connected to the student's jaw and sent tended to provoke the sea-
him sprawling to the floor. The seaman then man, this fact alone would
told the student that he wanted no further not justify the seaman's use
trouble. Getting up off the floor, the student of deadly force. A person
pulled a knife out of his pocket and charged may use deadly force in
at the seaman. Three other students were self-defense if he: (i) is with-
standing between the seaman and the exit out fault; (ii) is confronted
door. The seaman tried to dodge, but was with unlawful force; and (iii)
cut on the forearm by the student's knife. reasonably believes that he
The seaman immediately drew a gun and is threatened with imminent
76 / 184
Barbri MBE questions
Study online at https://quizlet.com/_6umlok
shot the student, killing him. The seaman death or great bodily harm.
was charged with murder. Generally, one who is at fault
for starting a confrontation
Which of the following points raised in the has no right to use force
seaman's defense will not be helpful for his in his own defense during
defense? that confrontation. However,
if the victim of the initial ag-
gression suddenly escalates
A The student had no reason to fear serious a relatively minor fight into
bodily injury when he drew the knife. one involving deadly force
and does not give the ag-
B The student's drawing of the knife consti- gressor a chance to with-
tuted an escalation of the fight. draw or retreat, the aggres-
sor may use deadly force in
C Three college students were standing be- his own defense. Here, al-
tween the seaman and the door, so there though the student instigat-
was no clear route of retreat. ed the hostile situation by
repeatedly insulting the sea-
D The student's comments were motivated man, the seaman's throwing
by a desire to provoke the seaman. of a punch probably calls for
his being characterized as
the aggressor. The student,
as the victim of the initial ag-
gression, escalated matters
by using a knife, especial-
ly because the seaman had
said that he wanted no fur-
ther trouble. This escalation
(which is the point stated in
choice (B)) entitled the sea-
man to employ deadly force
in his own defense against
the imminent threat of death
or great bodily harm posed
by the student's use of the
knife.

81. A developer owned a 240-acre parcel of land (C)


zoned for commercial and residential use.
77 / 184
Barbri MBE questions
Study online at https://quizlet.com/_6umlok
He prepared and recorded, after obtaining The court should issue
approval from all appropriate agencies, a the injunction because the
subdivision plan that included a commer- covenant runs with the land.
cial center and a number of lots for sin- A covenant will be enforce-
gle- and multi-family residences. The list able as an equitable servi-
of covenants, conditions, and restrictions tude—allowing a covenan-
recorded with the plan included provisions tee, covenantor, or succes-
that required every building constructed in sor to enforce the covenant
the subdivision to be of "simulated adobe in equity by way of injunc-
style" architecture approved in advance by tion—when there is (i) a
an association. A year later, the developer covenant in a writing satis-
sold many of the lots in the commercial cen- fying the Statute of Frauds,
ter, including several to a real estate firm. that (ii) touches and con-
Each deed prepared by the developer con- cerns the land (i.e., the effect
tained a reference to the design restriction of the covenant makes the
in the recorded plan. The developer also land more useful or valuable
sold almost all of the residential lots, the to the benefited party) and
deeds of which contained the same refer- that (iii) indicates an inten-
ence to the restriction. The following year, tion that the servitude exists,
the real estate firm sold one of its lots to and (iv) notice is given to fu-
a burger franchise. The deed contained no ture owners of the burdened
reference to the design restriction. The fran- land. Here, the covenant was
chise's prefabricated restaurant, complete in writing in the subdivision
with a giant burger logo mounted on the plan and presumably it sat-
roof, was constructed over the weekend. isfied the Statute of Frauds.
It touches and concerns the
A merchant, an original purchaser of one land—benefiting all of the
of the commercial lots, owned the lot next lots and burdening all of the
to the burger franchise. She did not learn lots. The intention to cre-
of construction of the restaurant until she ate the servitude is estab-
came in to work on Monday, and saw the lished by the writing and
giant burger logo. The merchant brings an can also be implied from
action seeking a mandatory injunction com- the common scheme for de-
pelling the burger franchise to demolish the velopment. There was suf-
restaurant. At trial, the merchant proves that ficient record notice of the
the burger franchise did not seek or obtain covenant because the plan
approval of the association for its building. was recorded and was noted
in all of the original deeds
Should the court issue the injunction? prepared by the developer,
78 / 184
Barbri MBE questions
Study online at https://quizlet.com/_6umlok
including the one in the burg-
er franchise's chain of title.
A No, because destruction of the restaurant Thus, the covenant is en-
would be a tremendous waste of resources. forceable and (C) is the best
answer.
B No, because the burger franchise's deed
contained no restriction on the type of build-
ing that could be constructed on the lot.

C Yes, because the restrictive covenant runs


with the land.

D Yes, unless the burger franchise can es-


tablish to the court's satisfaction that its
restaurant design has at least as much aes-
thetic merit as any "simulated adobe style"
design.

82. A landlord brought suit against a tenant in (D)


federal court for overdue rent payments on
a commercial lease. The landlord sought to
The plaintiff's suit is unlike-
recover on the six rent installments thatly to be successful because
were past due and unpaid at the time of the
the two suits arise out of
suit. The landlord won the case, and judg-
the same transaction or oc-
ment was entered in her favor. The lease has
currence. Claim preclusion
an acceleration clause that states that all
(res judicata) requires that
future rent payments become due if the ten-
(i) a valid, final judgment
ant falls behind three months or more. The
on the merits was entered
landlord now files suit against the tenant
in the first case; (ii) the
for the remaining rent payments. The tenant
cases were brought by the
moves to dismiss, asserting that the land-
same claimant against the
lord's claim is barred by claim preclusion
same defendant; and (iii) the
(res judicata) principles. same cause of action is in-
volved in the later lawsuit.
Should the tenant's motion to dismiss be Generally, a claimant is re-
granted? quired to assert all causes
of action arising out of the
same transaction or occur-
A No, because a landlord may choose when rence that is the subject mat-
79 / 184
Barbri MBE questions
Study online at https://quizlet.com/_6umlok
to sue on an acceleration clause. ter of the claim. In the situa-
tion of installment payments,
B No, because the two suits do not involve the claimant is required to
the same cause of action. sue on all installments due at
the time of the suit. If there is
C Yes, because a landlord may not sue on an acceleration clause, the
later installments of an installment contract. claimant must sue for all in-
stallments. Here, the plain-
D Yes, because the two suits arose out of the tiff should have sued for all
same transaction or occurrence. installments in the first law-
suit. Because she did not,
her second suit is barred by
claim preclusion. (A) is in-
correct because a plaintiff
may not choose when to sue
on an acceleration clause.
If there is a nonoptional ac-
celeration clause in the con-
tract, the plaintiff must sue
for all installments in her
original lawsuit. (B) is incor-
rect because the two suits
do involve the same cause of
action. They arise out of the
same transaction or occur-
rence, i.e., the lease. (C) is
incorrect because, although
usually a plaintiff may only
sue on the installments due
at the time of the suit, she
must sue for all installments
if there is an acceleration
clause.

83. A defendant was convicted in federal court (B)


of possession of one kilogram of heroin
with intent to distribute. She was sentenced The defendant's motion
to a prison term. Subsequently, the defen- should be denied because
dant was indicted by a federal grand jury for a prosecution for conspira-
80 / 184
Barbri MBE questions
Study online at https://quizlet.com/_6umlok
conspiracy to distribute the same kilogram cy is distinct from a pros-
of heroin. She moved to dismiss the indict- ecution for any substantive
ment. offense involving the same
conduct as the conspiracy.
Should her motion be granted? The Fifth Amendment pro-
vides that no person shall
A No, because the Double Jeopardy Clause be twice put in jeopardy for
does not apply when the second prosecu- the same offense. The gen-
tion is for violation of a separate statute. eral rule is that two crimes
do not constitute the same
B No, because each prosecution requires offense if each crime re-
proof of an element that the other does not. quires proof of an addition-
al element that the other
C Yes, because the Double Jeopardy Clause crime does not require, even
protects her against a second prosecution though some of the same
for the same criminal conduct. facts may be necessary to
prove both crimes. Further-
D Yes, because the Due Process Clause pro- more, a prosecution for con-
tects her against double punishment for the spiracy is not barred merely
same criminal conduct. because some of the alleged
overt acts of that conspira-
cy have already been prose-
cuted. [United States v. Felix
(1992)] Here, both the con-
spiracy charge and the pos-
session charge require proof
of an element that the oth-
er charge does not; hence,
there is no double jeopardy
problem with the indictment.
(A) is incorrect because it is
too broad a statement. The
fact that separate statutes
are involved does not estab-
lish that these are not the
"same offense" for purposes
of double jeopardy. (C) is in-
correct because the "same
conduct" test is not current-
81 / 184
Barbri MBE questions
Study online at https://quizlet.com/_6umlok
ly used by the Supreme
Court to evaluate a double
jeopardy claim. (D) is incor-
rect because the question in-
volves the defendant's mo-
tion to quash an indictment
and not her ultimate punish-
ment.

84. A corporation was in the business of (C)


purchasing real property at below-market
prices and reselling the properties to in- Judgment should be for the
vestors. The bylaws of the corporation au- hotel regardless of whether
thorized the chief executive officer ("CEO") the jurisdiction has a no-
and the director of the marketing division to tice statute or a race-notice
enter into contracts on behalf of the corpo- statute. Under either type of
ration for the purchase or sale of properties. recording statute, the only
The corporation had recently purchased a persons protected by the
large parcel of beachfront property for re- statute are bona fide pur-
sale. The CEO secretly opened negotiations chasers. To attain this status,
with an amusement park to sell the property. the person must take without
However, unknown to the CEO or anyone notice—either actual, con-
else in the corporation, the marketing direc- structive, or inquiry—of the
tor had already reached an agreement with prior instrument. Because
a hotel for the sale of the property. the marketing director-ho-
tel contract was properly
On April 23, the marketing director and the recorded, the amusement
hotel signed a written contract providing for park had constructive no-
sale of the property by the corporation to tice of the hotel's interest in
the hotel for $35 million. On April 25, the the property. Thus, the park
board of directors amended its bylaws, ef- could not become a bona
fectively depriving the marketing director of fide purchaser when it en-
authorization to bind the corporation in pur- tered into its contract. (A) is
chase or sale transactions. This action was wrong because the market-
immediately publicized and became known ing director had not been de-
to both the marketing director and the ho- prived of authority to bind the
tel. On April 26, the hotel duly recorded corporation at the time she
its contract. On May 1, the CEO, still un- signed the agreement with
aware of the marketing director-hotel agree- the hotel, and any subse-
82 / 184
Barbri MBE questions
Study online at https://quizlet.com/_6umlok
ment, approved sale of the property to the quent change in her powers
amusement park for $39 million. The neces- did not affect the validity of
sary documents of title were prepared and that agreement, nor the ho-
properly recorded by the amusement park tel's power to subsequent-
on May 5. Two days later, the amusement ly record the agreement. (B)
park learned of the marketing director-hotel is wrong because the ho-
agreement. On May 10, the date scheduled tel recorded its contract of
for closing of the hotel's sale agreement, sale. Any instrument creat-
the CEO refused to accept the hotel's tender ing or affecting an interest
of $35 million and refused its demand for a in land (e.g., deed, mort-
deed to the property. gage, contract to convey)
can be recorded, providing
The hotel subsequently brings action constructive notice to sub-
against the corporation and the amusement sequent purchasers. Thus,
park for specific performance and to quiet the hotel's failure to record
title to the property. For whom will the court a deed does not deprive it
likely rule? of protection of the record-
ing statute. (D) is wrong be-
A The defendants, because the board of di- cause regardless of the va-
rectors had deprived the marketing director lidity of the board's attempt
of authority to bind the corporation in the to divest the marketing di-
sale of real property. rector of authority, it is im-
material to the hotel's rights;
B The defendants, because the amusement it came after the valid mar-
park is the only purchaser who properly keting director-hotel contract
recorded a deed to the property. was properly signed by the
marketing director.
C The hotel, because the amusement park
had constructive notice of the hotel's inter-
ests in the property when the agreement
with the CEO was made.

D The hotel, because the attempt to divest


the marketing director of authority to ap-
prove sales of the corporation's property
was invalid.

85. The United States was involved in a dispute (C)


with a small island nation over the owner-
83 / 184
Barbri MBE questions
Study online at https://quizlet.com/_6umlok
ship of an archipelago. On discovering that The President's strongest
the archipelago was rich in oil, the Presi- argument is that the power to
dent announced that he would appoint an select ambassadors is vest-
ambassador to negotiate a treaty with the is- ed by the Constitution in the
land nation to jointly exploit the oil reserve. President, and the Senate's
A majority of Senators believed that the is- only power in this respect is
land clearly belonged to the United States to advise and give (or with-
and did not want to negotiate with the island hold) its consent. The Sen-
nation. They passed a resolution requiring ate is not given the power
the President to include a Senator in his to force ambassadors on the
diplomatic mission to ensure that the Sen- President.
ate's view was presented in any negotiation
with the island nation.

What is the strongest constitutional ground


for the President's refusal to do so?

A As commander in chief, the President has


the exclusive power to determine how to
protect our national interest abroad.

B The resolution is unreasonable because it


includes a Senator and not any Representa-
tives.

C The President has the exclusive power


to select diplomatic representatives of the
United States.

D The Senate, if it does not like the Pres-


ident's actions, can refuse to appropriate
the necessary monies for the President to
implement his policies.

86. A driver and his passenger were involved (B)


in an automobile accident when the driver
ran a red light and crashed into another The driver's negligence
car. Due to a manufacturing defect in the would bar recovery if it was
84 / 184
Barbri MBE questions
Study online at https://quizlet.com/_6umlok
automobile's airbag system, the passenger the sole legal cause of the
side airbag did not deploy. The passenger passenger's death. Regard-
was killed on impact. The passenger's es- less of the theory that the
tate brought suit against the driver and the plaintiff is using in a prod-
airbag's manufacturer. At trial it is estab- ucts liability action, actual
lished that the driver was negligent in run- and proximate cause must
ning the red light. be established. If the driver's
negligence is the sole legal
What effect would such proof have on the or proximate cause of the
claim of the passenger's estate against the passenger's death, it would
airbag manufacturer? preclude the estate's suit
against the airbag manufac-
A It would reduce recovery by the estate turer because the defect was
if the action against the manufacturer is not a legal cause of the
based on negligence. passenger's death. (A) is in-
correct because the driver's
B It would bar recovery by the estate if the contributory negligence will
trier of fact finds that the driver was the sole not be imputed to the pas-
legal cause of the passenger's death. senger; hence, it will not
reduce the estate's recov-
C It would bar recovery by the estate if it is ery under comparative negli-
shown that the driver is the sole legal heir gence rules. (C) is incorrect.
of the passenger's estate. A potential beneficiary who
was negligent will be sub-
D It would have no effect on recovery by ject to the jurisdiction's fault
the estate as long as the action against the rules. Hence, his negligence
manufacturer is based on strict liability. will reduce his recovery un-
der pure comparative negli-
gence rules but will not bar
it, even if he is the sole heir.
(D) is incorrect. In most pure
comparative negligence ju-
risdictions, the same com-
parative fault rules will apply
whether the action against
the manufacturer is based
on negligence or strict lia-
bility. As discussed above,
whether the estate can re-
85 / 184
Barbri MBE questions
Study online at https://quizlet.com/_6umlok
cover, and the extent of its
recovery, depends on cau-
sation issues and the dri-
ver's status as a beneficiary
of the estate.

87. An antique lover spotted a beautiful Early (D)


American bedroom ensemble at her favorite
antique store. The ensemble included a bed, Although the crossing offers
a mirror, and two dressers. Over a period as to price were identical,
of several weeks, the shop owner and the there is no requisite mutu-
antique lover negotiated over a price, but al assent absent an accep-
they were unable to come to an agreement. tance. If offers stating pre-
cisely the same terms cross
On April 3, the shop owner and the antique in the mail, they do not
lover signed a statement whereby the shop give rise to a contract de-
owner offered to sell to the antique lover an spite the apparent meeting
Early American bedroom ensemble, record- of the minds. An offer can-
ed as items 20465, 20466, 20467, and 20468 not be accepted if there is
in the shop's registry, if the parties agree no knowledge of it. Here,
upon a price on or before April 12. the shop owner and the an-
tique lover each sent of-
On April 6, the shop owner sent a letter to fers setting the price of the
the antique lover, telling her that she could ensemble at $22,000. De-
have the bedroom ensemble for $22,000. spite the fact that these of-
Also on April 6, the antique lover sent a fers were identical, there is
letter to the shop owner telling him that she no mutual assent without at
was willing to pay him $22,000 for the bed- least one of the parties man-
room ensemble. Both parties received their ifesting acceptance of the
letters on April 7. terms of the offer, and com-
municating that acceptance
Without assuming any additional facts, to the other. We are told
which of the following statements is most that this has not yet hap-
correct as of April 8? pened even though the shop
owner and the antique lover
both have received the let-
A The shop owner and the antique lover had ters. Consequently, although
a valid contract from the moment the letters there is an apparent meet-
of April 6 were mailed. ing of the minds as to price,
86 / 184
Barbri MBE questions
Study online at https://quizlet.com/_6umlok
there has not been a suffi-
B A contract exists between the shop own- cient objective manifestation
er and the antique lover, because the shop of this agreement as to de-
owner, a merchant, sent the antique lover an note a mutual assent. (C)
offer in writing. fails to account for the prin-
ciple discussed above, that
C A contract exists between the shop owner identical crossing offers do
and the antique lover, because the crossing not give rise to a contract.
offers were identical and received before Despite their receipt of iden-
April 12. tical offers before April 12,
there is no agreement be-
D No contract exists between the shop own- tween the parties. (A) is in-
er and the antique lover, because of a lack of correct because it misstates
mutual assent. the mailbox rule. Acceptance
by mail or similar means cre-
ates a contract at the mo-
ment of posting, properly ad-
dressed and stamped, un-
less the offer stipulates that
acceptance is not effective
until received, or unless an
option contract is involved.
This rule does not operate
to create a contract from the
moment an offer is mailed
(or in this case, two identi-
cal offers are mailed). Thus,
(A) is incorrect. Regarding
(B), the fact that a merchant
sends an offer in writing is
significant because it will lim-
it the offeror's power to re-
voke if it gives assurances
that it will be held open for
a stated time. Here, the writ-
ten offer by the shop own-
er is irrevocable at least until
April 12, but the issue in the
question is whether it has
87 / 184
Barbri MBE questions
Study online at https://quizlet.com/_6umlok
been accepted rather than
whether it has been revoked.

88. An uncle's will devised his lakefront es- (B)


tate "to my butler for life, remainder to
my niece." The 40-acre estate includes a The niece's best argument
mansion, a 20-acre orchard, a beach, and is that destruction of the
gardens. At the time of the uncle's death, residence constitutes waste.
the butler was 40 years old and of modest The other choices do not
means. The niece was 18 years old and quite present arguments giving
wealthy. The estate was encumbered by a her a chance of success. A
mortgage that was not entitled to exonera- life tenant is entitled to all or-
tion. After the first year, the butler could no
dinary uses and profits of the
longer make the mortgage payments, so the land, but he cannot lawfully
niece paid them. do any act that would injure
the interests of the remain-
Ten years after the uncle's death, the town derman. A grantor intends
in which the estate was located became a that the life tenant have the
hot resort area. A major resort chain ap- general use of the land in a
proached the butler with a multimillion-dol- reasonable manner, but that
lar offer for the easternmost 20 acres of the the land pass to the owner
estate, which included the residence and of the remainder, as near-
beach. The resort chain planned to raze the ly as practicable, unimpaired
mansion to erect a high-rise hotel. The but- in its nature, character, and
ler approached the niece about the offer. improvements. Even amelio-
He proposed to give her most of the mon- rative waste, which actually
ey from the sale and offered to build any increases the value of the
house she desired on the remaining land. land, is actionable if there
The niece refused to go along with the plan. is no reasonable justification
The butler decided to proceed with the sale, for the change. A life ten-
and the niece brought a suit to enjoin the ant can substantially alter or
butler's proposed actions. even demolish existing build-
ings if (i) the market val-
Which of the following is the niece's best ue of the future interests is
argument? not diminished and either (ii)
the remainderman does not
A The eventual use of the property by the object, or (iii) a substantial
remainderman will be as a residence. and permanent change in
the neighborhood conditions
88 / 184
Barbri MBE questions
Study online at https://quizlet.com/_6umlok
B Destruction of the mansion constitutes has deprived the property in
waste. its current form of reason-
able productivity or useful-
C Because the niece paid the mortgage pay- ness. Here, the market val-
ments, the butler is subrogated to her rights. ue of the property would not
be diminished. The remain-
D The butler has no right to transfer his life derman (the niece), howev-
estate. er, is objecting, making op-
tion (ii) unavailable. Further-
more, although the neigh-
boring properties have been
sold for hotels and resorts,
it does not necessarily fol-
low that the conditions have
changed to such a de-
gree that the estate should
be similarly converted. The
property is large enough to
be somewhat isolated from
the changes in the surround-
ing areas; thus, despite the
surrounding hotels, an own-
er could still enjoy the land
as a private residence, or-
chard, and beach. Therefore,
the property is still useful
and option (iii) is also un-
available. In this case, the
life tenant's desire to raze
the mansion is not because
the changes in the neighbor-
hood have made the man-
sion uneconomical or im-
practical. The life tenant can
make more money by tear-
ing the mansion down, but
its usefulness and value are
apparently unaffected by the
changes in the neighbor-
89 / 184
Barbri MBE questions
Study online at https://quizlet.com/_6umlok
hood. Thus, the niece will be
able to enjoin the butler from
allowing the resort chain to
raze the mansion and build a
hotel. (A) is wrong because
the fact that the niece in-
tended to use the property
as a residence is irrelevant.
Even if the niece intended to
change the use of the prop-
erty, she is still entitled to re-
ceive the land in the condi-
tion in which it passed to the
butler.

89. A plaintiff sued a defendant for defamation, (A)


asserting in her complaint that the defen-
dant had called the plaintiff a thief in front of The defense witness's tes-
a number of business associates. The plain- timony is admissible char-
tiff calls two witnesses to the stand, both of acter evidence because the
whom testify that they heard the defendant plaintiff's character is direct-
refer to the plaintiff as a thief in front of the ly in issue in the case. As
business associates. The plaintiff does not a general rule, evidence of
take the stand herself. The defendant pleads character to prove the con-
truth of the statement as an affirmative de- duct of a person in the litigat-
fense and calls a witness to the stand. The ed event is not admissible in
defense witness is prepared to testify that a civil case. However, when
he was a co-worker of the plaintiff when the proof of a person's charac-
plaintiff supplemented her income by tend- ter, as a matter of substan-
ing bar three nights a week. The witness will tive law, is an essential ele-
testify that he saw the plaintiff take a $20 bill ment of a claim or defense
from the tavern's cash register and secrete in a civil action, character
the money in her pocket. The plaintiff's at- evidence is admissible be-
torney objects. cause it is the best method
of proving the issue. Under
May the defense witness's testimony be al- the Federal Rules, any of
lowed? the types of evidence—rep-
utation, opinion, or specific
acts—may be used. Here,
90 / 184
Barbri MBE questions
Study online at https://quizlet.com/_6umlok
A Yes, as substantive evidence that the character is an issue in the
plaintiff is, in fact, a thief. plaintiff's defamation action
because the defendant has
B Yes, because theft is a crime indicating pleaded as an affirmative
dishonesty. defense that his statement
claiming that the plaintiff is a
C No, because specific bad acts may not be thief is the truth. The defense
used to show bad character. witness's testimony that he
saw the plaintiff take the
D No, because the plaintiff never took the money from the cash reg-
stand. ister is relevant because it
tends to show that the defen-
dant spoke the truth. Hence,
it should be allowed. (B) is
incorrect because the fact
that the theft here could be
considered a crime of dis-
honesty would be relevant
only if the plaintiff's credibil-
ity were being impeached,
and only then if proof of an
actual conviction were pro-
vided. Here, the testimony
is admissible because it is
being offered as substantive
evidence of an aspect of the
plaintiff's character that is an
essential element of a de-
fense in the case. (C) is in-
correct. One of the few cases
where testimony as to spe-
cific acts of a person may be
used to show that person's
character is when charac-
ter itself is one of the es-
sential issues in the case,
as it is here. (D) is incor-
rect because the fact that the
plaintiff never took the stand
91 / 184
Barbri MBE questions
Study online at https://quizlet.com/_6umlok
only means that she has not
placed her credibility in is-
sue and become subject to
impeachment. Here, howev-
er, the plaintiff's character is
in issue and the testimony
is being offered as substan-
tive evidence of her charac-
ter rather than to impeach
her credibility.

90. A husband and a wife were arrested by (D)


federal agents and charged with distribut-
ing obscene materials through the United The grand jury transcript is
States mails. When called before a grand not admissible because the
jury, the wife refused to say anything, invok-
husband's testimony was
ing her Fifth Amendment right to be protect-
not subject to cross-exami-
ed from compelled self-incrimination. The nation. The husband's testi-
husband was terrified of the grand jury and
mony was hearsay because
readily admitted under questioning that heit was an out-of-court state-
sent obscene matter through the mail. He ment offered to prove the
also incriminated his wife in the illegal activ-
truth of the matter asserted.
ity. The thought of a trial and a prison term
Under the former testimo-
drove the husband over the edge, and he ny exception to the hearsay
committed suicide two days before his trial
rule, the testimony of a now
was to begin. A month later, the wife was put
unavailable witness given at
on trial in federal district court. The federal
another hearing is admis-
prosecutor seeks to introduce a transcriptsible in a subsequent tri-
of the husband's grand jury testimony intoal as long as there is a
evidence against the wife. The defense at-sufficient similarity of par-
torney objects. ties and issues so that the
opportunity to develop tes-
How should the court rule on the admissi- timony or cross-examine at
bility of the grand jury transcript? the prior hearing was mean-
ingful. Thus, the grand jury
A Admissible, as a vicarious admission. testimony of an unavailable
declarant is not admissible
B Admissible, as former testimony. as former testimony against
the accused at trial. This
92 / 184
Barbri MBE questions
Study online at https://quizlet.com/_6umlok
C Inadmissible, because the wife can invoke is because grand jury pro-
the testimonial privilege, even though her ceedings do not provide the
husband is now deceased. opportunity for cross-exam-
ination. (A) is incorrect be-
D Inadmissible, because the husband's tes- cause the husband's tes-
timony was not subject to cross-examina- timony cannot be consid-
tion. ered a vicarious admission.
A statement by an oppos-
ing party is not hearsay. An
admission is a statement
made by a party and of-
fered against that party. An
admission does not have to
be the statement of the par-
ty against whom the state-
ment is being offered at tri-
al if it qualifies as a vicar-
ious admission. Here, how-
ever, the husband's grand
jury testimony was not made
in furtherance of a conspir-
acy. Because he was not
a party here, and his tes-
timony does not otherwise
qualify as a vicarious admis-
sion of the wife, it cannot
be considered an admission
of a party-opponent. (B) is
incorrect because the hus-
band's grand jury testimony
was not subject to cross-ex-
amination. The husband was
an unavailable declarant be-
cause he was unable to tes-
tify because of death. How-
ever, as discussed above,
his grand jury testimony is
not admissible as former tes-
timony because grand jury
93 / 184
Barbri MBE questions
Study online at https://quizlet.com/_6umlok
proceedings do not provide
the opportunity for cross-ex-
amination.

91. A foreign student who had entered the Unit- (C)


ed States on a student visa four years ago
was notified by federal immigration author- The court should find
ities that he was subject to being deported the resolution invalid. While
because his visa had expired. Federal law Congress has broad pow-
provided that an alien who is subject to be- er to delegate, the separa-
ing deported has the right to appear before tion of powers doctrine for-
an administrative officer appointed by the bids Congress from trying
Attorney General's office for a hearing on to control the exercise of
whether he should be deported. This offi- the power delegated in var-
cer, appointed by the executive branch of ious ways, such as by over-
the government, has the right under law to turning an executive agency
make a final order concerning whether the action without bicameralism
alien should be deported. After a hearing, (i.e., passage by both hous-
the administrative officer entered an order es of Congress). By enact-
allowing the student to remain in the United ing the federal law allowing
States as a permanent resident. the administrative law judge
to enter a final order with re-
However, a congressional rule permitted the gard to aliens, Congress has
House of Representatives, by resolution, to given up any control it may
deport "undesirable aliens." After the ad- have had previously in these
ministrative judge entered his order, the situations. The resolution by
House passed a resolution that the student the House here is an un-
should be deported. The student petitioned constitutional legislative veto
the federal court to declare the legislative that violates the separation
resolution invalid. of powers doctrine. (A) is in-
correct because, while Con-
Should the court find the resolution to be gress does have plenary
valid? power over aliens with re-
gard to immigration and nat-
uralization, here it has giv-
A Yes, because Congress has plenary pow- en up control over this area
ers with regard to aliens and naturalization. by enacting a law allow-
ing an administrative officer
B Yes, because aliens are not "citizens" appointed by the executive
94 / 184
Barbri MBE questions
Study online at https://quizlet.com/_6umlok
within the meaning of the Fourteenth branch to make a final or-
Amendment. der concerning whether an
alien should be deported. (B)
C No, because the federal law removed con- is incorrect because the fact
gressional power with regard to aliens in that aliens are not citizens
this circumstance, and the resolution of the has no bearing on whether
House violates the separation of powers the House resolution violat-
doctrine. ed the Constitution. (D) is in-
correct because, while resi-
D No, because the student was denied due dent aliens are entitled to no-
process when he was not given a hearing tice and hearing before they
before the House of Representatives. can be deported, the student
did receive a hearing be-
fore the administrative offi-
cer. There is no requirement
that persons affected by leg-
islative action have the right
to be heard by the legisla-
tive body taking the action.
Thus, the better argument as
to why the resolution was in-
valid is based on separation
of powers.

92. A plaintiff is suing a defendant in federal (A)


court for personal injuries arising out of
an automobile accident. The defendant's au- The defendant must disclose
tomobile insurance policy covers the acci- the existence of the insur-
dent. ance policy under the Fed-
eral Rules, which express-
Must the defendant disclose the existence ly permit discovery of in-
of the insurance policy? surance agreements as an
initial disclosure. (B) is in-
A Yes, because it is required under the Fed- correct. There is no need
eral Rules. to show that the discovery
of insurance coverage would
B Yes, unless the discovery of the insurance lead to other admissible evi-
coverage would not lead to other discover- dence. (C) is incorrect. Initial
able evidence. disclosures must be made
95 / 184
Barbri MBE questions
Study online at https://quizlet.com/_6umlok
regardless of whether the
C No, unless the plaintiff submits an inter- opposing party submits a re-
rogatory. quest for the information. (D)
is incorrect. Even though in-
D No, because insurance coverage cannot surance coverage is not rel-
be mentioned at trial. evant to the case, insurance
coverage is nonetheless dis-
coverable.

93. The defendant planned to break into a (C)


home, steal any valuables that he could eas-
ily pawn, and then burn down the home us- The defendant has commit-
ing gasoline from his lawnmower. When the ted burglary and larceny.
defendant got to the home that night, he re- Burglary consists of a break-
alized that he had forgotten the gas at home. ing and entry of the dwelling
Nonetheless, the defendant broke into the of another at nighttime, with
home through a basement window. Unbe- the intent of committing a
knownst to him, the police were alerted by a felony therein. The felony
silent alarm and arrested the defendant just need not be carried out—all
as he was leaving the home with a sack filled that is required is that the
with valuables. person committing the crime
have the intent to commit
At common law, what crimes has the defen- a felony at the time of en-
dant committed? try. At night, the defendant
broke into and entered the
A Burglary and attempted larceny. house with the intent to com-
mit the felony of larceny. Re-
B Burglary, attempted larceny, and attempt- gardless of whether the de-
ed arson. fendant took any property
or committed a burning, the
C Burglary and larceny. burglary was complete on
his breaking and entering
D Burglary, larceny, and attempted arson. the dwelling at nighttime with
the requisite intent. The de-
fendant has also committed
common law larceny. Larce-
ny is the taking and car-
rying away of the personal
property of another, by tres-
96 / 184
Barbri MBE questions
Study online at https://quizlet.com/_6umlok
pass, with the intent to per-
manently deprive the owner
of his interest in the prop-
erty. The element of carry-
ing away, or asportation, is
satisfied as long as there is
some movement of the prop-
erty as a step in carrying it
away. The movement need
only be slight as long as
it was part of the carrying
away process. Here, the de-
fendant placed valuables in
a sack and started to leave
the home. This movement
was sufficient to constitute
a carrying away. Having act-
ed with the requisite intent
to permanently deprive the
true owner of his property,
the defendant has commit-
ted larceny.

94. A locksmith knew that his friend had been (C)


having marital troubles. The friend had told
the locksmith that he suspected his wife The locksmith cannot be
was having an affair with his rival. One af- convicted as an accomplice
ternoon, the friend, visibly upset, asked to because he did not have the
borrow some of the locksmith's tools, telling requisite intent for attempt-
him that he knew that his rival was going ed murder. To be convict-
to meet up with his wife later that day. The ed as an accomplice under
locksmith gave his friend the tools, advising the prevailing rule, a person
him not to do anything that he would regret must have given aid, coun-
later. The friend stated that it would be oth- sel, or encouragement with
ers who would have regrets. The friend went the intent to aid or encour-
to his rival's apartment and picked the door age the principal and the in-
lock with the locksmith's tools. He found tent that the principal com-
his wife and rival in bed together. The friend mit the substantive offense.
stabbed his rival, seriously wounding him. Mere knowledge that a crime
97 / 184
Barbri MBE questions
Study online at https://quizlet.com/_6umlok
A few minutes later the locksmith called would result from the aid pro-
the apartment to try to warn the rival that vided is generally insufficient
his friend might come over. After the friend for accomplice liability. Here,
was arrested, he agreed to plead guilty to the locksmith did not pro-
aggravated battery and attempted volun- vide the tools to the friend
tary manslaughter in exchange for testify- with the intent that he kill
ing against the locksmith, who was charged the rival. His knowledge that
as an accomplice to attempted murder. the friend might be intending
harm to the rival is not suf-
Can the locksmith be convicted of that ficient to establish the intent
charge? to kill required for attempted
murder. (A) is incorrect be-
A Yes, because he recklessly disregarded a cause even if the locksmith's
substantial risk to human life and was not conduct constituted reckless
provoked. disregard of high risk to hu-
man life, that state of mind
B Yes, because his failed attempt to neu- is not sufficient for attempt-
tralize his assistance did not prevent the ed murder. Unlike murder, at-
crime from occurring and therefore did not tempted murder is a specific
constitute an adequate withdrawal. intent crime and requires the
intent to kill. (B) is incorrect.
C No, because he did not have the requisite Although the locksmith's at-
intent to be liable as an accomplice. tempt to neutralize his assis-
tance would not have been
D No, because an accomplice cannot be enough to raise the defense
found guilty of a more serious offense than of withdrawal if he had in-
that for which the principal has been con- curred liability as an accom-
victed. plice, here he did not have
the requisite intent for ac-
complice liability. (D) is an in-
correct statement of law; the
degree of liability of a princi-
pal is irrelevant to the poten-
tial liability of an accomplice.
If the locksmith had had the
intent to aid his friend in
killing the rival, the fact that
the friend could show ade-
quate provocation to reduce
98 / 184
Barbri MBE questions
Study online at https://quizlet.com/_6umlok
his offense to attempted vol-
untary manslaughter would
have no effect on the lock-
smith's liability for attempted
murder.

95. A man from a foreign country obtained a (D)


doctorate in political science from a state
university and applied to teach there. The A state generally may not
man was denied employment at the univer- discriminate against aliens
sity under a state law requiring all teachers absent a compelling state
within the state to be United States citizens. interest, and no compelling
interest is served by pro-
Is the state's citizenship requirement con- hibiting aliens from teach-
stitutional as it applies to the man? ing at a state university.
(A) is incorrect. The Tenth
A Yes, because states have the right to set Amendment reserves to the
minimal standards for state employees un- states power not granted to
der the Tenth Amendment. the federal government. The
Constitution vests the pow-
B Yes, because a university political science er to regulate aliens in Con-
teacher would exert a great deal of influence gress, and thus the states
over the attitudes of students toward gov- do not have power to con-
ernment, the political process, and citizen- trol aliens under the Tenth
ship. Amendment. (B) is incorrect
because it states the stan-
C No, because the citizenship requirement dard that the Supreme Court
is not rationally related to a legitimate state has applied to primary and
interest. secondary school teachers.
The Supreme Court has up-
D No, because the citizenship requirement held state statutes prohibit-
is not necessary to achieve a compelling ing aliens from teaching pri-
state interest. mary or secondary school
on the rationale that teach-
ers at the elementary and
high school level have a
great deal of influence over
the attitudes of young stu-
dents toward government,
99 / 184
Barbri MBE questions
Study online at https://quizlet.com/_6umlok
the political process, and cit-
izenship. It is doubtful that
the Court would extend this
rationale to university teach-
ers. (C) is incorrect because
it states the wrong stan-
dard. If state discrimination
against aliens relates to par-
ticipation of aliens in the
functioning of state govern-
ment, the rational basis test
applies. Merely teaching po-
litical science at a state uni-
versity is not equivalent to
participating in the political
process.

96. A grandfather told his granddaughter that (C)


she could have his house because he was
moving to a retirement home, and entered The builder will prevail be-
into a valid contract to convey it to her. He cause he may raise all de-
promised her that he would have another fenses that he had against
wing added to the house in the back before the grandfather against the
turning it over to her, and entered into a granddaughter. The grand-
written contract with a builder to construct daughter is an intended
the addition for his granddaughter. Before third-party beneficiary of the
the grandfather had entered into the con- contract between the grand-
tract with the builder, the granddaughter father and the builder. Gen-
had paid $5,000 for a 60-day option to pur- erally, a third-party benefi-
chase another house because she was not ciary has rights under the
sure she would like the addition. However, contract as soon as she
when her grandfather showed her the plans does something to vest her
for his house prepared by the builder, she rights (manifests assent to
liked it very much and decided to let her the promise, brings suit to
option to purchase the other house lapse. enforce the promise, or ma-
Shortly thereafter, the local zoning authority terially changes position by
increased the minimum lot line setbacks, justifiably relying on the
making it impracticable to put the addition promise). Here, the grand-
on the back of the house. The builder offered daughter materially changed
100 / 184
Barbri MBE questions
Study online at https://quizlet.com/_6umlok
to put an addition above the existing floor her position by justifiably al-
rather than in the back, and the grandfa- lowing her option on the oth-
ther agreed. After the granddaughter's op- er house to lapse. General-
tion had lapsed, she discovered that the ad- ly, once the third-party ben-
dition was now going up rather than in the eficiary's rights have vest-
back. She angrily demanded that the builder ed, the original contracting
either build the addition according to the parties may not modify the
original specifications that she approved or contract without the assent
pay her damages. The builder refused and of the third-party beneficia-
the granddaughter filed suit. ry. However, the third-party
beneficiary is subject to any
Who is more likely to prevail? defenses that the promisor
could have used against the
A The granddaughter, because she was an original promisee, and here
intended beneficiary of the contract whose the builder could have used
rights had vested. the defense of impractica-
bility against the promisee.
B The granddaughter, because the subse- Therefore, he could use that
quent agreement between her grandfather defense against the grand-
and the builder to modify the construction daughter to avoid having to
was unsupported by consideration. pay damages for not build-
ing the house as he orig-
C The builder, because he may raise all de- inally agreed. (A) is incor-
fenses that he had against the grandfather rect because although it is
against the granddaughter. true that the granddaugh-
ter's rights had vested, the
D The builder, because the granddaughter answer fails to take into ac-
is merely an incidental beneficiary of the count the defenses available
contract between the grandfather and the to the builder.
builder and, as such, has no power to en-
force the contract against the builder.

97. A brother and a sister held record title to a (A)


home as joint tenants with right of survivor-
ship. The brother moved out of the home The friend and the daugh-
shortly after conveying his interest in the ter own the home as co-ten-
home to his friend by quitclaim deed. The ants. A joint tenancy is an
friend did not record his deed. Several years estate between two or more
later, the sister died, leaving her adopted co-tenants who have a right
101 / 184
Barbri MBE questions
Study online at https://quizlet.com/_6umlok
daughter as her sole heir. Shortly after the of survivorship—when one
sister died, the brother asked his friend to joint tenant dies, the prop-
return his deed and give up his interest in erty is freed from her con-
the home. The friend agreed and returned current interest and the sur-
the deed, which the brother destroyed. vivor or survivors retain an
undivided right in the proper-
Who has title to the home? ty. An inter vivos conveyance
by one joint tenant of his
A The friend and the daughter as co-tenants. undivided interest destroys
the joint tenancy so that the
B The brother and the daughter as co-ten- transferee takes the interest
ants. as a tenant in common and
not as a joint tenant. Here,
C The brother as sole owner. when the brother conveyed
his interest to the friend, the
D The friend as sole owner. joint tenancy between the
brother and the sister was
severed. At that point, the
friend and the sister held
title to the home as ten-
ants in common. The adopt-
ed daughter then inherited
the sister's interest upon the
sister's death. Because de-
livery of a deed cannot be
canceled, the friend's return
and subsequent destruction
of his deed has no effect.

98. An elderly woman filed a complaint in feder- (C)


al district court, alleging that a salesperson
"made fraudulent statements that induced The court should grant the
the plaintiff to sign the contract to the plain- motion to dismiss for failure
tiff's detriment." The salesperson then filed to state a claim upon which
a motion to dismiss the complaint for failure relief can be granted. The
to state a claim upon which relief can be federal pleading rules gen-
granted. erally require the pleader to
assert short and plain state-
How should the court rule? ments in the complaint to put
102 / 184
Barbri MBE questions
Study online at https://quizlet.com/_6umlok
the other side on notice of
the claim being asserted; de-
A Deny the motion, because the complaint tailed assertions of facts un-
stated a possible claim for relief. derlying the claim are gen-
erally not required. Howev-
B Deny the motion, because the complaint er, there are certain special
stated a plausible claim for relief. pleading rules that require a
party to state more detail un-
C Grant the motion, because the complaint der special circumstances,
did not state the claim for relief with partic- including claims that assert
ularity. fraud or mistake. Under such
special circumstances the
D Grant the motion, because the complaint federal rules specifically re-
did not state the claim for relief with proba- quire that a plaintiff assert
bility. the claim for relief with par-
ticularity. Here, choice (C) is
the best answer because it
is the only choice to correct-
ly state the rule. The sales-
person's motion to dismiss
should be granted in this
case because the plaintiff's
complaint fails to state the
claim with particularity.

99. A local entertainment section of a newspa- (A)


per published a story on the town's busi-
ness district, accompanied by photos of The minister likely will pre-
various businesses in the district. A min- vail because unauthorized
ister who happened to be walking on the use of his picture that false-
sidewalk in front of an adult bookstore when ly makes him appear to be
a photo was taken for the story became very exiting the adult bookstore
upset when he saw it in the newspaper, be- would be highly offensive to
cause the camera angle made it appear that a reasonable person under
he was exiting the bookstore. the circumstances and con-
stitute a false light invasion
If the minister sues the newspaper for inva- of privacy. To establish a pri-
sion of privacy and establishes the above ma facie case for invasion of
facts, is he likely to prevail? privacy based on publication
103 / 184
Barbri MBE questions
Study online at https://quizlet.com/_6umlok
by defendant of facts plac-
A Yes, because the photo made it appear as ing plaintiff in a false light,
if he was exiting an adult bookstore. the following elements must
be proved: (i) publication of
B Yes, because the newspaper made a pub- facts about plaintiff by de-
lic disclosure of a private fact. fendant placing plaintiff in a
false light in the public eye;
C No, because he was on a public sidewalk and (ii) the "false light" must
when the photo was taken. be something that would be
highly offensive to a reason-
D No, because he has not alleged any eco- able person under the cir-
nomic or pecuniary damages. cumstances. Here, the photo
created the false impression
that the minister was exiting
an adult bookstore. Publica-
tion of the photo conveying
this false impression of the
minister's conduct would be
highly offensive to a reason-
able person under the cir-
cumstances.

100. A student started a small fire in a trash (C)


can in the men's room at his university. His
plan was to set off the school's fire alarms The student is guilty of the
so that he could break into the computer crimes of arson and at-
lab and steal a laptop computer while the tempted larceny. At com-
building was being evacuated. The student mon law, arson was defined
was stopped after he had set the fire and as the malicious burning of
was attempting to smash in the glass on the dwelling of another. The
the computer lab door. The fire was quickly mens rea required for arson
extinguished and no serious damage was is malice, which is broader
done to the building by the fire beyond some than the intent required for
charring on the walls in one stall of the specific intent crimes. All that
men's room. A statute in the jurisdiction ex- malice requires is that the
tends the crime of arson to buildings other defendant have acted with
than dwellings. the intent or knowledge that
the structure would burn, or
Which of the following best describes the with reckless disregard of an
104 / 184
Barbri MBE questions
Study online at https://quizlet.com/_6umlok
crimes of which the student could be prop- obvious risk that the struc-
erly convicted? ture would burn. Here, the
student's intent was to set
A Larceny only. off the fire alarm so that the
building would be evacuated.
B Larceny and attempted arson. Nevertheless, he intended to
start a fire with reckless dis-
C Attempted larceny and arson. regard of a high risk that it
would cause damage to the
D Arson only. building. The risk or hazard
is not that the building will
burn down, merely that dam-
age to the structure from a
burning will occur. The "burn-
ing" required for arson does
not require significant dam-
age to the building; a char-
ring of the combustible ma-
terial is sufficient. Here, the
wall of a stall in the men's
room was charred. This sat-
isfies the "burning" require-
ment. The common law re-
quirement that the struc-
ture be a dwelling has been
broadened by the statute in
this question to include oth-
er buildings. Thus, the stu-
dent's conduct satisfies all
of the elements of the crime
of arson. The student is also
guilty of attempted larce-
ny. Common law larceny re-
quires a taking and carrying
away of the personal prop-
erty of another by trespass
with intent to permanently
(or for an unreasonable time)
deprive the other of his inter-
105 / 184
Barbri MBE questions
Study online at https://quizlet.com/_6umlok
est in the property. Here, the
student planned to take and
carry away a laptop comput-
er by trespass (i.e., without
permission) and with the in-
tent to permanently deprive
the university of it. However,
he cannot be convicted of a
completed larceny because
he never actually took the
laptop.

101. A state law provides that all persons who (D)


have been residents of the state for more
than three years shall be entitled to free tu- There is a live controversy
ition at the state's main university. It further and the case is not moot. A
provides that persons who have resided in federal court will not hear a
the state for three years or less shall pay case unless there is a real,
the nonresident tuition rate, which is sig- live controversy at all stages
nificantly higher. A student at the state's of the proceeding, not mere-
university who had been a state resident for ly when the case is filed.
less than three years filed a class action Because the student is no
in federal court on behalf of himself and longer required to pay non-
other similarly situated university students, resident tuition, there is ar-
seeking a declaration that the state statute guably no controversy and
is unconstitutional. When the case came to the case may seem moot.
trial, the student had been a resident of the However, a class action is
state for more than three years and was no not moot, and the class rep-
longer required to pay tuition. By that time, resentative may continue to
a number of amicus curiae briefs had been pursue it—even if the rep-
filed in the case, some supporting and some resentative's own controver-
opposing the student's position. Neverthe- sy has become moot—be-
less, the state moved to dismiss the case as cause the claims of oth-
moot. ers in the class are still vi-
able. Here, the student filed
Should the state's motion to dismiss be his suit as a class action
granted? for university students with
less than three years' resi-
A Yes, because the student is now a dency; undoubtedly some of
106 / 184
Barbri MBE questions
Study online at https://quizlet.com/_6umlok
three-year resident. those students will still have
a real controversy at this
B Yes, because the student lacks standing. time. Thus, the case is not
moot. (A) is wrong although
C No, because amicus curiae briefs have it states a true fact. (A) im-
been filed. plies that the case should be
dismissed because the stu-
D No, because there is a live controversy. dent's claim is moot. As ex-
plained above, this is a class
action and other members
of the class have a viable
case. Thus, even though the
named student's case by it-
self would be moot, he may
continue the case as a repre-
sentative of the class action.

102. A manufacturer of high-speed computers (B)


entered into a written agreement with a dis-
tributor whereby the distributor would pur-By oral agreement, the writ-
chase a specified computer from the man- ten agreement between the
ufacturer for $50,000. The parties had oral-
manufacturer and the dis-
ly agreed that the delivery date would be tributor was not to take ef-
November 4. However, when the agreement fect unless the distributor
was reduced to writing, a glitch in the word
notified the manufacturer in
processor caused the printout to show the writing by October 7 that
delivery date as "12/4" instead of "11/4." it (the distributor) had ob-
Both parties signed the paper without notic-
tained a buyer. The distrib-
ing the incorrect delivery date. Before re-utor did not provide notice
ducing their agreement to writing, the par-that it had found a buyer
ties had also orally agreed that the agree-until October 30. General-
ment would not become binding unless the ly, under the parol evidence
distributor notified the manufacturer, in writ-
rule, when the parties ex-
ing, by October 7, that it (the distributor) had
press their agreement in a
obtained a buyer for the computer. writing with the intent that
it embody the final expres-
On September 25, the distributor found a sion of their bargain, any oth-
buyer who needed the computer for her er expressions—written or
business and who agreed to buy it from the oral—made prior to the writ-
107 / 184
Barbri MBE questions
Study online at https://quizlet.com/_6umlok
distributor. However, the distributor did not ing, as well as any oral ex-
inform the manufacturer that it had found a pressions contemporaneous
buyer until October 30. with the writing, are inad-
missible to vary the terms
In the meantime, due to a strike at the man- of the writing. However, if
ufacturer's leading competitor, the price of a party asserts that there
high-speed computers rose rapidly during was an oral agreement that
the month of October. By the end of the the written contract would
month, the market value of the computer not become effective until a
in question was $70,000. Because of the condition occurred, all ev-
increase in the value of the computer, the idence of the understand-
manufacturer does not want to deliver the ing may be offered and re-
specified computer to the distributor for ceived. The rationale is that
$50,000. the written agreement is not
being altered by parol evi-
Which of the following provides the manu- dence because the written
facturer the best defense if the distributor agreement never came into
sues to enforce the contract? being. Here, the notification
by October 7 was a condition
A The increase in value of the computer precedent to the effective-
makes the contract unconscionable. ness of the contract. Thus,
evidence of the oral agree-
B There has been a failure of a condition ment is admissible. Since
precedent. the contract never became
binding, the manufacturer
C A case of mutual mistake exists because has no obligation to deliv-
of the word processing error regarding the er the computer for $50,000
delivery date. or otherwise. (D) is incorrect
because the condition that
D There has been a failure of a condition failed was precedent rather
subsequent. than subsequent. The con-
dition was both precedent
to the agreement itself and
phrased in condition prece-
dent terms. A condition sub-
sequent is one the occur-
rence of which cuts off an al-
ready existing absolute duty
of performance. (A) is incor-
108 / 184
Barbri MBE questions
Study online at https://quizlet.com/_6umlok
rect because contractual du-
ties will be discharged by un-
conscionability only if, at the
time the contract was made,
it was one-sided and terri-
bly unfair to one of the par-
ties. Here, at the time the
contract was made, nothing
about it seemed unfair, and
the subsequent increase in
price does not render the
contract unconscionable. (C)
is incorrect because the mis-
take regarding the delivery
date is a mere clerical er-
ror, rather than a mistake of
fact going to a point that is
material to the transaction.
As such, this is not the type
of mistake that would relieve
either or both of the parties
of their obligations under the
contract.

103. A homeowner hired a contractor to make (D)


some improvements on his house. They en-
tered into a written contract providing that The daughter's best argu-
the contractor would do the improvements ment to enforce the contract
for $5,000. Shortly after the contract was in her favor is that she mar-
signed, the contractor told the homeown- ried in reliance on the con-
er to give the money to his (the contrac- tract (detrimental reliance),
tor's) daughter when the job was finished, although she will proba-
adding, "She is getting married soon and bly be unsuccessful. Here,
I want her to have a nice wedding present the daughter was a gratu-
from me." The daughter was aware that her itous assignee (because she
father made this statement to the home- gave no consideration) and
owner. She married, but soon thereafter the her rights under the con-
contractor told the homeowner to pay him tract were revoked. Thus, her
the $5,000, and not the daughter, because strongest argument will be
109 / 184
Barbri MBE questions
Study online at https://quizlet.com/_6umlok
his son-in-law had a gambling problem and one that nullifies the revo-
would probably use the money to bet at the cation. Under the doctrine
racetrack. of detrimental reliance, a
promise will be enforced to
What is the best argument in favor of the the extent necessary to pre-
daughter's being able to enforce a contract vent injustice if it was made
for $5,000 in her favor? with a reasonable expec-
tation that it would induce
A Statute of Frauds. reliance, and such reliance
was in fact induced. The
B Parol evidence rule. problem with this argument
here is that it is not clear that
C The daughter was an intended third-party the daughter relied on the
beneficiary. promise to give her $5,000,
because she already had
D The daughter married in reliance on the planned to get married. How-
promise. ever, none of the other choic-
es is a possible argument,
so (D) is her best choice.
(C) does not help her be-
cause she was not an in-
tended third-party beneficia-
ry. If a contract between two
parties contemplates perfor-
mance to a third party, that
third party may have rights
to enforce the contract. To do
so, the third party must be an
intended beneficiary at the
time the contract was made
(e.g., designated in the con-
tract). An assignment, on the
other hand, is a contract that
does not contemplate per-
formance to a third party
when the contract is made.
Rather, later one of the
parties transfers his rights
to another. Here, the con-
110 / 184
Barbri MBE questions
Study online at https://quizlet.com/_6umlok
tractor and the homeowner
signed their contract and lat-
er the contractor assigned
his rights to his daughter.
Thus, the daughter was not
an intended third-party ben-
eficiary who could enforce
the agreement, but merely
an assignee who gave no
consideration for the assign-
ment. As such, the contrac-
tor was free to revoke the as-
signment, and his daughter
cannot recover the $5,000.

104. A thief sold some stolen goods to a dealer. (C)


Several weeks later, the police raided the
dealer's store and arrested him. In this raid, The court should deny the
the police seized the goods the thief sold motion because the thief
to the dealer and a record book in which had no standing to object to
the dealer had recorded this transaction. the search. A person chal-
However, at the dealer's subsequent trial for lenging the admissibility of
receiving stolen goods, the charges against seized evidence must have
him were dismissed when the court ruled standing to do so. As a gen-
that the search warrant had been improperly eral rule, standing requires a
issued. person to have a reasonable
expectation of privacy in the
The police were able to trace the stolen place being searched or the
goods to the thief because of fingerprint item being seized. One may
identification and the information contained not challenge a search or
in the dealer's record book. seizure by claiming that an-
other person's constitution-
At his trial, the thief made a motion to sup- al rights have been violated.
press the stolen goods and record book. Here, the thief had no own-
ership interest in the deal-
What should the judge do? er's store. He had no rea-
sonable expectation of pri-
A Grant the motion, because the evidence vacy with respect to it; i.e.,
is the fruit of the poisonous tree in that the he was not present when the
111 / 184
Barbri MBE questions
Study online at https://quizlet.com/_6umlok
search of the dealer's store was improper. search was made, and he
had no ownership interest in
B Grant the motion, because the trial court the stolen goods. Thus, he
in the dealer's case has already ruled that lacks the standing to object
the evidence was improper. to their illegal seizure.

C Deny the motion, because the thief has no


standing to object to the search.

D Deny the motion, because the thief's fin-


gerprints on the stolen goods were what led
to his identification.

105. After the release of various news stories (C)


about the President's possible violation of
political campaign funding laws, a federal Executive privilege is an in-
grand jury investigation and an investiga- herent privilege necessary
tion by a special Senate subcommittee were to protect the confidentiali-
initiated. The Senate subcommittee subpoe- ty of presidential communi-
naed documents and records from several cations. Under this privilege,
top officers of the executive branch. Learn- presidential documents and
ing of the subpoenas, the President ordered conversations are presump-
all executive officials to refuse to turn over tively privileged, but this priv-
materials, claiming "executive privilege." ilege must yield to a demon-
strated need for such mate-
Which of the following statements is most rials as evidence in a crim-
accurate? inal case in which they are
relevant and otherwise ad-
A The subpoena violates the constitutional missible. [United States v.
principle of separation of powers. Nixon (1974)] Although the
Supreme Court has not ex-
B The President's executive privilege is ab- pressly decided that the priv-
solute, except in cases of impeachment. ilege must also yield to a
demonstrated need for evi-
C The presidential papers are presumptively dence in a pending legisla-
privileged, but the privilege must yield to a tive proceeding, such an ex-
demonstrated specific need for evidence in tension of Nixon is likely, and
a pending legislative proceeding. none of the other alterna-
tives is at all accurate.
112 / 184
Barbri MBE questions
Study online at https://quizlet.com/_6umlok
D The President's executive privilege ap-
plies to proceedings by Congress, but not
to proceedings by the courts.

106. A comprehensive federal health-care re- (B)


form statute created a Federal Health Pol-
icy Board, which was directed to monitor The fact that the federal
the fees charged for various medical pro- board was similar to the
cedures covered by insurance. The board state board but was not giv-
also had the power to subpoena records to en the power to restrict fee
determine whether fee increases were a true increases and impose sanc-
reflection of cost increases. Nothing in the tions in an otherwise com-
statute provided for caps on fee increases. prehensive bill suggests that
such provisions in the state
Because of the continuing escalation of law violate the Supremacy
health-care costs while the statute was Clause. A state law may
being debated, several states had passed fail under the Supremacy
health-care legislation on their own. One Clause even if it does not
state passed legislation that prohibited directly conflict with a feder-
most fee increases of 10% or more per year al statute or regulation if it
for specified health-care services covered interferes with the achieve-
by insurance, and created a health-care re- ment of a federal objective
view board to regulate these costs and or the federal regulations oc-
impose monetary penalties on health-care cupy the entire field. Where
providers or insurers that tried to circum- the federal laws are compre-
vent the cap. hensive or a federal agency
is created to oversee the
Which of the following would be the best field, preemption will often
basis for finding the state provision uncon- be found. The fact that the
stitutional? health-care legislation was
comprehensive but the fed-
A The federal legislation was passed after eral board was not given reg-
the state legislation and therefore super- ulatory or enforcement pow-
sedes it. er suggests that Congress
did not want specific restric-
B The Federal Health Policy Board was con- tions in these areas and
stituted with many of the same powers as may have wanted free-mar-
the state board but was not given the power ket principles to determine
to impose sanctions. fee increases at the out-
113 / 184
Barbri MBE questions
Study online at https://quizlet.com/_6umlok
set. The state board's power
C The state provision impairs existing con- to impose these restrictions
tracts between health-care providers and in- may violate the Supremacy
surers in violation of the Contract Clause. Clause under these circum-
stances. (A) is incorrect be-
D Health-care fee caps create an undue bur- cause the fact that the fed-
den on interstate commerce even in the ab- eral legislation was passed
sence of federal regulation. later does not automatically
mean that the state legisla-
tion has been superseded.
In areas of concurrent leg-
islative power, a state reg-
ulation will be upheld if it
does not conflict with and
is not preempted by feder-
al legislation. (C) is incor-
rect because the Contract
Clause prevents only sub-
stantial impairments of exist-
ing contracts by state legis-
lation, and only if the legis-
lation does not serve an im-
portant and legitimate public
interest or is not a reason-
able and narrowly tailored
means of promoting that in-
terest. Here, the law has a
prospective effect only, and
even if existing contracts be-
tween health-care providers
and insurers are affected
by the legislation, the oth-
er requirements for the Con-
tract Clause to apply are
not likely to be satisfied. (D)
is incorrect because states
may regulate local aspects
of interstate commerce in
the absence of federal reg-
114 / 184
Barbri MBE questions
Study online at https://quizlet.com/_6umlok
ulation as long as the reg-
ulation is nondiscriminatory
and does not unduly burden
interstate commerce, which
is a case-by-case balancing
test. Here, the legislation ap-
pears to be nondiscrimina-
tory and there are insuffi-
cient facts to establish that
it would constitute an un-
due burden; hence, (B) pre-
sents a stronger argument
than (D).

107. A gang member threatened to kill the de- (B)


fendant unless he robbed a convenience
store and gave the proceeds to the gang The defendant should be
member. The gang member also demanded convicted of common law
at gunpoint that the defendant kill the clerk murder, but acquitted of the
to prevent identification. In abject fear of his robbery. At common law,
life, the defendant did everything that the murder is the unlawful killing
gang member requested. of a human being with
malice aforethought. "Malice
If the defendant is arrested and charged aforethought" exists if the
with murder and robbery in a common law defendant has any of the fol-
jurisdiction, what result? lowing states of mind: (i) the
intent to kill (express malice);
(ii) the intent to inflict great
A The defendant should be convicted of bodily injury; (iii) a reckless
murder and robbery. indifference to an unjustifi-
ably high risk to human life
B The defendant should be acquitted of the ("abandoned and malignant
robbery and convicted of murder. heart"); or (iv) the intent to
commit a felony. In the in-
C The defendant should be convicted of rob- stant case, malice could be
bery, and the killing will be reduced to vol- found either by the intent to
untary manslaughter. kill (because the clerk was
intentionally killed to prevent
D The defendant should be acquitted of the identification) or by the in-
115 / 184
Barbri MBE questions
Study online at https://quizlet.com/_6umlok
robbery, and the killing should be reduced tent to commit a felony (the
to voluntary manslaughter. killing was committed dur-
ing the course of a robbery).
Robbery is an aggravated
form of larceny and consists
of the following elements: (i)
a taking; (ii) of the personal
property of another; (iii) from
the other's person or pres-
ence; (iv) by force or intimi-
dation; (v) with the intent to
permanently deprive him of
it. Clearly, the elements for
robbery are met here. Thus,
at first glance, the defendant
has committed both murder
and robbery. However, the
fact pattern also raises the
defense of duress. A per-
son is not guilty of an of-
fense, other than intentional
homicide, if he performs an
otherwise criminal act under
the reasonable belief that
another will imminently in-
flict death or great bodily
harm on him or an immedi-
ate family member if he does
not commit the criminal act.
In the instant case, the de-
fendant committed the rob-
bery under duress and thus
should be acquitted of that
charge, making (A) incor-
rect. However, duress would
not be effective against a
murder charge based on an
intent-to-kill theory, and here
the defendant intentionally
116 / 184
Barbri MBE questions
Study online at https://quizlet.com/_6umlok
killed the store clerk under
instructions from the gang
member to prevent identi-
fication. Thus, because the
defendant could be convict-
ed of an intent-to-kill murder,
but acquitted of the robbery
charge based on duress, (B)
is the correct answer.

108. A company that was the leading suppli- (C)


er of home water filtration systems had a
network of sales promoters who were un- The court should not grant
der contract for two- or three-year terms the promoter's motion be-
and were compensated solely by commis- cause the jury could find that
sions earned from sales and by occasion- the promoter used improp-
al bonuses. Veteran promoters also earned er means, while working for
commissions by recruiting other promoters the company, to divert the
for the company. One of the company's vet- sales rep for his own pur-
eran promoters was contacted by a former poses. To establish a pri-
top sales representative for another manu- ma facie case for interfer-
facturer who was looking for similar sales ence with business relations,
opportunities in the region. The sales rep the following elements must
knew that the promoter might be able to get be proved: (i) existence of
her a position with his company, which was a valid contractual relation-
looking for additional promoters. At the time ship between plaintiff and a
he met with the sales rep, the promoter's third party or a valid busi-
contract with the company had one more ness expectancy of plaintiff;
month to run. When the promoter's contract (ii) defendant's knowledge of
with the company expired, he announced the relationship or expectan-
that he was forming his own business to cy; (iii) intentional interfer-
market a different line of water filtration sys-
ence by defendant that in-
tems manufactured by a competitor of the duces a breach or termina-
company, and that the sales rep would be in tion of the relationship or
charge of his promotional network. expectancy; and (iv) dam-
age to plaintiff. Thus, a plain-
The company brought an action against the tiff has a cause of action
promoter for interference with business re- for interference with proba-
lations for hiring the sales rep. At a prelim- ble future business relation-
117 / 184
Barbri MBE questions
Study online at https://quizlet.com/_6umlok
inary hearing, the parties stipulated to the ships for which the plaintiff
above facts and that the promoter was an has a reasonable expecta-
independent contractor rather than an em- tion of financial benefit. On
ployee of the company. The promoter then the other hand, an interfer-
filed a motion for a summary judgment in er's conduct may be privi-
his favor. leged where it is a proper
attempt to obtain business
Should the court grant the promoter's mo- for the interferer, particular-
tion? ly if the interference is only
with a prospective business
A Yes, because the sales rep had no busi- relationship rather than with
ness relationship with the company at the an existing contract. What
time the promoter's alleged interference oc- is proper depends on var-
curred. ious factors, including the
means of persuasion used.
B Yes, because the promoter was an inde- Here, the promoter's con-
pendent contractor rather than an employee duct would not be privileged
of the company. if the jury were to find that he
improperly used his position
C No, because the jury could find that the with the company to develop
means the promoter used to obtain the a relationship with the sales
sales rep were not privileged. rep.

D No, because the jury could find that the


promoter breached his contract with the
company by meeting with the sales rep.

109. A buyer and seller entered into a written (C)


contract on March 31 for the sale of a beach
house. Under the terms of the agreement, The buyer's best argument
the buyer would purchase the house for is that the seller's assur-
$275,000, with 10% due at closing on May 1 ances that there was no
and a 15-year mortgage. At the time the con- problem with the buyer's fail-
tract was entered into, the parties agreed ure to provide written notifi-
orally that the written agreement would not cation by April 15 amounts
become binding unless the buyer notified to a waiver of the condi-
the homeowner, in writing, by the end of the tion. The buyer's written no-
day on April 15, that she had secured the tification by April 15 that
proper financing. With the summer season he had obtained the prop-
118 / 184
Barbri MBE questions
Study online at https://quizlet.com/_6umlok
approaching, the seller did not wish to risk er financing was a condition
any delay in selling the house if the buyer precedent to the seller's ab-
was not in a position to buy it. On the morn- solute duty to perform un-
ing of April 15, the buyer's financing was der the contract. It is clear
approved. On April 16, the buyer telephoned that the buyer did not pro-
the seller and told him that her financing vide the required notification
had been approved. The buyer also told the by April 15; thus, the condi-
seller that she was not able to get written tion was not fulfilled. How-
confirmation to him by April 15 because of ever, one having the ben-
the postal workers' slowdown and because efit of a condition may in-
her fax machine just broke down. The seller dicate by words or conduct
assured the buyer that this was not a prob- that he will not insist upon
lem. However, before closing, the seller had it. When a condition is bro-
a change of heart and decided not to sell ken, the beneficiary of the
the beach house after all. The buyer files an condition has an election: (i)
action for breach. he may terminate his liability;
or (ii) he may continue un-
Which of the following would be the basis der the contract. If a choice
for the buyer's best argument? is made to continue under
the contract, the person is
A Statute of Frauds. deemed to have waived the
condition. The seller was ful-
B Parol evidence rule. ly aware that the buyer had
not satisfied the condition,
C Waiver of condition. yet, when speaking with the
buyer on April 16, he stat-
D Excuse of condition by hindrance. ed unequivocally that it was
not a problem. This is a def-
inite indication that the sell-
er elected to continue under
the contract. Having so elect-
ed, the seller is deemed to
have waived the condition.
Therefore, the seller's duty
of performance under the
contract became absolute.
Regarding (A), the only way
the Statute of Frauds could
bolster the buyer's position
119 / 184
Barbri MBE questions
Study online at https://quizlet.com/_6umlok
would be if the original oral
agreement setting forth the
condition were required by
the Statute to be in writ-
ing. If that were the case,
the buyer could argue that
the condition is unenforce-
able because it is not in writ-
ing. However, the oral agree-
ment is not of a type that
falls within the purview of the
Statute of Frauds. Therefore,
the Statute of Frauds will
provide no help to the buy-
er. Similarly, regarding (B),
the parol evidence rule could
help the buyer only if it
could be used to preclude
admissibility of the original
oral agreement. Under the
parol evidence rule, where
the parties to a contract ex-
press their agreement in a
writing with the intent that
it embody the final expres-
sion of their bargain, any ex-
pression made prior to the
writing and any oral expres-
sion contemporaneous with
the writing is inadmissible to
vary the terms of the writ-
ing. However, where it is as-
serted that there was an
oral agreement that the writ-
ten contract would not be-
come effective until the oc-
currence of a condition, ev-
idence of the oral agree-
ment may be offered and re-
120 / 184
Barbri MBE questions
Study online at https://quizlet.com/_6umlok
ceived. Because the original
oral agreement between the
seller and buyer established
a condition precedent to the
effectiveness of the written
agreement, the buyer will be
unable to raise the parol ev-
idence rule as a bar to the
admissibility of evidence re-
lating to the oral agreement.
(D) is unsupported by the
facts. If a party with a duty
of performance that is sub-
ject to a condition prevents
the condition from occurring,
he no longer has the benefit
of the condition. This is re-
ferred to as excuse of con-
dition by hindrance. The sell-
er did nothing to prevent the
occurrence of the condition
regarding written notification
by April 15. Consequently,
it cannot be said that the
condition is excused by hin-
drance.

110. The state has the following homicide (B)


statutes:
The defendant's strongest
Murder is the unlawful killing of a human argument is that her asso-
being with malice aforethought. Such mal- ciate's death was justifiable
ice may be express or implied. It is express homicide. Most courts today
when there is manifested a deliberate in- would not allow the defen-
tention to unlawfully take away the life of a dant to be convicted on a
fellow creature. It is implied when no con- felony murder theory when a
siderable provocation appears or when the co-felon is killed by a third
circumstances attending the killing show an party during the crime. Some
abandoned and malignant heart. All mur- courts base this result on
121 / 184
Barbri MBE questions
Study online at https://quizlet.com/_6umlok
der that is perpetrated by willful, deliberate, the fact that the person who
or premeditated killing or committed in the did the killing was justified
perpetration of or attempt to perpetrate ar- in doing so. (A) is wrong.
son, rape, robbery, or burglary is murder of Aside from the fact that the
the first degree. All other kinds of murders attempt to steal from the jew-
are of the second degree. elry store is probably statu-
The defendant and her associate entered a tory burglary, the fact that
jewelry store to shoplift a diamond bracelet. the associate attempted to
Just as the defendant put the bracelet aid the defendant in steal-
into her pocket, a sales clerk saw her and ing the bracelet by attack-
grabbed her by the wrist. The associate ing the clerk with a knife
grabbed a knife from one of the silver dis- is probably robbery. When
plays and lunged at the sales clerk, but then or how the associate came
a store guard shot and killed her. The defen- by the dangerous weapon
dant is charged with the first degree murder is immaterial. (C) is wrong
of her associate. because the circumstances
of one co-felon breaking an
Which of the following is the defendant's agreement not to commit vi-
strongest argument? olence would not prevent the
application of the felony mur-
A The defendant cannot be convicted of der rule if it were otherwise
murder because when they went into the applicable. (D) is wrong be-
store they were not carrying any weapons; cause if the felony murder
therefore, there was no felony on which the rule is otherwise applicable,
felony murder rule may arise. the fact that the person who
killed the co-felon may have
B The defendant cannot be convicted of mistaken the co-felon's in-
murder because the associate's death was tentions does not prevent the
not murder but justifiable homicide. operation of this rule.

C The defendant cannot be convicted of


murder because she and her associate had
an agreement never to use violence when
they stole anything.

D The associate did not intend to hurt the


sales clerk, but just wanted to scare him so
that the defendant could run.

122 / 184
Barbri MBE questions
Study online at https://quizlet.com/_6umlok
111. A seller entered into a written contract to (B)
sell a tract of land to a buyer. The buyer was
to pay $1,500 per month for five years, at The seller is more likely to
which time the seller would deliver a war- prevail because the buyer
ranty deed. The contract was silent as to the
has no basis on which to re-
quality of title to be conveyed. After making
scind the contract. Absent a
12 payments, the buyer discovered that a provision to the contrary, a
neighbor had an easement of way over the contract for the sale of land
land, which was not discussed at the time contains an implied promise
the seller and buyer entered into the con- by the seller that she will
tract. The neighbor had not used the ease- deliver to the buyer a mar-
ment over the previous year because she ketable title at the time of
had been out of the country. On the basis closing. This promise impos-
of the easement, the buyer wishes to cancel es on the seller an obliga-
the contract. tion to deliver a title that is
free from reasonable doubt;
Which party is more likely to prevail? i.e., free from questions that
might present an unreason-
able risk of litigation. Title
A The seller, because the neighbor's ease- is marketable if a reason-
ment has been extinguished. ably prudent buyer would ac-
cept it in the exercise of or-
B The seller, because the buyer has no basis dinary prudence. An ease-
on which to rescind the contract. ment that reduces the val-
ue of the property (e.g., an
C The buyer, because the obligation to con- easement of way for the ben-
vey marketable title is implied. efit of a neighbor) general-
ly renders title unmarketable.
D The buyer, because the seller has If the buyer determines, pri-
breached the covenant against encum- or to closing, that the sell-
brances. er's title is unmarketable, he
must notify the seller and
allow a reasonable time to
cure the defect. If the sell-
er is unable to acquire ti-
tle before closing, so that
title remains unmarketable,
the buyer can rescind, sue
for damages caused by the
123 / 184
Barbri MBE questions
Study online at https://quizlet.com/_6umlok
breach, or obtain specific
performance with an abate-
ment of the purchase price.
However, the buyer cannot
rescind prior to closing on
grounds that the seller's ti-
tle is unmarketable. Where
an installment land contract
is used, the seller's obliga-
tion is to furnish marketable
title when delivery is to oc-
cur, e.g., when the buyer
has made his final payment.
Thus, a buyer cannot with-
hold payments or seek oth-
er remedies on grounds that
the seller's title is unmar-
ketable prior to the date
of promised delivery. Here,
there is a valid easement
on the property (see below),
but the seller has four years
in which to cure this defect.
Thus, the buyer cannot yet
rescind on grounds that title
is unmarketable.

112. Specific Intent Crimes solicitation, conspiracy, at-


tempt, first degree murder,
assault, larceny, embezzle-
ment, false pretenses, rob-
bery, burglary, forgery

113. Congress created a seven-member safety (D)


commission to investigate and make recom-
mendations to Congress for new fireworks The strongest argument is
safety laws, to make further rules for estab- that the commission lacks
lishing safety and performance standards, authority to enforce its stan-
and to prosecute violations of these safety dards. The Appointments

124 / 184
Barbri MBE questions
Study online at https://quizlet.com/_6umlok
standards. The chairman of the commission Clause of the Constitution
was appointed by the President. Three mem- permits Congress to vest ap-
bers were selected by the Speaker of the pointments of inferior offi-
House of Representatives, and three mem- cers only in the President,
bers were selected by the President pro the courts, or the heads of
tempore of the Senate. An organization with departments. Enforcement
proper standing seeks to enjoin enforce- is an executive act; there-
ment of the commission's rules. fore, Congress cannot ap-
point members of a com-
Which of the following presents the mission that exercises en-
strongest constitutional argument that the forcement powers. In these
organization can make against the validity facts, the safety commis-
of the commission? sion consists of some mem-
bers appointed by Congress.
A The commerce power does not extend to This commission therefore
activities occurring solely within a state. violates the Appointments
Clause of the Constitution
B Legislative power may not be delegated because it has enforcement
by Congress to an agency in the absence of powers, since it can pros-
clear guidelines. ecute violations. Legislative
power can be delegated
C The organization is denied due process even under vague guidelines
of law because it is not represented on the without creating an uncon-
commission. stitutional delegation. Thus,
(B) is incorrect. (A) is incor-
D The commission lacks authority to en- rect because the commerce
force its standards because some of its power extends to many ac-
members were appointed by Congress. tivities occurring solely with-
in a particular state, if, taken
as a whole, the activities in
question have a substantial
economic effect on interstate
commerce. There is no con-
stitutional provision requir-
ing that a party or group to
be regulated be represented
on the commission that reg-
ulates it. (C) is therefore in-
correct.
125 / 184
Barbri MBE questions
Study online at https://quizlet.com/_6umlok

114. A park board in a large suburb announced (B)


that it was accepting bids for renovation
work on its recreation center. A builder The electrician's best argu-
advertised for sub-bids for the electrical ment would be that the great
work, and a local electrician submitted to difference between the elec-
the builder by electronic bidding service a trician's sub-bid, as trans-
sub-bid of $130,000. However, due to the mitted, and the next lowest
bidding service's negligence, the sub-bid sub-bid should have alert-
that the builder received from the electri- ed the builder to the ob-
cian read $30,000 instead of $130,000. Be- vious mistake in the elec-
cause this was the lowest sub-bid that the trician's sub-bid. Typically, if
builder received for the electrical work, and
only one of the parties en-
$60,000 less than the next lowest sub-bid, tering into a contract is mis-
the builder awarded the subcontract to the taken about facts relating to
electrician. Based in part on the electrician's
the agreement, the unilateral
sub-bid, the builder came up with a bid for mistake will not prevent for-
the job that beat out all of the competitionmation of a contract. Howev-
and won the job. er, if the nonmistaken par-
ty is or had reason to be
What is the electrician's best argument to aware of the mistake made
successfully refuse to perform the resulting by the other party, the con-
contract? tract is voidable by the par-
ty who made the mistake.
A The contract would be unconscionable. Thus, the electrician's best
argument is that the $60,000
B The builder should have been alerted to difference between the elec-
the existence of a mistake in the sub-bid. trician's bid of $30,000 and
the next lowest bid should
C The bidding service, not the electrician, have alerted the builder to
was responsible for the faulty bid. the existence of a mistake,
so the electrician should be
D The builder's failure to check out all able to refuse to perform the
sub-bids precludes enforcement of the con- contract. (C) is incorrect be-
tract. cause, under the prevailing
view, where there is a mis-
take in the transmission of
an offer by an intermediary,
the offer as transmitted is
operative unless the other
126 / 184
Barbri MBE questions
Study online at https://quizlet.com/_6umlok
party knew or should have
known of the mistake. Thus,
the electrician would not be
excused from performance
based on the faulty transmis-
sion alone.

115. A criminal suspect filed an action in federal (A)


district court against the police officer who
arrested him and the city that employed the The court has subject matter
officer, claiming that the officer beat him jurisdiction over both claims.
even though he was not resisting arrest. The Federal question jurisdiction
suspect's action asserted a battery claim is available when the plain-
against the police officer and a federal statu- tiff, in his well-pleaded com-
tory claim against the city for violation of the plaint, alleges a claim that
suspect's civil rights. The suspect and the arises under federal law.
police officer are citizens of State A, and the Here, the suspect claims
city in which the arrest took place is in State that his civil rights were vio-
A. The action seeks $50,000 each from the lated, which is a valid feder-
police officer and the city. al claim; thus, the court has
federal question jurisdiction
Does the federal district court have subject over that claim. The battery
matter jurisdiction over either of the claims? claim against the police of-
ficer does not arise under
A The court has federal question jurisdic- federal law, but rather un-
tion over the federal statutory claim against der state law. However, when
the city and supplemental jurisdiction over the federal court has subject
the tort claim against the police officer. matter jurisdiction over one
claim, it has discretion to ex-
B The court has federal question jurisdic- ercise supplemental jurisdic-
tion over the federal statutory claim against tion over related claims that
the city, but it does not have subject matter derive from the same com-
jurisdiction over the tort claim against the mon nucleus of fact and are
police officer because the amount of that such that a plaintiff would or-
claim is too small. dinarily be expected to try
them in a single judicial pro-
C The court has federal question jurisdic- ceeding. Here, the suspect's
tion over the federal statutory claim against claim against the city is de-
the city, but it lacks subject matter jurisdic- rived from the officer's bat-
127 / 184
Barbri MBE questions
Study online at https://quizlet.com/_6umlok
tion over the tort claim against the police of- tery, which is the basis of the
ficer because it is a state law claim between state law claim. As a result,
two citizens of the same state. the two claims arise from
the same nucleus of com-
D The court does not have subject matter mon fact. Thus, the battery
jurisdiction over either claim because the claim may be heard in feder-
amount of each claim is too small. al court under supplemental
jurisdiction.

116. A wholesale seller sent a fax to a manu- (D)


facturer with whom he had done business
before: "Send 500 'Madewell' chairs at your The manufacturer will re-
usual price." The manufacturer responded, cover only his incidental
also by fax, that the line was being discon- damages, i.e., the costs of
tinued, but he would ship his last 500 chairs preparing to ship the chairs.
at $75 per chair, his usual price. The man- An offer calling for shipment
ufacturer immediately began the paperwork of goods, such as the offer
for processing the order and started prepar- here, may be accepted by
ing and packing the chairs for shipment. prompt shipment with notice
Before the chairs could be delivered, the or by a promise to ship. Ac-
wholesaler canceled his order, noting that ceptance forms a contract.
the price was too high. The day after receiv- Here, the manufacturer ac-
ing the wholesaler's cancellation, the manu- cepted the wholesaler's of-
facturer sold the chairs to another buyer for fer by promising to ship,
$75 each. and a contract was formed.
The wholesaler breached
If the manufacturer sues the wholesaler for the contract by canceling his
damages, how much should he recover? order. When a buyer breach-
es by repudiating his offer,
A Nothing, because this was a contract be- as the wholesaler did here,
tween merchants and the wholesaler can- the seller has a right to
celed within a reasonable time. recover his incidental dam-
ages plus either the differ-
B Nothing, because the manufacturer was ence between the contract
able to cover by selling the chairs at the price and the market price or
same price he would have received from the the difference between the
wholesaler. contract price and the re-
sale price of the goods, re-
C $37,500, the full contract price, because duced in either case by any
128 / 184
Barbri MBE questions
Study online at https://quizlet.com/_6umlok
the wholesaler breached the contract and expenses saved as a re-
$75 per chair was a fair price. sult of the breach. Here, the
manufacturer made what he
D The incidental costs of preparing the pa- would have if the sale with
perwork and other office costs connected the wholesaler had gone
with preparing and packing the chairs for through—there was no dif-
shipment to the wholesaler. ference between the con-
tract price and the resale
price. Thus, the manufactur-
er would be limited to his in-
cidental damages. Lost prof-
its would not be available be-
cause the chairs were the
last ones that the manufac-
turer had and would have
because the line was be-
ing discontinued. Therefore,
the manufacturer would not
have been able to sell anoth-
er set of 500 chairs to anoth-
er potential buyer.

117. A homeowner returned home from work one (C)


day to find a robber in her living room. After
a brief physical altercation, the homeowner The testimony should not
ran to a bedroom, hid in a closet, and called be admitted. Under the Con-
911 on her cell phone. Police officers arrived frontation Clause, an ac-
in less than two minutes and were able to cused has the right to be
apprehend the robber as he tried to run out confronted by the witness-
the front door. Once they made sure he was es against him. A hearsay
locked in the police car, one of the officers statement will not be admit-
went to speak with the homeowner about ted—even if it falls within
what had happened. She was still crying a hearsay exception—when:
and shaking when the officer found her, and (i) the statement is offered
she said, "Thank you for catching him! He against the accused in a
punched me in the head as I was running criminal case; (ii) the de-
away!" The robber was charged with rob- clarant is unavailable; (iii)
bery and assault. Traumatized, the home- the statement was testimo-
owner left the country and cannot be traced, nial in nature; and (iv) the
129 / 184
Barbri MBE questions
Study online at https://quizlet.com/_6umlok
despite the efforts of the prosecutor. The accused had no opportunity
prosecutor intends to call the officer to tes- to cross-examine the declar-
tify as to the homeowner's statement. ant's "testimonial" statement
Should the court allow the officer's testimo- prior to trial. The Supreme
ny? Court has established that if
the primary purpose of po-
A Yes, because the homeowner's statement lice interrogation is to en-
is an excited utterance. able the police to help in an
ongoing emergency, state-
B Yes, because the homeowner's statement ments made in the course
is not testimonial. of the interrogation are non-
testimonial. When the prima-
C No, because admitting the homeown- ry purpose of the interroga-
er's statement would violate the defendant's tion is to establish or prove
constitutional rights. past events potentially rele-
vant to a later criminal prose-
D No, because the homeowner's statement cution, statements are testi-
is hearsay not within any exception. monial. Here, the homeown-
er's statement was hearsay
because it was made out of
court and is being offered
for its truth—that the rob-
ber punched her in the head.
It appears that the emer-
gency had already resolved
by the time the statement
was made. Although only
a few minutes had passed
since the physical altercation
and the homeowner was still
upset, the robber no longer
posed any danger because
he was locked in the police
car and the homeowner was
aware of this ("Thank you
for catching him!"). There-
fore, the homeowner's state-
ments to the officer were tes-
timonial. Because the home-
130 / 184
Barbri MBE questions
Study online at https://quizlet.com/_6umlok
owner is unavailable to tes-
tify at trial and the rob-
ber has had no opportunity
to cross-examine the state-
ments, admitting them at tri-
al through the testimony of
the officer would violate the
Confrontation Clause. (A) is
incorrect. Under the excited
utterance exception, a dec-
laration made by a declarant
during or soon after a star-
tling event is admissible if it
relates to the startling occur-
rence and was made under
the stress of excitement pro-
duced by the startling event.
Here, finding a robber in
one's home is a startling
event, and the homeowner
was still under the stress
of the excitement when she
made the statement—she
was crying and shaking. The
statement likely qualifies as
an excited utterance. How-
ever, even a hearsay state-
ment that falls within an ex-
ception may be barred by the
Confrontation Clause, as is
the case here. (B) is incor-
rect. As stated above, when
the primary purpose of the
interrogation is to establish
or prove past events poten-
tially relevant to a later crim-
inal prosecution, statements
are testimonial. Because the
homeowner's statement was
131 / 184
Barbri MBE questions
Study online at https://quizlet.com/_6umlok
made for this purpose and
not to address an ongoing
emergency, it was testimo-
nial in nature and its ad-
mission would violate by the
Confrontation Clause.

118. A boater taking his new powerboat out on (B)


a large lake ran out of gas because of a
defective seal in the gas tank. The defect The captain's estate can re-
was not discoverable by an ordinary inspec- cover from the dealer on
tion. His frantic signaling alerted the cap- a strict products liability
tain of a sightseeing boat passing by. The ground because the captain
captain pulled up alongside to assist and was a foreseeable bystander
attempted to restart the boat. A spark ignit- and the dealer is a commer-
ed a pool of gas that had leaked from the cial supplier. Recovery in a
gas tank and collected in the lower part of wrongful death action is al-
the boat, causing an explosion and fire. The lowed only to the extent that
captain was severely burned and died from the deceased could have re-
his injuries. The captain's estate brought a covered in a personal injury
wrongful death action based on strict lia- action had he lived. The cap-
bility against the powerboat dealer and the tain could have recovered
manufacturer. Evidence at trial established from the dealer in a products
that the dealer had sold the manufacturer's liability action based on strict
boats for years without any problems re- liability because the dealer
ported by customers. is a commercial supplier of
the boat and is liable if it
Can the captain's estate recover any dam- sold a product that was so
ages from the dealer? defective as to be unreason-
ably dangerous. The defect
A Yes, unless the jury finds that the boater in the assembly of the gas
was negligent in failing to investigate where tank was unreasonably dan-
the gas had gone. gerous because it allowed
gas to leak out and collect
B Yes, because harm to someone in the cap- where it could be ignited.
tain's position was a foreseeable result of The dealer would be liable to
the gas leak. the captain, despite the fact
that he was not in privity with
C No, because the dealer had no reason to the dealer, because he was
132 / 184
Barbri MBE questions
Study online at https://quizlet.com/_6umlok
anticipate that the manufacturer assembled a foreseeable plaintiff. The
the gas tank improperly. disabling effect of the gas
leak made it foreseeable that
D No, because the captain did not have a someone passing by would
sufficient relationship to the boater to make come to the boater's as-
the dealer liable for the captain's death. sistance and thereby come
within the zone of danger
from the leak (i.e., danger
invites rescue). The explo-
sion that resulted from the
leak was the actual and
proximate cause of the cap-
tain's death. Therefore, the
captain's estate can recov-
er damages from the deal-
er. (A) is wrong because, as
with proximate cause analy-
sis in ordinary negligence
actions, the negligence of a
subsequent actor is foresee-
able and therefore not a su-
perseding cause that would
cut off the liability of the orig-
inal tortfeasor. In any prod-
ucts liability case, the negli-
gent failure of an intermedi-
ary to discover the defect or
the danger does not void the
commercial supplier's strict
liability. Hence, the boater's
negligence would be irrele-
vant. (C) is wrong because
in products liability actions
based on strict liability, the
retailer may be liable even
if it had no reason to antic-
ipate that the product was
dangerous or had no oppor-
tunity to inspect the product
133 / 184
Barbri MBE questions
Study online at https://quizlet.com/_6umlok
for defects. While the deal-
er could assert that defense
if the action were based on
negligence, the call of the
question indicates that the
action is based on a strict
liability theory. Under strict
liability, the dealer is liable
simply because it is a com-
mercial supplier of a product
with a dangerous defect. (D)
is wrong because the fact
that the captain was not in
privity with the dealer is ir-
relevant in a products liability
action based on strict liabil-
ity. Strict liability applies not
only to buyers, but also to the
buyer's family, friends, and
employees, and to foresee-
able bystanders. As a res-
cuer, the captain was a fore-
seeable bystander to whom
the dealer may be liable.

119. A husband and wife were traveling in a car (D)


with the wife driving when they were in an
accident with a truck. The accident occurred The court should deny the
in a jurisdiction that followed the traditional truck driver's motion be-
rule as to joint and several liability among cause the wife is not "need-
tortfeasors. The husband sued the truck dri- ed for just adjudication."
ver in federal district court. The truck driver, When deciding whether an
contending that the wife was an indispens- absentee party is indispens-
able party, filed a motion to dismiss the ac- able, the court considers a
tion because the husband did not join his number of factors. However,
wife as a party to the action. the Supreme Court has held
that a joint tortfeasor sub-
How should the court rule on the truck dri- ject to joint and several lia-
ver's motion? bility is not a person needed
134 / 184
Barbri MBE questions
Study online at https://quizlet.com/_6umlok
for just adjudication. [Temple
A Grant the motion, because the wife is an v. Synthes Corp., 498 U.S.
indispensable party. 5 (1990)] Because the wife
here is a joint tortfeasor sub-
B Deny the motion and order service of ject to joint and several li-
process on the wife to make her a party, pro- ability, she is not an indis-
vided she is subject to the court's personal pensable party in this case.
jurisdiction. Therefore, (D) is correct, and
(A) and (B) are incorrect. (C)
C Deny the motion, because the wife and the is incorrect because the wife
truck driver cannot be joined as co-defen- could be joined as a co-de-
dants in the action. fendant if the husband chose
to do so; a spouse's immu-
D Deny the motion, because the wife is not nity from being sued by the
"needed for just adjudication." other spouse has been abol-
ished in almost all jurisdic-
tions.

120. The owner of an apartment building con- (D)


tracted with a painter to paint the porches
of the apartments for $5,000. The contract The painter will be unable
was specifically made subject to the own- to recover the $500 because
er's good faith approval of the work. The he did not satisfy the con-
painter finished painting the porches. The dition precedent to payment
owner inspected the porches and believed under the contract. A party
in good faith that the painter had done a does not have a duty to per-
bad job. The painter demanded payment, but form if a condition precedent
the owner told him that the paint job was to that performance has not
poor and refused to pay. The painter pleaded been met. Here, the parties
that he was desperately in need of money. made the owner's satisfac-
The owner told the painter that she would tion with the painter's paint
pay him $4,500, provided he repainted the job a condition precedent to
porches. The painter reluctantly agreed, and the owner's duty to pay the
the owner gave the painter a check in the $5,000. Because the own-
amount of $4,500. The painter went to his er was not satisfied with the
bank, indorsed the check "under protest" paint job, her duty to pay the
and signed his name, then deposited the painter never arose. The fact
check in his account. He never returned to that the owner offered to give
repaint the porches. the painter $4,500 if he re-
135 / 184
Barbri MBE questions
Study online at https://quizlet.com/_6umlok
painted the porches has no
The painter sues the owner for $500, which effect on this analysis, be-
he believes is still owed to him on his con- cause the offer constituted
tract to paint the porches. Will he prevail? a new contract, the owner
having been excused from
A Yes, because he indorsed the check "un- the old one. (A) is wrong
der protest." because it does not mat-
ter whether the painter in-
B Yes, but only if he repaints the porches. dorsed under protest. The in-
dorsement will not change
C Yes, because he performed the contract the result here because the
by painting the porches the first time. new contract did not seek
to discharge any contractu-
D No, even if he repaints the porches. al duty—the owner was al-
ready excused from her du-
ties because the condition
precedent was never met.
(B) is wrong because the old
contract, which provided for
payment of $5,000, is con-
sidered to be at an end. Un-
der the terms of the new con-
tract, the painter is entitled
to only $4,500, provided he
repaints the porches. (C) is
wrong because the condition
precedent to the payment
of $5,000, the owner's sat-
isfaction, was not met. The
courts have held such con-
ditions to be valid—not illu-
sory promises—because of
the promisor's duty to exer-
cise good faith in assessing
satisfaction. Here, the facts
state that the owner believed
in good faith that the painter
had done a bad job; thus, the
painter is not entitled to pay-
136 / 184
Barbri MBE questions
Study online at https://quizlet.com/_6umlok
ment under the original con-
tract. Note that since he has
not performed under the new
contract, he is in breach and
not entitled to the $4,500 al-
ready paid.

121. A landlord entered into a written lease of a (B)


bakery for a term of 25 years with a baker.
The parties agreed to a right of first refusal
The chocolatier can enforce
if the bakery was offered for sale during the option to purchase be-
the term of the lease. The lease also per- cause it is a covenant that
mitted assignments and subleases on no- runs with the land. When
tice to the landlord. Three years later, thea tenant makes a complete
baker retired and, after notifying the land-transfer of the entire remain-
lord, transferred the lease to a chocolatier.
ing term of his leasehold in-
Twenty-one years later, the landlord enteredterest, it constitutes an as-
into a contract with a buyer for the sale ofsignment. The assignee and
the bakery for $100,000. The landlord had the landlord are then in
informed the buyer of the lease but had for-privity of estate, and each
gotten about the right of first refusal. When
is liable to the other on
the chocolatier learned of the sale to the all covenants in the lease
buyer, she informed both the landlord and that run with the land. The
the buyer that she wanted to exercise her covenant runs with the land if
option and was prepared to purchase the the original parties so intend
bakery for the contract price. The jurisdic-and the covenant "touches
tion's Rule Against Perpetuities is unmodi- and concerns" the leased
fied by statute. land, i.e., burdens the land-
lord and benefits the tenant
Can the chocolatier enforce the option? with respect to their interests
in the property. Here, the
A Yes, because an option held by a tenant on transfer of the lease to the
leased property cannot be separated from chocolatier was an assign-
the leasehold interest. ment, making all covenants
in the lease that run with the
B Yes, because the option touches and con- land enforceable by the as-
cerns the leasehold estate. signee. The right of first re-
fusal burdens the landlord's
C No, because the transfer to the choco- power of alienation over the
137 / 184
Barbri MBE questions
Study online at https://quizlet.com/_6umlok
latier made the option void under the Rule bakery, and there is nothing
Against Perpetuities. to indicate that the parties in-
tended the option to be per-
D No, because the option was not specifical- sonal to the baker. Hence,
ly included when the lease was transferred the chocolatier can enforce
to the chocolatier. the option and purchase the
property. (D) is incorrect be-
cause, as discussed above,
the option is a covenant that
runs with the land regard-
less of whether it was speci-
fied in the assignment to the
chocolatier. The chocolatier,
as the assignee of the lease-
hold, can enforce the option
on privity of estate grounds.

122. Pursuant to a contract, a landscaper per- (B)


formed $30,000 of landscape work for a
homeowner. By coincidence, the homeown- The landscaper cannot as-
er and the landscaper were involved in an sert and maintain a coun-
automobile accident that was unrelated to terclaim against the home-
the landscape work. The homeowner was in- owner for breach of con-
jured in the accident and sued the landscap- tract because the federal
er in federal district court for negligence, court does not have sub-
seeking $100,000 in damages. The home- ject matter jurisdiction over
owner and the landscaper are citizens of the contract claim. The court
different states. has diversity of citizenship
jurisdiction over the negli-
May the landscaper assert and maintain gence claim because there
a counterclaim against the homeowner for is complete diversity and the
breach of contract, seeking the $30,000 due amount in controversy ex-
under the landscape contract? ceeds $75,000. A compul-
sory counterclaim (a claim
that arises out of the same
A No, because the Federal Rules of Civil transaction or occurrence as
Procedure permit a counterclaim only if the the plaintiff's claim) does not
counterclaim arises from the same transac- need to meet the jurisdiction-
tion or occurrence as the plaintiff's initial al amount requirement for
138 / 184
Barbri MBE questions
Study online at https://quizlet.com/_6umlok
claim. diversity jurisdiction. Howev-
er, a permissive counter-
B No, because, while the court has diversity claim (a claim that arises
of citizenship jurisdiction over the home- out of an unrelated trans-
owner's negligence claim and the Federal action) must meet the ju-
Rules of Civil Procedure permit the counter- risdictional amount require-
claim, the federal court does not have sub- ment. Here, the landscap-
ject matter jurisdiction over the landscap- er's contract claim would be
er's contract claim. a permissive counterclaim
because it is unrelated to
C Yes, because the Federal Rules of Civil the negligence claim arising
Procedure permit the counterclaim, and the from the accident. Therefore,
court has diversity of citizenship jurisdic- the claim's amount in con-
tion over both claims. troversy would have to ex-
ceed $75,000. The claim is
D Yes, because the court has diversity of cit- only for $30,000; therefore,
izenship jurisdiction over the homeowner's the court does not have sub-
negligence claim, and it has supplemental ject matter jurisdiction over
jurisdiction over the landscaper's contract the contract claim. (A) is in-
claim because it is a compulsory counter- correct because a defendant
claim. may assert claims against
a plaintiff that are unrelated
to the plaintiff's claims, pro-
vided there is subject mat-
ter jurisdiction. (C) is incor-
rect because the court does
not have diversity jurisdiction
over the contract claim. (D)
is incorrect because the con-
tract claim is not a compulso-
ry counterclaim.

123. In an action by the plaintiff against the de- (A)


fendant, one of the issues is whether the
defendant is a licensed physical therapist. The document should be ad-
Normally, the names of all licensed physical mitted. Related to the excep-
therapists are registered with the office of tion to the hearsay rule for
the state Department of Professional Reg- public records and other offi-
istrations. The plaintiff wishes to introduce cial writings, Federal Rule of
139 / 184
Barbri MBE questions
Study online at https://quizlet.com/_6umlok
a certified document, signed by the chief Evidence 803(10) provides
registrar of the department (who cannot be that evidence in the form of
located), stating that an examination of the a certification or testimony
department's rolls does not disclose the de- from the custodian of pub-
fendant's name. lic records that she has dili-
gently searched and failed
Should the document be admitted? to find a record is admissi-
ble to prove that a matter
A Yes, because a statement of absence from was not recorded, or, infer-
public record is admissible. entially, that a matter did not
occur. Here, the defendant's
B Yes, because the chief registrar is unavail- status as a licensed physi-
able. cal therapist would normally
be revealed in the records
C No, because the document is hearsay not of the department. The doc-
within an exception. ument here at issue is ad-
missible, under the forego-
D No, because the document is not self-au- ing hearsay exception, as a
thenticating. means of proving that the
defendant is in fact not li-
censed. (B) is incorrect be-
cause this hearsay excep-
tion does not require un-
availability of the declarant.
(C) is incorrect because, as
explained above, the state-
ment of absence from pub-
lic record forms an excep-
tion to the hearsay rule.
(D) is incorrect because a
public document that has
been signed and certified
is self-authenticating under
Rule 902; hence, no testimo-
nial sponsorship for the doc-
ument is required.

124. The defendant was arrested, given Miran- (D)


da warnings, and charged with burglary. At
140 / 184
Barbri MBE questions
Study online at https://quizlet.com/_6umlok
the police station, he telephoned his mother The defendant's confession
and asked her to come to the station to should be admitted because
post bail. Instead, his mother immediately he waived his Fifth Amend-
called the family attorney. In the meantime, ment privilege against com-
the police had begun questioning the defen- pelled self-incrimination af-
dant. Although he never told the police to ter receiving Miranda warn-
stop the questioning, his answers were at ings. Miranda v. Arizona re-
first vague or clearly unresponsive. During quires that a person in cus-
the course of the questioning, the family tody be informed of his right
attorney phoned the station and told the to remain silent and his right
police that she had been hired to represent to the presence of an at-
the defendant and would be there in half torney during questioning. A
an hour. The police did not inform the de- suspect may subsequently
fendant of the attorney's call. Ten minutes waive his rights by mak-
later, the defendant admitted to committing ing a confession, as long
the burglary, and signed a statement to that as the waiver was knowing
effect prepared by the police. The attorney and voluntary. In this case,
arrived a few minutes later and advised the the defendant received prop-
defendant to remain silent, but he told her er Miranda warnings, and
that he had already signed a confession. there is no indication that
he did not understand what
How should the court rule on the attorney's his rights were. Although his
pretrial motion to exclude the confession as answers during questioning
evidence at trial? were initially unresponsive,
he never asked for an at-
A Grant the motion, because the police had torney or indicated that he
a duty to inform the defendant that an attor- wished to remain silent, and
ney was coming to represent him. he voluntarily confessed af-
ter a relatively short period
B Grant the motion, because the defendant of interrogation. Hence, he
has been deprived of his Sixth Amendment validly waived his Miranda
right to counsel. rights.(C) is incorrect even
though it is true that the
C Deny the motion, because the defendant's defendant made a voluntary
statement admitting the crime was volun- statement. Due process re-
tary. quires that for confessions to
be admissible, they must be
D Deny the motion, because the defendant "voluntary," based on the to-
waived his Miranda rights. tality of the circumstances,
141 / 184
Barbri MBE questions
Study online at https://quizlet.com/_6umlok
and here all of the circum-
stances indicate that the
defendant's confession was
voluntary. However, even a
voluntary confession will be
inadmissible if it was ob-
tained in violation of Miranda
rights. (D) is therefore a bet-
ter choice than (C).

125. The plaintiff was exiting from a parking (B)


garage owned and operated by the city
when he discovered that the exit ramp was The fact least helpful to the
blocked by construction barricades and a city's defense of the plain-
pile of broken-up concrete. No workers or tiff's lawsuit is the identity
detour signs were around and the plaintiff of the workers who blocked
was in a hurry, so he backed up and drove the exit ramp. Under vicar-
down an entrance ramp that was clearly ious liability rules, a princi-
marked as such. As he came around a cor- pal will be liable for the tor-
ner, his car was broadsided by a pickup tious acts of an independent
truck. The plaintiff was seriously injured in contractor if the duty is non-
the collision. A statute in the jurisdiction re- delegable on public policy
quires drivers to obey all traffic directional grounds; included is the duty
markings in both public and private parking of a possessor of land to
lots and garages. The jurisdiction retains keep its premises safe for
governmental immunity for municipalities. its invitees. If the workers
were negligent in leaving the
If the plaintiff brings a lawsuit against the ramp blocked without provid-
city to recover for his injuries, which of the ing another means of exit-
following facts will be LEAST helpful in the ing, the fact that they were
city's defense? not city employees would
not absolve the city of li-
A The plaintiff was aware that another exit ability; hence, their identi-
on the other side of the garage was open. ty would be of no help to
the city's defense.(D) is in-
B The construction workers responsible for correct because the pick-
blocking off the exit ramp were employees up truck driver's conduct
of an independent contractor rather than the under these circumstances
city. would be deemed a super-
142 / 184
Barbri MBE questions
Study online at https://quizlet.com/_6umlok
seding force that breaks the
C The city does not collect fees or make a causal connection between
profit in the operation of the garage. any negligence on the part
of the city and the plain-
D The pickup truck driver could have avoid- tiff's injury. Assuming that
ed the plaintiff but recognized him as an old the city workers were neg-
enemy and deliberately ran into him. ligent, the fact that an in-
dependent intervening force
caused the injury generally
would not cut off the city's lia-
bility, because its negligence
created a foreseeable risk
of that harm occurring. How-
ever, where this foreseeable
harm is caused by an unfore-
seeable crime or intention-
al tort of a third party, most
courts would not hold the city
liable, treating the crime or
tort as a superseding force.
Here, while blocking the exit
ramp created a foreseeable
risk that someone might col-
lide with the plaintiff, it was
not foreseeable that his en-
emy would take that oppor-
tunity to commit an inten-
tional tort against him. Be-
cause the pickup truck dri-
ver's conduct was unfore-
seeable under the circum-
stances in choice (D), the
city would be relieved of li-
ability for any negligence in
blocking the ramp.

126. A landowner's will left his ranch to a ranch- (C)


er, his heirs, and assigns, so long as the
property was used exclusively for ranch The grandson prevails be-
143 / 184
Barbri MBE questions
Study online at https://quizlet.com/_6umlok
purposes, then to the landowner's grand- cause the ranch passed
son. The remainder of the landowner's prop- through the residuary
erty passed through the residuary clause of clause. Under the Rule
his will to the grandson. Seven years after Against Perpetuities, the at-
the landowner's death, the rancher began tempt to give the grand-
strip mining operations on the ranch. The son an executory interest
grandson brought an action to quiet title is void, so (D) is incorrect.
to the ranch against the rancher, and the However, the courts would
rancher counterclaimed on the same theory. simply read the conveyance
without the language of
Who should prevail? the executory gift, leaving
a possibility of reverter in
the grantor, the landowner.
A The rancher, because the condition im- Thus, (A) is incorrect. Be-
posed on his interest under the will is void cause the grandson suc-
as violating the Rule Against Perpetuities. ceeded to the landowner's
interest as grantor via the
B The rancher, because the condition im- residuary clause of the will,
posed is a restraint against alienation. he will prevail. (B) is incorrect
because there is no restraint
C The grandson, pursuant to the residuary on alienation contained in
clause. the will.

D The grandson, because the condition im-


posed is valid and he takes according to the
subsequent provision.

127. After a single vehicle accident, the pas- (C)


senger filed a negligence action in federal
district court against the driver to recov- The driver may not obtain
er for a whiplash injury allegedly suffered in discovery the opinions of
in the accident. On the advice of his attor- the four physicians unless he
ney, the passenger consulted and retained can demonstrate exception-
five physicians in search of one who would al circumstances. The opin-
serve as an expert witness on his behalf ions of experts who are re-
at trial. Four of the physicians determined tained in anticipation of liti-
that the passenger had suffered no injury. gation but who are not ex-
Obviously, the passenger does not intend to pected to testify at trial may
use those four physicians as witnesses at be discovered only upon a
144 / 184
Barbri MBE questions
Study online at https://quizlet.com/_6umlok
trial. showing of exceptional cir-
cumstances under which it is
May the driver obtain in discovery the opin- impracticable to obtain facts
ions of the four physicians whom the pas- or opinions by other means
senger does not intend to have testify at or when a medical report for
trial? an exam conducted under
FRCP 35 is requested. (D) is
A Yes, the passenger must disclose in his therefore incorrect. (A) is in-
required disclosures the identity of the four correct because it describes
physicians and provide a written report con- the requirements for expert
cerning their qualifications and opinions. witnesses who are expect-
ed to be used at trial. (B) is
B Yes, the driver may obtain information incorrect because the opin-
concerning their opinions through inter- ions of experts who will not
rogatories, but the driver may not depose testify at trial cannot be dis-
the four physicians. covered by any means un-
less there is a showing of ex-
C No, the driver will not be permitted to ceptional circumstances.
discover the facts known and opinions held
by these four physicians unless the dri-
ver can demonstrate exceptional circum-
stances under which it is impracticable to
obtain facts or opinions on the same sub-
ject by other means.

D No, the parties may not under any circum-


stances obtain discovery of opinions held
by experts retained by another party when
the other party does not intend to have the
expert testify at trial.

128. employees with members of the press un- (D)


less prior permission had been obtained
from a supervisor. Executive Department The President's order is un-
employees were subject to dismissal for vi- constitutional because it is
olation of the order. An employee of the Unit- too broad a limit on the free-
ed States Department of Agriculture spoke dom of speech and associ-
to a reporter and told the reporter that the ation of government employ-
USDA sanctioned the approval of a danger- ees. No government employ-
145 / 184
Barbri MBE questions
Study online at https://quizlet.com/_6umlok
ous pesticide because of payoffs made by ees were allowed to have
lobbyists to a high-ranking USDA official. even a conversation with the
The newspaper printed the story and quoted press without the permission
the employee by name. After a hearing in of a supervisor. Thus, it is
line with civil service regulations, the em- unconstitutional. (A) is incor-
ployee was found to have violated the ex- rect because the President's
ecutive order barring unauthorized conver- plenary power to control ex-
sations with news media reporters, and the ecutive employees is limit-
employee was fired. The employee brought ed by the Constitution. For
suit in federal court for reinstatement, backexample, the President can-
pay, and other benefits and also prayed that not violate the First Amend-
the court should strike down the executive ment (or other constitution-
order as unconstitutional. al guarantees) in regulating
federal employees. (B) is in-
If the employee's case reaches the Unit- correct because the Court
ed States Supreme Court, how should the no longer draws a distinc-
Court rule? tion between rights and priv-
ileges. Furthermore, govern-
A The executive order is constitutional, be- ment employees would re-
cause the President has plenary power to tain their First Amendment
control Executive Department employees. rights even if government
employment were termed a
B The executive order is constitutional, be- "privilege." (C) is incorrect
cause government employment is a privi- because the President has
lege and not a right. authority to enact some reg-
ulations for federal employ-
C The executive order is unconstitutional, ees. The problem with this
because Congress, rather than the Presi- regulation is that it unduly re-
dent, has authority to set the terms of fed- stricts freedom of speech.
eral employment.

D The executive order is unconstitutional,


because the President cannot broadly limit
all executive employees' freedom of speech
and association.

129. While fleeing from an armed robbery he had (B)


just committed, a man struck a pedestrian
with his car, seriously injuring the pedestri- The robber's claim is correct
146 / 184
Barbri MBE questions
Study online at https://quizlet.com/_6umlok
an. The robber was soon apprehended and because the victim died be-
charged with armed robbery and reckless fore jeopardy attached for tri-
driving, both felonies. Just prior to trial, the
al on the lesser included of-
pedestrian died from his injuries. The trial fense. The Fifth Amendment
on the robbery and driving charges pro- right to be free of double
ceeded, and the robber was convicted of jeopardy provides that once
the armed robbery charge and acquitted of jeopardy attaches for an of-
the reckless driving charge. The robber was fense, the defendant may
then indicted under the jurisdiction's felonynot be retried for the same
murder statute for causing the death of the offense. Under the Block-
pedestrian during the course of committing burger test, two crimes do
an armed robbery. The robber moved to dis- not constitute the same of-
miss the indictment on the ground that a fense if each crime requires
second trial would violate double jeopardy. proof of an additional ele-
ment that the other crime
Is the robber's claim correct? does not require. Under this
test, a lesser included of-
A Yes, because he was acquitted of the reck- fense and the greater of-
less driving charge. fense would be considered
the "same offense," because
B Yes, because the pedestrian died before the lesser included offense
the robber's first trial had begun. consists entirely of some,
but not all, elements of the
C No, because he was convicted of the greater crime. Hence, under
armed robbery charge. double jeopardy rules, at-
tachment of jeopardy for the
D No, because felony murder requires proof greater offense bars retrial
of an additional element not required by the for lesser included offens-
felony itself. es, and attachment of jeop-
ardy for a lesser included of-
fense generally bars retrial
for the greater offense. An
exception to this latter rule
exists if all of the elements
for the greater offense had
not occurred at the time of
prosecution for the lesser of-
fense, but in this case the
final element for the felony
147 / 184
Barbri MBE questions
Study online at https://quizlet.com/_6umlok
murder charge—the death
of the victim—occurred be-
fore jeopardy had attached
in the first trial, so the pros-
ecution could have added
a charge of felony murder
prior to proceeding with the
first trial. Thus, the under-
lying felony of armed rob-
bery was a lesser included
offense of the felony mur-
der and the robber's being
placed in jeopardy for it bars
the subsequent trial for the
felony murder. (D) is incor-
rect because, as discussed
above, two crimes are not
the "same offense" for dou-
ble jeopardy purposes only
if each crime requires proof
of an additional element that
the other crime does not re-
quire. Because the underly-
ing felony is a lesser includ-
ed offense of the felony mur-
der charge, i.e., it has no
other elements not required
by the felony murder charge,
it constitutes the "same of-
fense" for purposes of dou-
ble jeopardy.

130. served several alcoholic drinks by a bar- (B)


tender at a local bar. The pedestrian sued
the bartender in a jurisdiction that does not Because the jurisdiction
have a dramshop act. does not have a dramshop
statute, the bartender will
Is the bartender vicariously liable for the not be liable for the injuries
pedestrian's injuries? caused to the pedestrian
148 / 184
Barbri MBE questions
Study online at https://quizlet.com/_6umlok
by the intoxicated driver. At
common law, no liability was
A No, because the driver acted recklessly by imposed on vendors of intox-
driving while intoxicated. icating beverages for injuries
resulting from the vendee's
B No, because there is no dramshop act in intoxication, whether the in-
the jurisdiction to impose liability. juries were sustained by the
vendee or by a third person
C Yes, because there is no dramshop act in as a result of the vendee's
the jurisdiction to limit liability. conduct. Many states, in
order to avoid this com-
D Yes, because the intoxicated driver mon law rule, have enact-
caused the pedestrian to suffer personal in- ed "dramshop acts." Such
juries acts create a cause of action
in favor of any third person
injured by the intoxicated
vendee. Without a dramshop
act, the bartender will not be
vicariously liable.

131. A town with a population of 30,000 merged (D)


with a city of 60,000. To protect voting rights
of the citizens of the former town, a propos- The Supreme Court may
al was made that for a period of 20 years, grant certiorari to review a
beginning at the date of the merger, the city case from the highest court
council of the merged city would consist of in a state that can render
six persons. Each formerly separate munic- an opinion on the matter if
ipality would be divided into three council a state statute's validity is
districts. Each district from the former town called into question under
would have approximately 10,000 residents, the federal Constitution. [28
and each district from the former city would U.S.C. ý1257] The Court may
have 20,000 residents. A mayor would be decide the federal issues,
elected at large. Before this proposal was but cannot rule on the state
placed on the ballot, the state attorney gen- law issues. (A) is incorrect
eral issued an advisory opinion stating that for several reasons: (i) the
the proposal was not in violation of any attorney general evaluated
state statutory or constitutional provision. only the proposal's validity
The proposal was placed on the ballot and under the state constitution;
was carried by large majorities in both the and (ii) even if her opinion
149 / 184
Barbri MBE questions
Study online at https://quizlet.com/_6umlok
town and the city, and the districts were had addressed the propos-
carved out. al's federal constitutional va-
lidity, the Supreme Court is
Three taxpayers filed suit to enjoin the not bound by advisory opin-
holding of an election with council dis- ions of state attorneys gen-
tricts of such disparate proportions. The eral. (B) is incorrect even
suit reached the state supreme court, which though the state supreme
ruled that the governmental formula was court may have had an in-
constitutional under both the state and dependent state ground for
United States Constitutions. The plaintiffs finding the law constitution-
wish to take the case to the United States al under its state consti-
Supreme Court. tution. The Supreme Court
will refuse to hear the case
How should the Supreme Court proceed? only if the state ground is
adequate by itself to sup-
A Rely on the attorney general's opinion and port the decision as well
not hear the case on its merits. as independent, so that the
Court's review of the feder-
B Not hear the case, because it was decided al ground for the decision
below on an independent state ground. would have no effect on the
outcome of the case (such
C Not hear the case, but remand it to federal as if the state court had
district court. found the law invalid under
both the state and feder-
D Hear the federal issues involved, but de- al Constitutions). Here, the
cline to rule on state issues. Supreme Court's review of
the state court opinion on
the law's federal constitu-
tional status may have an
outcome on the case regard-
less of the state court's de-
cision on the state constitu-
tional issue; the Court there-
fore will hear the federal is-
sues involved. (C) is incor-
rect because 28 U.S.C. sec-
tion 1257 provides that ap-
pellate review of a matter
from a state's highest court
150 / 184
Barbri MBE questions
Study online at https://quizlet.com/_6umlok
is to the Supreme Court by
petition for a writ of certiorari,
rather than to a federal dis-
trict court.

132. A lawyer sued a client for his fee, based (A)


on an agreed hourly rate. The client sub-
poenaed the lawyer's time records for the The subpoena should be up-
days on which he purported to have worked held because the informa-
for the client to show that the lawyer had tion about hours billed is not
billed an impossible number of hours to within the privilege. There
the client and others on those days. The is no privilege regarding a
client's subpoena provided that any infor- communication that is rele-
mation concerning the matters handled for vant to an issue of breach
other clients be deleted or masked. The of duty by the lawyer to his
lawyer moved to quash the subpoena on the client or by the client to her
ground of attorney-client privilege. lawyer. Thus, the billing data
does not fall within the am-
Should the subpoena be upheld? bit of the privilege. (B) is in-
correct because the informa-
A Yes, because the information about hours tion here is not covered by
billed is not within the privilege. the privilege. Furthermore,
if the privilege were applic-
B Yes, because a lawyer has no right to in- able, the lawyer would be
voke his client's privilege without instruc- able to invoke the privilege
tions from the client. on behalf of his clients. The
lawyer's authority to do this
C No, because a lawyer is entitled to a right is presumed in the absence
of privacy for the work product in his files. of any evidence to the con-
trary. (C) is a correct state-
D No, because no permission was obtained ment of law, but the work
from the other clients to divulge information product rule does not apply
from their files. to these facts. Documents
prepared by the lawyer for
his own use in prosecuting
his client's case are protect-
ed by this rule. Time records
are not prepared for litigation
purposes; they are not relat-
151 / 184
Barbri MBE questions
Study online at https://quizlet.com/_6umlok
ed to the substance of the
client's case. (D) is incorrect
because the other clients are
protected by the blacking out
of confidential information.
Moreover, because the time
records are not communica-
tions to or from the client and
the identity of clients is often
not considered to be with-
in the privilege, the billing
records of other clients may
not be privileged and their
consent may not have been
necessary even without the
deletions.

133. A state statute provided for criminal penal- (A)


ties for "knowingly selling alcoholic bever-
ages in violation of the regulations of the The bartender's reasonable
State Liquor Commission to any person un- belief that the minor is 25
der the age of 18." One of the State Liquor years old is a mistake of
Commission regulations provided that "be- fact that negates the state of
fore an alcoholic beverage is sold to any mind required by the statute.
person between the ages of 17 and 24, the Ignorance or mistake as to a
seller must demand some form of photo matter of fact will affect crim-
identification to determine the buyer's age."inal guilt only if it shows that
the defendant did not have
A minor who looked much older than his the state of mind required
age of 17 walked into a tavern located in for the crime. In addition, the
the state and asked the bartender for a beer. mistake must be reasonable
The bartender never asked the minor for unless the offense is a spe-
any form of identification, as he thought cific intent crime. Here, the
that he was at least 25 years old. Had the statute requires that the de-
bartender asked for identification, the minor fendant have acted "know-
would have shown him a fake identification ingly" with respect to each
card showing that he was 21 years old. The of the material elements of
bartender served the beer to the minor, who the offense. A person acts
consumed it on the premises. The bartender knowingly with respect to the
152 / 184
Barbri MBE questions
Study online at https://quizlet.com/_6umlok
was subsequently charged under the state nature of his conduct when
statute for selling the beer to the minor. he is aware that his conduct
is of that nature or that cer-
Is the bartender guilty? tain circumstances exist. At
least one of the material el-
A No, because he reasonably believed that ements of the offense here
the minor was older than 25 years. is that the sale be to a per-
son under the age of 18. If
B No, because the minor had fake identifica- the bartender believed that
tion with which he could have obtained the the minor was 25 years old,
beer. the bartender has not act-
ed knowingly with respect to
C Yes, because he sold an alcoholic bever- the fact that the purchaser
age to a minor, a strict liability crime. was under 18, and he can-
not be convicted of violat-
D Yes, because he failed to ask for identifi- ing the statute.(D) is not as
cation, and the regulation does not provide good an answer as (A). Al-
for a mens rea requirement. though the regulation appar-
ently does not have a state
of mind requirement, it is
not entirely clear that crim-
inal liability can result from
the violation of the regula-
tion. There would have to be
a separate statute providing
for criminal penalties for fail-
ing to check for identifica-
tion. Furthermore, the crimi-
nal statute in question clear-
ly has a state of mind re-
quirement that must be sat-
isfied, as explained above.

134. At trial, questions have been raised as to (B)


whether the proposed testimony of the wit-
ness is relevant and whether it falls within The judge determines
the present sense impression exception to whether the testimony falls
the hearsay rule. within an exception to the
hearsay rule, and is gen-
153 / 184
Barbri MBE questions
Study online at https://quizlet.com/_6umlok
How should a preliminary determination of erally not limited by the
the admissibility of the witness's testimony rules of evidence in mak-
be made? ing that determination. The
Federal Rules of Evidence
distinguish between prelim-
A A judge should determine whether the inary facts to be decided
proposed testimony falls within the excep- by the jury, which involve
tion before it is heard by the jury, and in whether the proffered evi-
making that determination she is limited by dence is relevant, and pre-
the rules of evidence. liminary facts decided by the
judge, which involve whether
B A judge should decide whether the tes- the evidence is competent,
timony falls within the present sense im- i.e., not barred by an exclu-
pression exception, but in making that de- sionary rule. All preliminary
termination she is not limited by the rules of fact questions that deter-
evidence other than privilege. mine the applicability of an
exception to the hearsay rule
C The jury, after being instructed on the must be determined by the
rules of evidence by a judge, should de- judge, because the compe-
termine whether the testimony falls within tency of the evidence will de-
the scope of the present sense impression pend on that preliminary fact
exception. determination. In making this
preliminary fact determina-
D The jury should determine whether the tion, the trial court may con-
testimony falls within the scope of the ex- sider any nonprivileged rele-
ception and the judge should then instruct vant evidence, even though
the jury on the appropriate uses for that it would not otherwise be
evidence. admissible under the rules
of evidence. [Fed. R. Evid.
104(a)] In this case, then, the
judge should decide whether
the testimony falls within the
present sense impression
exception, and she is not lim-
ited in making this determi-
nation by the rules of evi-
dence other than privilege.

135.
154 / 184
Barbri MBE questions
Study online at https://quizlet.com/_6umlok
A plaintiff was injured in an automobile ac- (C)
cident caused by the defendant. The plain-
tiff sued the defendant for his injuries. In The testimony should be ex-
preparation for trial, the plaintiff's attorney cluded because the attor-
hired a doctor to examine the plaintiff. At ney-client privilege applies
trial, the defense attorney attempts to call to the examination done
the doctor as a witness to testify about in preparation for trial. The
statements the plaintiff made in confidence communication between the
to the doctor about his injuries, which the doctor and the attorney's
doctor then communicated to the plaintiff's client is necessary to help
attorney. The state recognizes only the com- the client convey his con-
mon law privileges. dition to the attorney. (A)
is incorrect because admis-
Should this testimony be admitted? sions by party-opponents,
while not hearsay under the
A Yes, because the plaintiff's statements are Federal Rules, are still sub-
the statements of a party-opponent. ject to potential privilege as-
sertions. (B) is a true state-
B Yes, because the plaintiff waived the ment; the physician-patient
physician-patient privilege by placing his privilege does not apply to
physical condition in issue. any proceeding in which the
condition of the patient has
C No, because the plaintiff's statements are been put in issue by the
protected by the attorney-client privilege. patient. This is the case in
the plaintiff's suit, so (D)
D No, because the plaintiff's statements are is incorrect. However, (B)
protected by the physician-patient privilege. is incorrect because when
a client is examined by a
doctor at the attorney's re-
quest, the communications
involved between the client
and doctor (and the doctor
and attorney) are not cov-
ered by the physician-pa-
tient privilege because no
treatment is contemplated.
Moreover, the physician-pa-
tient privilege is a statuto-
ry privilege, and this juris-
155 / 184
Barbri MBE questions
Study online at https://quizlet.com/_6umlok
diction recognizes only the
common law privileges (e.g.,
the attorney-client privilege).

136. A landowner owns 15 acres of undeveloped (C)


property. He plans to build a stadium com-
plex on the property to house a football The best way for the
team two years from now, but would like to landowner to accomplish his
open the 15 acres to public use for picnick-
goals is to grant the city
ing and similar activities until then. an easement for recreational
use for two years. An ease-
Which of the following would best accom- ment would allow the city
plish the landowner's goal? to use the land only for the
purposes provided for in the
A Dedicate the 15 acres for use as a public easement, and the landown-
park. er could limit the purposes to
recreational uses. Thus, (C)
B Lease the 15 acres to the city for two is the best answer. (A) would
years. not be a good choice be-
cause if the landowner dedi-
C Grant the city an easement for public cated the land to public use,
recreational uses for two years. he would be giving title to
the land to the government,
D Covenant that the city may use the 15 so he would not be able to
acres for recreation for two years. reclaim the land and build
his stadium in the future. (B)
would not be a good choice
because a lease would give
the city more control over the
land than an easement, and
would be more complicated
to create. A lease grants the
lessee the exclusive right to
possess the premises, and
broad rights to use them in
any manner, unless specifi-
cally restricted. Thus, if the
landowner leases the land to
the city, he would not have
156 / 184
Barbri MBE questions
Study online at https://quizlet.com/_6umlok
access to the land, and if he
wanted it used only for recre-
ational purposes, he would
have to specifically restrict
any undesired uses. Any re-
striction not included in the
lease will be unenforceable.
An easement, on the other
hand, grants only a limited
interest in the land—to use it
for only those purposes stat-
ed in the easement; thus, it
would be better than a lease.
(D) is not a good choice be-
cause covenants usually are
made in conjunction with a
lease, deed, or other instru-
ment; they promise some act
or forbearance with respect
to property and are generally
not used to grant rights for
access to property.

137. A man purchased a large flat screen televi- (B)


sion and decided to mount it on the ceiling
over his bed. The manual that came with the Knowledge on the part of the
product included detailed instructions and manufacturer that its televi-
illustrations on how to mount the television sion was being mounted on
on different types of walls, along with all the the ceiling would give rise
required hardware, but contained neither in- to a duty to include in the
structions nor warnings regarding mount- manual warnings against the
ing on the ceiling. The man carefully fol- practice or detailed instruc-
lowed the wall-mounting instructions and tions on how to safely mount
was satisfied that it would hold. In fact, how- it. The television hardware
ever, the mounting was not appropriate for and instructions were appro-
ceilings. The next night, a woman who was priate for its intended mount-
the man's overnight guest was seriously in- ing on the wall. However,
jured when the television came loose and courts in a strict liability case
fell on the bed. require a commercial sup-
157 / 184
Barbri MBE questions
Study online at https://quizlet.com/_6umlok
plier to anticipate reason-
Will the woman prevail in a suit against the ably foreseeable uses even if
company that manufactured the television? they are misuses of the prod-
uct. If the manufacturer knew
A Yes, because the manufacturer had a duty that members of the public
to include warnings for all potential place- were sometimes mounting
ments of its product. the television on the ceiling,
marketing the product with-
B Yes, if the manufacturer knew that its tele- out including either warnings
vision was sometimes mounted on ceilings against the practice or ap-
rather than walls. propriate hardware and in-
structions on how to safe-
C No, if the manufacturer's manual had all ly do so made the prod-
of the customary warnings for this type of uct so defective as to be
product. unreasonably dangerous if
it were improperly mounted.
D No, because the man was negligent in Under a strict liability the-
mounting the television on the ceiling. ory, the manufacturer is li-
able for supplying a defec-
tive product. As a guest of
a purchaser of the product,
the woman is a foreseeable
plaintiff; thus, the manufac-
turer may be liable to her.
The defective product actu-
ally and proximately caused
the woman to suffer seri-
ous injuries. Therefore, the
manufacturer is liable to the
woman in a strict products li-
ability action. (A) is incorrect
because the facts do not es-
tablish that the manufacturer
was under a duty to include
the warnings in its manu-
al. Such a duty would ex-
ist if the manufacturer knew
(as (B) states) or should
have known that the televi-
158 / 184
Barbri MBE questions
Study online at https://quizlet.com/_6umlok
sion was being mounted on
ceilings.

138. A plaintiff sued a defendant for negligence, (C)


seeking $100,000 in the federal court of
State A, after the two were involved in a car The defendant may not bring
accident while driving through State B. The a negligence claim against
defendant's defense was that the plaintiff the plaintiff. This question
was contributorily negligent; contributory asks not only about preclu-
negligence is a complete defense under the sion, but about the relat-
applicable state law. The plaintiff won after ed concept of compulsory
a full trial. counterclaims. If a counter-
claim arises out of the same
May the defendant now bring a negligence transaction or occurrence as
claim against the plaintiff in a new suit for in- one of the plaintiff's claims,
juries from the accident that was previously it is a compulsory counter-
litigated? claim and must be plead-
ed or it will be barred. Any
A Yes, because the defendant's claim is not other counterclaim is per-
the same as that of the plaintiff. missive and may be as-
serted (assuming there is
B Yes, because claim preclusion (res judica- subject matter jurisdiction)
ta) applies only if the claims are brought by even though there is no
the same parties. connection at all between
it and the plaintiff's claim.
C No, because the counterclaim is barred as Although the defendant's
an unasserted compulsory counterclaim. compulsory counterclaim is
not claim precluded under
D No, because the defendant's claim preclusion principles (as dis-
merged with the prior judgment and is now cussed below), it is barred
barred. as an unasserted compul-
sory counterclaim. (A) and
(B) are incorrect because,
while they correctly note that
there is no claim preclusion
for claims against the defen-
dant, that is not the only is-
sue, as discussed above. (D)
is incorrect because claims
159 / 184
Barbri MBE questions
Study online at https://quizlet.com/_6umlok
by the opposing party can-
not be merged. Once a fi-
nal judgment on the merits
is reached on a claim, the
claimant is barred by claim
preclusion (also known as
res judicata) from asserting
the same claim in a subse-
quent suit against the same
defendant. In this case, there
is a final judgment on the
plaintiff's negligence claim,
which would bar the plain-
tiff from asserting any other
claims relating to the same
facts in a subsequent suit
against the same defendant.
Any such claims are consid-
ered to be "merged" with the
adjudicated claim and thus
bar the plaintiff from prop-
erly asserting them. Howev-
er, this bar is only against
the plaintiff's claim. Claim
preclusion does not bar a
claim by an opposing party.

139. A buyer entered into a contract with a seller (D)


to purchase the seller's farm. The contract
of sale referred to the farm as containing This answer states the tradi-
250 acres. The agreed-on price was $1 mil- tional rule where the amount
lion. Before the date on which escrow was of land in a land sale con-
to close, the buyer learned from a survey- tract is less than as agreed.
or he had hired that the farm actually con- When a buyer has a rem-
tained 248 acres. On the date the sale was to edy of specific performance
close, the buyer instructed the escrow agent in a land sale contract, a
to release all but $8,000 of the purchase court of equity will order a
money because he was not getting what he seller to convey the title if
bargained for. The seller refused to proceed the buyer tenders the pur-
160 / 184
Barbri MBE questions
Study online at https://quizlet.com/_6umlok
with the sale. The buyer brings an action chase price. If the seller can-
for specific performance and also seeks an not provide marketable title
$8,000 reduction of the agreed-upon con- under the terms of the con-
tract price. tract, but the buyer wishes
to proceed with the trans-
What will be the probable outcome of the action, the buyer can usual-
litigation? ly get specific performance
with an abatement of the
A The seller will win, because the buyer re- purchase price in an amount
fused to tender the contract price when the reflecting the title defect. A
seller tendered substantially what the con- defect as to the quantity of
tract called for her to perform. land conveyed is usually cor-
rected by a pro rata abate-
B The seller will win, because both parties ment of the price. (D) states
had seen the farm before the contract was the factors that a court of eq-
formed. uity will look for when decid-
ing whether to grant specif-
C The buyer will win, because he is not ic performance with abate-
receiving what he bargained for under the ment.A) is incorrect because
contract. the parties' contract did not
merely refer to the farm as
D The buyer will win, if the court finds that a named parcel of land;
the $8,000 reduction in price is a fair reflec- it recited that it contained
tion of the title defect. 250 acres. Based on this
recital, a court could readily
conclude that the difference
of two acres is a material
change in the terms of the
contract and that the seller's
tender of 248 acres was not
substantial performance. (B)
is incorrect because viewing
the property did not put the
buyer on notice as to the dis-
crepancy; the buyer is not
required to visually calculate
the amount of acreage a par-
cel of land contains.

161 / 184
Barbri MBE questions
Study online at https://quizlet.com/_6umlok
140. A township located in a farming commu- (D)
nity was composed mostly of persons be-
longing to a specific religious sect. To help The punishment here is con-
instill proper respect for authority in chil- stitutional because it does
dren, which was a central tenet of the sect, not violate any constitu-
and to maintain order in the classroom, the tional provision. The best
local school board allowed teachers to in- answer reflecting this rea-
flict corporal punishment. Such punishment soning is (D)—there was
was inflicted on a fourth grader in a town- no Eighth Amendment vio-
ship school immediately after his teacher lation here—because pad-
saw him pulling a girl's hair. Neither he nor dling students as a discipli-
his parents belonged to the religious sect. nary measure has not been
When the boy's parents learned of the in- found to be cruel and un-
cident, they hired an attorney. Rather than usual punishment. (A) is in-
suing the teacher for battery as permitted correct because there is no
under state law, the attorney brought an Establishment Clause viola-
action against the teacher under a feder- tion here. Under the Estab-
al statute providing a cause of action for lishment Clause, if there is
damages against any government employee no sect preference, govern-
who deprives a person of his constitutional ment action generally will be
rights. upheld if the action serves
a secular purpose, its prima-
Should the court find the policy allowing ry effect neither advances
corporal punishment to be constitutional? nor inhibits religion, and it
does not excessively en-
A No, because the punishment policy vio- tangle government with reli-
lates the First Amendment Establishment gion. There is no sect pref-
Clause. erence under the school
board's corporal punishment
B No, because the boy was denied any kind rule here, the rule has the
of hearing, in violation of his right to pro- secular purpose of main-
cedural due process under the Fourteenth taining order in the class-
Amendment. room (the fact that this co-
incides with the tenets of
C Yes, because under the doctrine of parens a local religion does not
patriae states may impose any punishment change that conclusion), its
they see fit. main purpose neither ad-
vances nor inhibits religion,
D Yes, because the punishment was not and there is no excessive
162 / 184
Barbri MBE questions
Study online at https://quizlet.com/_6umlok
grossly disproportionate under the Eighth entanglement. (B) is incor-
and Fourteenth Amendments. rect because there has been
no deprivation of procedural
due process. The Supreme
Court has held that although
corporal punishment may in-
volve a liberty interest, no
hearing is required prior to
inflicting such punishment;
the possibility of a common
law action in tort is suffi-
cient procedural protection.
[Ingraham v. Wright (1977)]
(C) is incorrect because it
is too broad. The doctrine
of parens patriae allows the
state to stand in the shoes of
a parent, but even a parent
may not impose any punish-
ment he sees fit (e.g., a par-
ent may not break a child's
arm as punishment for steal-
ing).

141. A feed store owner agreed to purchase (B)


several tons of grain products at a spec-
ified price from a large supplier of cattle The owner will have to show
feeds. The supplier later failed to deliver the that he would have recov-
promised grains, and the owner was forced ered damages in his law-
to cover by purchasing from local producers suit. The following elements
at a higher price. The owner contacted a must be proved for a prima
large law firm in the city and obtained their facie case of negligence: (i)
agreement to represent him in connection the existence of a duty on
with his possible claims against the suppli- the part of the defendant to
er. Due to error, the applicable statute of lim- conform to a specific stan-
itations period passed without the filing of dard of conduct for the pro-
any action on the owner's behalf. The owner tection of the plaintiff against
retained another lawyer and sued the large unreasonable risk of injury,
law firm for malpractice. The jurisdiction re- (ii) breach of that duty by
163 / 184
Barbri MBE questions
Study online at https://quizlet.com/_6umlok
tains traditional contributory negligence. the defendant, (iii) that the
breach of duty was the ac-
In addition to the firm's negligence, what tual and proximate cause of
else does the owner have to establish as the plaintiff's injury, and (iv)
part of his prima facie case? damage to the plaintiff's per-
son or property. Here, the
A He had a good faith claim against the owner can establish that the
supplier that was lost by the law firm's dila- law firm breached its pro-
toriness. fessional duty of care by
failing to file a claim with-
B He would have recovered from the suppli- in the statute of limitations.
er if an action had been timely filed. He must also establish that
this breach was an actual
C He did not contribute to the failure to time- and proximate cause of his
ly file an action through his own negligence. damages, which here would
be the loss of the contract
D The losses resulting from breach of the damages that he could have
sales agreement by the supplier severely recovered from the breach
harmed his financial situation. by the supplier. (A) is incor-
rect because merely having
a good faith claim that was
lost because of the firm's
negligence is not sufficient.
The owner has to show by
a preponderance of the evi-
dence that he suffered dam-
ages because of the firm's
negligence.

142. A consumer purchased a new television (B)


set from an electronics store. When he got
home, he opened the box and found an The consumer will most like-
owner's manual that contained operation in-ly recover for breach of
structions, warnings regarding the danger the implied warranty of mer-
of electricity, and a warranty that stated:chantability. In every sale
of goods, unless express-
"The store expressly warrants that this set ly disclaimed, there arises
shall be free of manufacturing defects for a warranty that the goods
30 days. If a set is defective, the store's will be merchantable, which
164 / 184
Barbri MBE questions
Study online at https://quizlet.com/_6umlok
liability shall be limited to the cost of re-means that they will be fit
pair or replacement of defective parts. The for the ordinary purposes for
store "HEREBY DISCLAIMS ANY AND ALL which such goods are used.
OTHER WARRANTIES, EXPRESS OR IM- A television that explodes af-
PLIED, INCLUDING THE WARRANTY OF FIT- ter five weeks of use like-
NESS FOR PARTICULAR PURPOSE AND ly breaches this warranty.
THE WARRANTY OF MERCHANTABILITY."" The warranty will not be dis-
claimed because, to be ef-
Five weeks later, after the set was properly fective, a disclaimer must be
installed, the consumer turned on the set, part of the offer and ac-
heard a crackling noise, and watched as his ceptance process or must
television exploded and was destroyed. be agreed to by the buyer
as a modification. Here, the
Under which of the following theories will "disclaimer" was in the box,
the consumer most likely recover? and the consumer did noth-
ing to accept the disclaimer.
A Breach of express warranty. Thus, (B) is correct. (A) is
wrong because no express
B Breach of the implied warranty of mer- warranty was made as to
chantability. quality at a time sufficient
to become part of the bar-
C Breach of the implied warranty of fitness gain. An express warranty
for a particular purpose. will arise from any affirma-
tion of fact or promise made
D Breach of the warranty of reasonable by the seller to the buyer, or
workmanship. from any description of the
goods, and any sample or
model if the statement, de-
scription, sample, or model
is part of the basis of the
bargain. Here, there was no
such affirmation, promise,
description, sample, or mod-
el indicating that the televi-
sion would not explode; that
is something that would be
understood (implied). (C) is
wrong because an implied
warranty of fitness for a par-
165 / 184
Barbri MBE questions
Study online at https://quizlet.com/_6umlok
ticular purpose arises when
(i) the seller had reason to
know the particular purpose
for which the goods are to
be used and that the buyer
is relying on the seller's skill
and judgment to select suit-
able goods; and (ii) the buyer
in fact relies on the seller's
skill or judgment. The facts
in this case do not point to
such a warranty; on the con-
trary, the consumer selected
the television himself. (D) is
wrong because the warranty
of reasonable workmanship
is a common law doctrine
generally applied to builders.
This is a contract for the sale
of goods and is governed by
the UCC, not common law.

143. At a victory party after a hard-fought elec- (D)


tion, the campaign director consumed sev-
eral drinks. A campaign worker who had Evidence of prior consensu-
also been drinking took the director to her al sexual relations between
hotel room for a nightcap. They later had in- the director and the work-
tercourse. The worker filed a complaint with er is most likely admissi-
the police, claiming that the director had ble. Although Federal Rule
intercourse with her against her will, and the 412 generally excludes evi-
director was charged with rape. dence of an alleged victim's
sexual behavior, evidence of
Which of the following is most likely to be specific instances of sexual
admitted in the director's defense? conduct between the alleged
victim and the accused may
A The worker has a reputation in the com- be admitted to show con-
munity as being sexually promiscuous. sent. Thus, if the director
raises consent as a defense
B Since the incident occurred, the worker to the rape charge, evidence
166 / 184
Barbri MBE questions
Study online at https://quizlet.com/_6umlok
has had sexual intercourse with two other of his previous consensu-
campaign workers. al sexual encounters with
the worker is admissible. (A)
C Two years ago during the candidate's pre- is incorrect because it pre-
vious campaign, the worker maintained a sents evidence that Feder-
sexual relationship with the former cam- al Rule 412 specifically in-
paign director. tends to exclude; i.e., evi-
dence of the alleged victim's
D The director and the worker had had con- sexual behavior. The Feder-
sensual sex on two prior occasions. al Rules also contain an ex-
ception for specific instances
of the alleged victim's sexu-
al conduct tending to show
that someone other than the
accused was the source of
semen, injury, or other phys-
ical evidence. (B) does not
fit this exception because
the worker had intercourse
with two other workers after
the incident with the direc-
tor occurred. Likewise, (C) is
not likely to fit this exception
as the worker's relationship
with the previous campaign
director occurred two years
ago, and is therefore proba-
bly not helpful in explaining
the presence of physical ev-
idence.

144. A 13-year-old boy who lived on a farm with (B)


his parents in a rural area had learned to
drive the family's tractor when he was 11. The most correct statement
A state statute permitted persons without a is that the trier of fact should
driver's license to operate farm vehicles on take into account the boy's
public roads for short distances. One morn- experience when consider-
ing the boy took the tractor onto a public ing the applicable standard
road to reach one of the outlying fields a of care. Regardless of the
167 / 184
Barbri MBE questions
Study online at https://quizlet.com/_6umlok
few hundred yards away. As he neared the specific standard of care that
field he was distracted by a girl riding by is applied, someone with
on a bicycle, and cut in front of a milk de- knowledge superior to that
livery truck that was starting to pass him. of the average person is re-
The truck swerved off the road, injuring the quired to use that knowl-
driver. edge. Hence, the trier of fact
should take into account the
If the driver sues the boy to recover dam- fact that the boy had driven
ages for his injuries, which of the following a tractor since he was 11
statements is most correct regarding the years old. (A) is incorrect.
standard of care to be applied? The precise standard of care
in a common law negligence
A The state statute replaces the general case may be established by
common law standard of care with a statu- proving the applicability to
tory standard. that case of a statute pro-
viding for criminal penalties,
B The trier of fact should take into account so that the statute's specif-
the boy's experience at driving a tractor ic duty will replace the more
when considering the applicable standard general common law duty
of care. of due care, and a viola-
tion of the statute will estab-
C Persons 13 years of age or older are held lish duty and breach of duty.
to the same standard as adults. Here, nothing in the facts in-
dicates that the boy violated
D An adult standard of care will not be ap- any provisions in the statute,
plied because it is common in that region for but he still may be liable to
children of that age to be operating tractors. the truck driver for breach of
a general duty of care. (C)
is incorrect. The usual stan-
dard of conduct to which a
child must conform is that
of a child of like age, edu-
cation, intelligence, and ex-
perience. While a child must
conform to an adult stan-
dard of care when engag-
ing in a potentially danger-
ous activity in which usual-
ly only adults engage, there
168 / 184
Barbri MBE questions
Study online at https://quizlet.com/_6umlok
is no blanket rule that chil-
dren 13 years of age or older
are held to the same stan-
dard of care as adults. (D)
is incorrect because the fact
that 13-year-olds commonly
drive tractors in that region
does not preclude the court
from applying an adult stan-
dard of care when a tractor
is driven on a public road.

145. An acquaintance asked the defendant to (A)


give him a lift downtown because he did
not have bus fare. While riding on the defen-
The defendant can be found
dant's motorcycle, the acquaintance asked guilty of robbery and felony
to stop at a convenience store to get a bottle
murder, but the death penal-
of wine, showing the defendant a tire iron ty cannot be imposed. The
in his backpack that he was going to use. defendant can be found
The defendant nodded in acknowledgment guilty of robbery as an
of what the acquaintance was planning to accomplice. The Supreme
do. The defendant stopped at the store and Court has held that, under
waited in the parking lot while the acquain-the Eighth Amendment, the
tance went in. He demanded money from death penalty may not be
the clerk, brandishing the tire iron. The clerk
imposed for felony murder
tried to grab a gun under the counter while where the defendant, as an
he was filling a bag with money, and a strug-
accomplice, did not take or
gle ensued. The gun discharged, killing the attempt or intend to take life,
clerk. The defendant heard the gunshot and or intend that lethal force be
raced off, but was eventually apprehended. employed. [Enmund v. Flori-
da (1982)] Here, because
The jurisdiction's criminal code provides the defendant's involvement
that a death caused during the commis- in the crime was only to pro-
sion of certain felonies, including robbery, vide transportation, it can-
is first degree felony murder, for which the not be said that he par-
death penalty is permitted. The code also ticipated in such a major
permits cumulative penalties for first de- way that he acted with reck-
gree felony murder and for the underly- less indifference to human
ing felony. The defendant was charged and life; hence, the death penal-
169 / 184
Barbri MBE questions
Study online at https://quizlet.com/_6umlok
convicted of both robbery and felony mur- ty cannot constitutionally be
der. After appropriate consideration of all imposed against him.
relevant circumstances, the jury imposed
the death penalty. On appeal, the defendant
challenged both the convictions and the
sentence.

Assuming that the above facts were prop-


erly admitted into evidence, how should the
appellate court rule?

A The defendant's conviction for both of-


fenses should be upheld, but imposition of
the death penalty was not proper.

B The defendant's conviction for both of-


fenses should be upheld, and imposition of
the death penalty was proper.

C The defendant's conviction should be


overturned under double jeopardy princi-
ples because robbery is a lesser included
offense of felony murder.

D The defendant's conviction for felony mur-


der should be overturned because the cir-
cumstances do not establish the necessary
degree of culpability.

146. In a drug raid, police in a city searched 25 (B)


apartments selected at random in a 300-unit
housing project. In a class action approved The tenant may opt out if the
by the federal court, the 25 tenants sued court allows class members
the city for violation of their constitution- a second opportunity to opt
al rights. The tenant named as class repre- out. In a "common question"
sentative gave notice to all unnamed class class action, a judge may
members, including another tenant who de- refuse to approve a settle-
cided not to opt out. The class action was ment of a class action unless
then certified as a "common question" type. the class members are given
170 / 184
Barbri MBE questions
Study online at https://quizlet.com/_6umlok
After negotiating with the class represen- a second opportunity to opt
tative, the city police agreed to pay each out. [Fed. R. Civ. P. 23(e)] (A)
tenant $500 and to conduct no further raids is incorrect because there
without proper warrants. The representa- are only two opt-out periods,
tive and the city signed a settlement agree- with the initial notice and
ment and a stipulation of dismissal of the with the settlement agree-
class action. The other tenant objects to the ment, and the latter only ap-
amount of damages he is to receive and plies if the judge requires it.
would rather opt out now and proceed on (C) is incorrect and reflects a
his own. prior version of Rule 23. (D)
is incorrect for the reasons
May the tenant opt out now? stated above.

A Yes, because a class member of a "com-


mon question" class action always has the
right to opt out.

B Yes, if the court allows the tenant a second


opportunity to opt out.

C No, because the tenant did not opt out


after receiving notice of the class action.

D No, if class certification was proper.

147. A plaintiff sued a defendant over a claimed (A)


debt. At the trial, the plaintiff established the
existence of the debt and testified that he The burden of persuasion
never received payment. In response, the and the burden of going for-
defendant presents evidence sufficient to ward with the evidence are
establish that she took her check to the post on the plaintiff because the
office and sent it to the plaintiff's proper defendant's testimony rais-
address by certified mail. The defendant of- es a rebuttable presumption
fers a certified mail receipt with an illegible that the check had been de-
signature, which she claims is the plaintiff's livered in the mail. The bur-
signature. The defendant also presents evi- den of persuasion is the bur-
dence that her basement flooded on March den of a party to persuade
28, and she claims that she cannot produce the jury to decide an issue
a canceled check because her box of can- in its favor. If, after all the
171 / 184
Barbri MBE questions
Study online at https://quizlet.com/_6umlok
celed checks was destroyed from the water proof is in, the issue is equal-
damage. Evidence is also presented that, ly balanced in the mind of the
due to a computer glitch, the defendant's jury, then the party with the
bank cannot reproduce her checking ac- burden of persuasion must
count records for the months of February lose. The burden of persua-
and March. sion does not shift from par-
ty to party during the course
After the defendant's testimony, which of of a trial. Because the plain-
the following is correct? tiff sued the defendant for
the debt, the plaintiff has the
A The burden of persuasion and the burden burden of persuasion when
of going forward with the evidence are on the time for the jury to make
the plaintiff. a decision arrives. The bur-
den of going forward with
B The burden of persuasion is on the plain- the evidence is the burden
tiff, but he has no burden of going forward of producing sufficient evi-
with the evidence. dence to create a fact ques-
tion of the issue involved. If
C The plaintiff has satisfied his burden of a plaintiff makes out a pri-
persuasion, but he has a burden of going ma facie case, he has met
forward with the evidence. his burden of going forward
with the evidence and the
D The plaintiff has satisfied both his burden burden shifts to the defen-
of persuasion and his burden of going for- dant. Here, when the plain-
ward with the evidence. tiff made out a prima facie
case of the defendant's debt,
the burden of going forward
with the evidence shifted to
the defendant. The defen-
dant met this burden through
the use of a presumption.
Federal Rule 301 provides
that a presumption imposes
on the party against whom
it was directed the burden of
going forward with the evi-
dence to rebut the presump-
tion. The defendant's evi-
dence regarding the proper
172 / 184
Barbri MBE questions
Study online at https://quizlet.com/_6umlok
posting of the check raises a
rebuttable presumption that
the check was delivered to
the plaintiff because a letter
shown to have been proper-
ly addressed, stamped, and
mailed is presumed to have
been delivered in the due
course of mail. Therefore,
the burden of going forward
with the evidence has shift-
ed back again to the plaintiff,
who must now produce evi-
dence to rebut the presump-
tion (i.e., evidence that he
did not receive the check).

148. A shoe manufacturer entered into a con- (B)


tract to purchase all cowhide refined by a
tannery for the next five years at a price Yes, the modification is en-
set at 95% of the domestic market price forceable in its entirety. Al-
at the time of delivery. The shoe manufac- though there was no consid-
turer agreed to purchase no less than 500 eration for the shoe manu-
pounds of cowhide a week. At the time this facturer to take only one-half
contract was signed, the shoe manufacturer of the bargained-for produc-
gave written notice to the tannery that it tion, none is required be-
intended to buy all cowhide produced by cause this is a contract for
the tannery until further notice. For the first the sale of goods. All that
year, the shoe manufacturer continued to is required is good faith.
purchase all cowhide produced by the tan- Here, the shoe manufactur-
nery. However, by the end of that year, the er did not need twice as
tannery doubled its production of cowhide. much cowhide, and the tan-
At a meeting between the tannery and the nery could make more mon-
shoe manufacturer, the tannery's president ey selling the excess to oth-
noted that the shoe manufacturer was get- er parties. Thus, the modi-
ting as much cowhide as it needed, and fication was made in good
that the tannery intended to sell the extra faith. (A) is wrong because
cowhide it was producing on foreign mar- the enforceability of the mod-
kets at a higher price than the shoe manu- ification is not limited by
173 / 184
Barbri MBE questions
Study online at https://quizlet.com/_6umlok
facturer was paying. The shoe manufacturer the manufacturer's needs.
agreed to maintain its purchases at the first (C) is wrong because, as
year's level and signed an addendum to the discussed above, under the
original agreement reflecting this change. UCC, a good-faith modifi-
cation is enforceable even
Is the modification of this contract made by without consideration. (D) is
the addendum enforceable? wrong because the agree-
ment indicates the amount of
A Yes, as long as the shoe manufacturer's cowhide that the shoe man-
needs continue to be met by the first year's ufacturer will purchase—all
amount. cowhide refined by the the
tannery. Although no specific
B Yes, the modification is enforceable as quantity is mentioned, these
agreed. contracts are sufficiently def-
inite because the quantity is
C No, because there was no consideration capable of being made cer-
for the shoe manufacturer's agreement to tain by reference to objec-
take only one-half of the cowhide produced. tive, extrinsic facts (i.e., the
seller's actual output).
D No, because the contract did not state the
amount of cowhide that the tannery would
produce.

149. A three-year-old girl attending nursery (D)


school punched a boy in the face because
he was teasing her about wearing glasses. The best defense of the girl's
The blow knocked out the boy's newly ac- parents is that they were un-
quired front teeth. aware of any potentially vio-
lent behavior by the girl. At
If the boy's parents sue the girl's parents for common law, parents are not
the injury, what is their best defense? vicariously liable for the torts
of their child. (Statutes in
A The boy was the initial aggressor. most states allow for limited
liability for intentional torts,
B The girl is too young to be responsible for but there is no indication of
her actions. such a statute here.) Parents
can be liable, however, for
C A parent cannot be liable for damages due their own negligence, i.e., in
to the child's conduct. not exercising due care un-
174 / 184
Barbri MBE questions
Study online at https://quizlet.com/_6umlok
der the circumstances. Thus,
D The parents were unaware of any poten- if the parents know their child
tially violent behavior by the girl. may be violent, they could
be negligent if they do not
take precautions to prevent
that behavior or injury from
that behavior. However, if the
parents have no reason to
know their child could be vi-
olent, they have no duty to
protect against such behav-
ior. Here, if the girl had never
done anything like this be-
fore, and her parents had no
idea that she would be vi-
olent, they were not negli-
gent in allowing her to attend
nursery school. (A) is wrong
because, although the boy's
teasing may have provoked
the girl, he did not initiate
the violence. He did nothing
to allow the girl a right of
self-defense, and so his ac-
tions would not provide the
girl's parents with a good
defense. (B) is wrong be-
cause there is no gener-
al tort immunity for children.
As long as the child is old
enough to intend the act, she
can be held liable. Here it
seems that the girl intended
to cause a battery. She either
intended or knew with sub-
stantial certainty that swing-
ing her fist would strike the
boy in the face, i.e., would
cause a harmful or offensive
175 / 184
Barbri MBE questions
Study online at https://quizlet.com/_6umlok
contact. Thus, this choice
does not present the best
defense for the defendants.
(C) is wrong because par-
ents can be liable for dam-
ages due to their child's con-
duct. As explained above, al-
though the parents are not
vicariously liable at com-
mon law, they can be liable
based on their own negli-
gence (e.g., for negligent su-
pervision)

150. Acting with probable cause, the police ar- (C)


rested a man in connection with the armed
robbery of a liquor store. After being giv- The man's motion should
en Miranda warnings, the man confessed be denied because neither
to the robbery but denied his involvement his Fifth nor Sixth Amend-
with several other recent armed robberies ment rights were violated by
of businesses in the area. He was formally the informant's conduct. The
charged with the one robbery and put into Sixth Amendment right to
a cell with a paid informant working un- counsel applies to all critical
dercover for the police. The informant had stages of a criminal prosecu-
been instructed to find out what he could tion after formal proceedings
about the other robberies but not to ask have begun, but does not
any questions. The informant began talk- apply in precharge custo-
ing about a convenience store robbery in dial interrogations. Because
which a bystander was shot and seriously this right is "offense specif-
injured by the robber, and he deliberately ic," the fact that the right
misstated how it happened. The man, un- to counsel has attached for
aware that his cellmate was an informant, one charge does not bar
interrupted to correct him, bragging that he questioning without coun-
knew what really happened because he was sel for an unrelated charge.
there, and proceeded to make incriminating Because the man has not
statements about the robbery. The man was been charged with the con-
subsequently charged with armed robbery venience store robbery, his
and attempted murder in the convenience Sixth Amendment right to
store robbery. counsel has not been vio-
176 / 184
Barbri MBE questions
Study online at https://quizlet.com/_6umlok
lated. The Fifth Amendment
At a motion-to-suppress hearing on that privilege against self-incrim-
charge, if the man's attorney moves to ex- ination requires Miranda
clude the statements made to the informant, warnings and a valid waiver
should the motion be granted? before any statement made
by the accused during cus-
A Yes, because the informant deliberately todial interrogation can be
elicited incriminating statements in viola- admitted. However, this re-
tion of the man's Sixth Amendment right to quirement does not apply
counsel. where interrogation is by an
informant who the defen-
B Yes, because the informant's conduct con- dant does not know is work-
stituted custodial interrogation in violation ing for the police, because
of the man's Fifth Amendment privilege the coercive atmosphere of
against self-incrimination. police-dominated interroga-
tion is not present. [Illinois
C No, because the man had not yet been v. Perkins (1990)] Because
charged with the robbery of the conve- the man was not aware of
nience store when he made the statements the informant's status, the
to the informant. informant's conduct did not
constitute a police interro-
D No, because the informant's conduct did gation. (A) is wrong despite
not constitute interrogation. the fact that the informant's
conduct may have been de-
liberately designed to elic-
it incriminating remarks. As
discussed above, the man's
right to counsel did not at-
tach for purposes of the con-
venience store robbery. (B)
is incorrect because, as dis-
cussed above, the Miranda
warnings need not be giv-
en before questioning by a
cellmate working covertly for
the police. (D) is incorrect
because interrogation refers
not only to express question-
ing, but also to any words
177 / 184
Barbri MBE questions
Study online at https://quizlet.com/_6umlok
or actions on the part of
the police that the police
should know are reasonably
likely to elicit an incriminat-
ing response from the sus-
pect. Here, the informant,
working for the police, made
statements about the con-
venience store robbery that
were intended to, and rea-
sonably likely to, prompt a
response from his cellmate.
Hence, it is not the absence
of "interrogation" that avoids
the Miranda problem, but the
fact that the man did not
know that his cellmate was
working for the police.

151. While the defendant was committing a rob- (C)


bery, he shot and killed the victim. The de-
fendant is charged with first degree murder Because the defendant was
in a state that defines first degree murder charged with first degree
as murders committed with premeditation murder, the theory of the
or deliberation or during the commission case is most likely felony
of burglary, arson, rape, or robbery, and de- murder, and (C) is the only
fines second degree murder as all other choice that sets out a theo-
murders. The state also defines voluntary ry to avoid a felony murder
manslaughter as the unlawful killing of a hu- conviction. If the defendant
man being with malice upon a sudden quar- was so intoxicated that he
rel or heat of passion, and it defines invol- could not form the intent to
untary manslaughter as the unlawful killing steal, then he is not guilty
of a human being without malice in the com- of robbery, and there would
mission of an unlawful act, not amounting to be no "felony" from which
an enumerated felony, or in the commission the felony murder rule is to
of a lawful act that might produce death in arise. (A) and (B) are incor-
an unlawful manner or without due caution rect because even though
and circumspection. the defendant could argue
that no premeditation or de-
178 / 184
Barbri MBE questions
Study online at https://quizlet.com/_6umlok
Assuming evidence to support, what expla- liberation was present, he
nation for the shooting would best help the would still be guilty of first
defendant in avoiding conviction for first de- degree murder under the
gree murder? felony murder rule because
the felony was robbery. (D)
A In an act of resistance, the victim sudden- is wrong because insulting
ly attacked the defendant and knocked him someone is not adequate
down, so the defendant pulled the trigger provocation that would miti-
because he was afraid the victim was going gate a homicide to voluntary
to hit him again. manslaughter; neither would
this "threat" suffice, in all like-
B The defendant had the gun for many years, lihood. At most, the circum-
it was old and rusty, and he did not think it stances might produce the
would fire. sort of unreasonable anger
that would negate premedi-
C The defendant had taken "angel dust" be- tation and deliberation. How-
fore the incident and does not remember ever, because the defendant
getting a gun or holding up the victim. clearly caused the homicide
while committing the felony
D When the defendant tried to hold up the of robbery, his crime remains
victim, the victim said, "Get out of here, you first degree murder.
dirty bum, or I'll kill you," and the defendant
became so upset that he did not know what
he was doing.

152. A decedent died without having executed (C)


a will, leaving a substantial estate to be
distributed by the probate court. The juris- The court should admit the
diction's applicable statute provides that evidence. Statements in a
where a decedent leaves neither issue nor document affecting an inter-
spouse, nor parents, his estate goes to his est in property are admis-
brothers and sisters and their descendants. sible, pursuant to Federal
The decedent was never married, had no Rule 803(15), if they are rel-
children, and both of his parents are dead. evant to the purpose of the
A woman whose birth certificate was de- document. Thus, (B) is in-
stroyed by fire seeks to establish that she is correct. (A) is incorrect be-
the daughter of the decedent's only sibling, cause properly authenticat-
who is now also deceased. The woman of- ed copies of recorded writ-
fers into evidence a statement in a properly ings may be used in lieu
179 / 184
Barbri MBE questions
Study online at https://quizlet.com/_6umlok
recorded trust instrument. The instrument of originals. [Fed. R. Evid.
was executed by the decedent's father and 902(4)] (D) is incorrect be-
recited that certain specified real property cause the trust instrument
conveyed by the decedent's father into the cannot qualify as a recorded
trust should be held for her benefit, as "my recollection; there is no wit-
loving granddaughter." The document actu- ness testifying that he made
ally offered is an enlarged print photocopy or adopted the writing while
of microfilm records, authenticated by an the events were fresh in his
employee of the county. mind and he has no present
recollection.
What should the trial court do?

A Exclude the evidence, because it is not the


best evidence.

B Exclude the evidence, because it is inad-


missible hearsay not within any recognized
exception.

C Admit the evidence, because it is a record


of a document affecting an interest in prop-
erty.

D Admit the evidence, because it consti-


tutes a past recollection recorded.

153. An employee of the United States Depart- (A)


ment of Labor was instructed by his supe-
rior to solicit subscriptions to the Depart- The employee's best de-
ment's bulletin on a door-to-door basis in fense is intergovernmental
the city in which he worked. While doing so, immunity. State and local
the employee was arrested for violation of governments cannot tax or
a city ordinance that prohibited commercial regulate the activities of
solicitation of private residences. the federal government. This
principle is often termed "in-
What is the employee's best defense? tergovernmental immunity."
The arrest and prosecution
A Intergovernmental immunity. of a federal employee who
was on the job violates this
180 / 184
Barbri MBE questions
Study online at https://quizlet.com/_6umlok
B The First Amendment freedom of expres- principle, which is based on
sion as it applies to the states through the the supremacy of the fed-
Fourteenth Amendment. eral government and feder-
al law. (B) is not a bad an-
C The Equal Protection Clause as it ap- swer because door-to-door
plies to the states through the Fourteenth solicitation is protected by
Amendment. the First Amendment. How-
ever, at best, (B) would sub-
D The city ordinance effectively restricts in- ject the city's actions to strict
terstate commerce. scrutiny and allow the city
to prevail if it could prove
that its action was neces-
sary to achieve a compelling
government purpose. In con-
trast, (A) would automatical-
ly invalidate the city's en-
forcement of the law against
the employee, and so (A) is
a better answer. (C) is ir-
relevant because the Four-
teenth Amendment's restric-
tion on the states has to do
with persons, not the fed-
eral government, and here
there is no claim that the city
was discriminating against
the employee. The city's ordi-
nance, as briefly described,
does not seem to provide
the basis for an equal pro-
tection claim. (D) is wrong
because nothing in the facts
shows any burden on inter-
state commerce. Moreover,
at most such a claim would
trigger heightened scrutiny;
it would not automatically in-
validate the enforcement of
the law as would (A).
181 / 184
Barbri MBE questions
Study online at https://quizlet.com/_6umlok

154. A landowner conveyed his land "to my son (A)


for life, then to my son's widow for her life,
then to my son's children." At the time of the The granddaughter and the
conveyance, the son was 20 years old and city zoo each own one-half of
unmarried. The son eventually married and the land, subject to the sec-
had two children, the landowner's grandson ond wife's life estate. At the
and granddaughter. time of the conveyance by
the landowner, the son had
Many years later, the landowner and the a life estate, the son's wid-
grandson were involved in a train acci- ow had a contingent inter-
dent. The landowner was killed instantly. est (because the son's "wid-
The grandson died a short time later of his ow" cannot be ascertained
injuries. The landowner left his entire estate until the son's death), and
by will to his friend. The grandson's will de- the son's children had a con-
vised his entire estate to the city zoo. The tingent remainder (because
son's wife was so grief-stricken that she they have not yet been born).
became ill and died the next year, leaving When the grandson and the
her entire estate to her husband. granddaughter were born,
however, their interests be-
Eventually the son met and married a came vested subject to open
21-year-old. Ten years later, the son died, (i.e., if the son had more chil-
leaving everything to his second wife. When dren). Thus, when the grand-
the second wife moved onto the land, the son died, he had a vested
granddaughter filed suit to quiet title to the remainder subject to open
land, joining all of the appropriate parties. that he was free to devise
by will; the city zoo took his
If the jurisdiction recognizes the common vested remainder subject to
law Rule Against Perpetuities, unmodified open. At the son's death, the
by statute, in whom will the court most likely class of his "children" closed
find that title to the land is held? (because the son could not
have any more children),
A One-half in the granddaughter and and the granddaughter's and
one-half in the city zoo, subject to the sec- the zoo's vested remainders
ond wife's life estate. subject to open became in-
defeasibly vested. Also at
B One-half in the granddaughter and the son's death, his widow
one-half in the second wife, because the was ascertained and her in-
second wife took the son's interest. terest vested in possession.
Because the second wife
182 / 184
Barbri MBE questions
Study online at https://quizlet.com/_6umlok
C Entirely in the friend, subject to the sec-
was the son's widow, she is
ond wife's life estate, because the gift to entitled to the valid life es-
the son's children violates the Rule Againsttate. Thus, the granddaugh-
Perpetuities. ter and the city zoo hold
one-half interests, subject to
D Entirely in the granddaughter, subject to the second wife's life estate.
the second wife's life estate, because the (D) is wrong because the
grandson did not survive the son. gift to the son's children was
not conditioned on their sur-
vival of the son. The law does
not imply such a condition.
The grandson's interest was
vested subject to open and
could be disposed of by his
will.

155. Two neighbors owned summer homes ad- (D)


jacent to each other on the lake. After a
week-long stay by the son of one of the Modern courts would hold
property owners, the neighbor called the that a promise to forbear suit
owner and said that his boat dock had been on a claim that the promisor
badly damaged and was told by another res- honestly and reasonably be-
ident that the owner's son and some friends lieves to be valid is good
had gotten drunk and accidentally crashed consideration to support an
their boat into his dock. The owner was sur- agreement, even if the claim
prised at the accusation because he was ultimately turns out not to
sure that if his son had caused the damages, be valid. Hence, (A) and (B)
he would have told him. However, he did not are wrong. (C) is wrong be-
want to get into a dispute with his neighbor, cause mutual mistake is not
so he told his neighbor that he would have a defense when the adverse-
the dock repaired and pay for the repairs ly affected party bore the risk
if the neighbor agreed not to bring a claim that the parties' assumption
against his son for the damage to the dock. was mistaken. To be a de-
The neighbor agreed, and the owner hired a fense, it must be a true mis-
local carpenter to do the work. Later, how- take, not merely an uncer-
ever, the owner discovered that his son did tainty. Here, the owner al-
not damage the dock because the damages ways had the right to inves-
occurred after his son had returned to col- tigate the truth of the facts
lege.
183 / 184
Barbri MBE questions
Study online at https://quizlet.com/_6umlok
before he agreed to pay for
Is the owner obligated to pay for the re- the dock.
pairs?

A No, because the owner never really be-


lieved that his son caused the damage.

B No, because his son in fact did not cause


the damage.

C No, because the neighbor was wrong


when he accused his son of causing the
damage and it would be unfair to enforce
an agreement when there was a mutual mis-
take of fact.

D Yes.

184 / 184

You might also like

pFad - Phonifier reborn

Pfad - The Proxy pFad of © 2024 Garber Painting. All rights reserved.

Note: This service is not intended for secure transactions such as banking, social media, email, or purchasing. Use at your own risk. We assume no liability whatsoever for broken pages.


Alternative Proxies:

Alternative Proxy

pFad Proxy

pFad v3 Proxy

pFad v4 Proxy